Vous êtes sur la page 1sur 282

Q

& A in Internal Medicine


2nd Edition

Associate Professor Wong Yin Onn


Clinical School, Johor Bahru,
Monash University Malaysia

This eBook is a collation of questions that I ask my medical students at my


Bedside teaching sessions. Dr Chok Yin Ling, a brilliant young doctor keen
on a career in Internal Medicine, was given these questions on a daily
basis. This was to help her to prepare for her post graduate examinations.
These questions and answers were edited and documented in her digital
diary; this eBook is the result. I hope that this work which is FREE to all
will help students at both undergraduate and post graduate levels.
I dedicate this eBook to all the teachers of this ancient art.

dr wong yin onn


MBBS(Mal), MRCP (UK), AM, FRCP(Glas)
22 March 2014

Foreword by Dr Ng Kian Seng, the founder of Aequanimitas



The Dear Yin Ling Series... The words are of course not to be taken
literally... "Dear" is the heart attitude that Prof Punna Wong brings to his
teaching, he loves his students...

Yin Ling is a real name but of course it is also a metaphor for the HO,
MO, Consultant, Medical student who is passionate for medicine and
patient... please do not allow overworked, underpaid Yin Ling to be the
sole person interacting with Prof Wong... think of that... I am also a Yin
Ling (older, male and fatter version) and Prof Wong is also addressing me
when he says, Dear Yin Ling...and Yin Ling should be proud that Prof
Wong has used her name...as a metaphor

I love the title and so do many of the people in Kluang and perhaps
around the world!

Dear Yin Ling,


Osler said that to succeed you must Have a Calling, The Calling to be the
BEST Physician that you can be, the most compassionate doctor available,
and the most dedicated student soaking up all that your seniors can
teach.

Even after he was well established in medicine, Osler was urged to
consider other career paths eg university presidency and politics, but he
always declined. Thank goodness for that. Politics would have killed him
off early.

In perhaps his greatest speech to a medical student body, titled
Aequanimatus, he spoke of having found his calling. You must have the
same calling that will push you to study till the wee hours, to see patients
till you drop from exhaustion, to learn like your life depends on it, and
hopefully to also teach your juniors like a woman possessed.

To prevent disease, to relieve suffering, and to heal the sickthis is our
work. The profession in truth is a sort of guild or brotherhood, any
member of which can take up his calling in any part of the world and
find brethren whose language and methods and whose aims and ways
are identical with his own."

Yin Ling :
Prof spent the whole weekend editing the book by himself, sentence by
sentence and he has added in alot of important topics nearing the end of
the book, information that he kept reiterating but has not been featured
in the series. Do read!

Thank you Prof for spending the last few months dedicatedly teaching all
of us here, passing on what u know to us a little bit each day. For me who
is doing my orthopedics posting during these teachings.. most of the
days these teachings has been the only source of learning- ''thank
goodness Im actually learning something from a consultant today".

10 Sept 2014
Today is Yin Lings last day of work in Hospital Sultanah Aminah in Johor
Bahru.
She will be leaving to work as a Medical Officer in Penang soon and will
continue her training up north which is near to her home. She had spent
the last 2 years in Johor Bahru where I had the blessing of interacting
with her almost daily; this has been a blessing and honour for me. I had
learnt MORE in teaching her than she had from me.
I wish her all the very best and extend to her my Blessings and Love.
It was and will continue to be my honour to have taught you.
Today I shall put in the last entry into the 2nd edition of the Dear Yin Ling
eBook. Much has been added since it was first compiled. I hope this
eBook will help the students and doctors be better clinicians.
Sukhi Hotu dear Yin Ling!

prof

Questions
1) ON H.PYLORI
Dear Yin Ling,

What can we use in HP resistant to Amox/clarythro/PPI combination treatment?

The issue is what to do with treatment failures. The success rate for the current
combination is assumed to be over 85%. It is likely that the success rates are
gradually decreasing as macrolide resistance becomes more prevalent. There is
some cross-resistance to clarithromycin from the use of other macrolides.

After failure of a combination of PPI, amoxicillin and clarithromycin, a theoretically
correct alternative would be the use, as second option, of other PPI-based triple
therapy including amoxicillin (that does not induce much resistance) and
metronidazole (an antibiotic not used in the first trial). However in practice this
approach as second-line treatment has proven to be disappointing (approx 50%
eradication rate).

Levofloxacin-based 'rescue' therapy appears to be the best second-line strategy,
representing a good alternative to quadruple therapy in patients with previous PPI-
clarithromycin-amoxicillin failure this treatment has higher efficacy, simplicity of
use (better compliance) and less adverse effects. Levofloxacin has, in vitro,
remarkable activity against H. pylori and primary resistance is relatively infrequent
(when compared with metronidazole or clarithromycin).

A combination of a PPI, amoxicillin and levofloxacin, as first-line regimen, has
mean eradication rates of about 90%. For patients with one previous eradication
failure, H. pylori cures rates range from 60% to 94%. A recent systematic review
showed a mean eradication rate with levofloxacin-based 'rescue' regimens
(combined with amoxicillin and a PPI in most studies) of 80%, which represents a
relatively high figure when considering 'rescue' therapy will have have eradication
rates lower than first-line treatments. A systematic review found higher H. pylori
cure rates with 10-day than with 7-day regimens with the levofloxacin-amoxicillin-
PPI combination in particular (80% versus 68%), suggesting that the longer (10-
day) therapeutic scheme should be chosen. The daily dose is still unclear but
500mg daily may be as effective as 500mg bd.


Please check with Urea Breath Test post treatment for cure!


The problem of Resistance
While Triple therapy, amoxicillin and clarithromycin, and a proton pump inhibitor
given for a week has been recommended as the treatment of choice, this treatment
may fail for several reasons, the main reason for failure being H pylori resistance to
clarithromycin. For children from all European countries a high prevalence has been
reported, ranging from 12.4% to 23.5%. The essential risk factor for clarithromycin
resistance is previous consumption of macrolides, and if resistance is higher in
children it is because there was increased prescription of these drugs during the last
decade essentially for respiratory tract infections.

Metronidazole similarly has resistance problem as well as tetracycline,


fluoroquinolones, and rifamycins for which resistance has become an emerging
issue. The prevalence of H pylori resistance to metronidazole varies from 20% to
40% in Europe and the USA. Prevalence is much higher in developing countries (50
80%), for example Mexico (76.3%).
Fluoroquinolones: Only one country, Portugal, has reported a high resistance rate:
20.9% in strains isolated from 110 adult patients. In France, a rate of 3.8% was
reported between 1993 and 1999. In five Eastern European countries the rate was
also 3.9%.
Again, resistance to fluoroquinolones mirrors the use of these drugs. Despite a high
rate of resistance in adults in Portugal, children who were not treated with
quinolones did not harbour any resistant strains.
Resistance to amoxicillin is either null or less than 1%, indicating that it is not yet a
problem.


2) ON LIPIDS
Dear yin ling,
High cholesterol plus high Triglycerides is common clinical problem. Statins alone
often do not return one to normal physiology.

WHAT can you offer?

http://www.ncbi.nlm.nih.gov/m/pubmed/23324122/
Adverse events following statin-fenofibrate therapy versus statin alone: a metaanalysis of randomized controlled trials.
Geng Q, et al. Journal Clin Exp Pharmacol Physiol. 2013 Mar;40(3):219-26. doi: 10.1111/1440-1681.12053.


Abstract
The combination of fenofibrate with statins is a beneficial therapeutic option for
patients with mixed dyslipidaemia, but concerns about adverse events (AEs) make
physicians reluctant to use this combination therapy. Medline, Embase and the
Cochrane Library were searched to identify 13 randomized controlled trials,

involving 7712 patients, of statin-fenofibrate therapy versus statin alone for review.
There were significant decreases in low-density lipoprotein-cholesterol, triglycerides
and total cholesterol and increases in high-density lipoprotein-cholesterol in
patients receiving combination therapy compared with statin therapy alone. The
incidence of aminotransferase elevations in the fenofibrate-statin therapy group
was significantly higher than in the statin monotherapy group (odds ratio (OR), 1.66;
95% confidence interval (CI) 1.17-2.37; P < 0.05). The incidence of elevated creatine
kinase levels (OR 0.88; 95% CI 0.63-1.23; P > 0.05), muscle-associated AEs (OR 0.98;
95% CI 0.88-1.09; P > 0.05) and withdrawals attributed to liver and muscle
dysfunction did not differ significantly between the two groups. The efficacy of
fenofibrate + standard-dose statin and incidence of AEs in the fenofibrate +
standard-dose statin group were almost identical to those in the fenofibrate-statin
group. In conclusion, combination therapy improves the blood lipid profile of
patients. Fenofibrate-statin combination therapy appears to be as well tolerated as
statin monotherapy. Physicians should consider fenofibrate-statin combination
therapy in patients but monitor aminotransferase levels to avoid hepatic
complications.

YL: i learnt that it's also important to realize that both alcohol and insulin resistance
can give a high TG. insulin resistance caused the breakdown of triglycerides and
release FFA. one dietary advice is to cut down simple carbs and sugars to control TG.

TG will also increase SMALL, DENSE LDL particle, aka LDL3, which is highly
atherogenic compared to LDL1 which is larger and less dense. we cannot measure
both of them, hence the surrogate marker of high TG and low HDL reflects the
small and dense LDL3 particle. by controlling TG with a fibrate, we make the small
dense LDL become bigger less dense LDL, and decrease the overall atherogenic risk,
despite no change in LDL levels.

We rarely use gemfibrozil now.

Only fibrate and niacin increase one's HDL. but niacin makes one look like a crab-
flushing!!

We may combine with using a statin at night and fenofibrate in the morning.

PROF : Under what conditions will we still use gemfibrosil?



Many of our patients on follow up have renal impairment of various stages. They are
commonly monitored by eGFR which is convenient vs 24h urine for CCl.

Fenofibrate increases creatinine levels significantly hence a calculated eGFR will be
falsely lowered. Gemfibrosil avoids this effect.

http://www.ncbi.nlm.nih.gov/m/pubmed/12372935/
Fenofibrate increases creatininemia by increasing metabolic production of
creatinine.
Hottelart C, et al.

Journal Nephron. 2002;92(3):536-41.

Abstract
Fenofibrate is a potent hypolipemic agent, widely used in patients with renal
insufficiency in whom dyslipidemia is frequent. A moderate reversible increase in
creatinine plasma levels is an established side effect of fenofibrate therapy, which
mechanism remains unknown. We have previously reported that in 13 patients with
normal renal function or moderate renal insufficiency, two weeks of fenofibrate
therapy increased creatininemia without any changes in renal plasma flow and
glomerular filtration rate [1]. In 13 additional patients, muscular enzymes (AST, GPT,
CPK, LDH) and myoglobin were measured before and after 2 weeks on fenofibrate,
and the values of creatininemia obtained by the Jaff technique and HPLC were
compared. CPK and AST activity and plasma myoglobin increased in 2 patients with
fenofibrate, but muscular enzymes remained unchanged in the population as a
whole, and were not correlated to the changes in creatininemia. The changes in
creatininemia induced by fenofibrate measured by the Jaff technique were strongly
correlated to those measured by HPLC (r(2) = 0.675, p = 0.0006). Analysis of the
pooled data of the two arms of the study showed in 26 patients that two weeks of
fenofibrate therapy efficiently reduced total cholesterol and triglycerides plasma
levels, and raised creatininemia from 139 +/- 8 to 160 +/- 10 micromol/l (p <
0.0001), but confirmed that creatininuria also increased to the extent that creatinine
clearance remained unchanged (68 +/- 6 vs. 67 +/- 6 ml/min, n.s.). It is concluded
that the increase in creatininemia induced by fenofibrate in renal patients does not
reflect an impairment of renal function, nor an alteration of tubular creatinine
secretion, and is not falsely increased by a dosage interference. Fenofibrate-induced
increase of daily creatinine production is neither readily explained by accelerated

muscular cell lysis. It is proposed that fenofibrate increases the metabolic


production rate of creatinine.

Dear Yin Ling,
the most impt reminder I wanted to share is that doctors should switch all patients
taking simvastatin 40mg and amlodipine to a lower dose of statin, or another statin
altogether, because of the risk of muscle damage. Not many are aware that
Amlodipine when combined with simvastatin INCREASES its bioavailability. Simva
20mg is the maximal dose when used with Amlodipine. In view of the recent AHA
guidelines on using high doses when managing high risk patients, we must be
mindful not to overdose.
Simvastatin is an HMG-CoA reductase inhibitor that is metabolized by the
cytochrome P450 (CYP) 3A4. The problem is that amlodipine is also metabolized by
CYP3A4.
Athovastatin is metabolised by the same enzyme but there were no interactions
found in database between amlodipine and athovastatin!
Coadministration with amlodipine may significantly increase the plasma
concentrations of simvastatin and its active metabolite, simvastatin acid, and
potentiate the risk of statin-induced myopathy.

The proposed mechanism is amlodipine inhibition of simvastatin metabolism via
intestinal and hepatic CYP450 3A4.

When a single 80 mg dose of simvastatin was administered on day 10 of amlodipine
given at a dosage of 10 mg once daily, simvastatin peak plasma concentration
(Cmax) and systemic exposure (AUC) increased by an average of 1.5- and 1.8-fold,
respectively, while simvastatin acid Cmax and AUC increased by an average of 1.6-
fold each.

Simvastatin dosage should not exceed 20 mg daily when used in combination with
amlodipine. The benefits of this combination should be carefully weighed against
the potentially increased risk of myopathy including rhabdomyolysis.

Fluvastatin, pravastatin, and rosuvastatin are probably safer alternatives in patients
receiving amlodipine, since they are not metabolized by CYP450 3A4.

All patients receiving statin therapy should be advised to promptly report any
unexplained muscle pain, tenderness or weakness, particularly if accompanied by
fever, malaise and/or dark colored urine. Therapy should be discontinued if creatine
kinase is markedly elevated in the absence of strenuous exercise or if myopathy is
otherwise suspected or diagnosed.
Yin Ling,
The present emphasis on high intensity treatment for patients with DM, HPT AND
CVS DISEASE will make us automatically write higher dosages. The interaction here
saves money for some and puts others at risk
Lipid treatment based on the 2013 AHA guidelines had a shift in focus.
The 3 groups of known CV disease, DM, and high LDL > 190 requiring treatment is
not controversial.
A High intensity statin is one that reduces LDL by more than 50%. At risk patients
require such aggressive lipid lowering.
Ezitimide can be used in those with LDL more than 190 to achieve >50% lowering
Is ldl targets relevant anymore? Now we are treating the CV risk rather than the ldl
value.
Where did the ldl targets originally come from?
Trials show that the lower the ldl the lower the CV event
The quantity of the ldl value is made in looking back at the data. The emphasis was
not on the statin dose that achieved the lowering.
In effect, the lowering of the CV event may be more from the intense statin
treatment than the actual ldl value.
Previous guidelines emphasise ldl targets and NOT STATIN DOSES. You were left to
decide which dose! Present AHA guidelines does the reverse! And NEW ONSET DM
is not an issue. 255 patients treated for 4 years will theoretically have ONE
additional diabetic.

What is the risk of Rhabdomyolysis?
In 252,460 patients treated with lipid-lowering agents, 24 cases of hospitalized
rhabdomyolysis occurred during treatment. Average incidence per 10,000 person-
years for monotherapy with atorvastatin, pravastatin, or simvastatin was 0.44 (95%

confidence interval [CI], 0.20-0.84); for cerivastatin, 5.34 (95% CI, 1.46-13.68); and
for fibrate, 2.82 (95% CI, 0.58-8.24). By comparison, the incidence during unexposed
person-time was 0 (95% CI, 0-0.48; P = .056). The incidence increased to 5.98 (95%
CI, 0.72-216.0) for combined therapy of atorvastatin, pravastatin, or simvastatin
with a fibrate, and to 1035 (95% CI, 389-2117) for combined cerivastatin-fibrate use.
Per year of therapy, the number needed to treat to observe 1 case of
rhabdomyolysis was 22,727 for statin monotherapy, 484 for older patients with
diabetes mellitus who were treated with both a statin and fibrate, and ranged from
9.7 to 12.7 for patients who were treated with cerivastatin plus fibrate.
CONCLUSIONS: Rhabdomyolysis risk was similar and low for monotherapy with
atorvastatin, pravastatin, and simvastatin; combined statin-fibrate use increased
risk, especially in older patients with diabetes mellitus.

If we have a patient with DM and HPT who have symptoms typical or suggestive of
IHD pls treat aggressively. All such patients benefit from high dose statin, in that we
are not in doubt. Simva is cheap in Msia and is commonly used. We cannot afford
half hearted treatment in high risk patients. And Diabetics are high risk. My late
mother lived in an era without statins. Her DM was treated with an SU full stop.
Nothing else. By her 50s her coronaries etc were all blocked. Our present medicines
allow us to offer better lives for our diabetics. Let us not deny them this.
Paul Ling Kah Hing: even lower at 10 mg in combo with other ca antagonist like
verapamil and diltiazem. No drug rep ever mentioned this labelling
Yia Hua Jern : additional caution needs to be taken in chronic kidney disease,
especially in terms of myositis & rhabdomyolysis.
Combination of a statin with a fibrate is very high risk.


3) ON CANTLIES LINE
Dear Yin Ling,

Who is Sir James Cantlie and why is his contribution not only to medicine but also
to Asia and the world?

Cantlie's line is named in his honour. What is this line and how is it so important to
our understanding of the treatment of liver tumours? Everyday in my work I look

out for diseases and carefully see if it is to the left or right of Cantlie's line and hence
potentially treatable.

Dr James Cantlie was a Foundation Professor at the primordial College of Medicine
in HongKong (later to become the Faculty of Medicine at the University of
HongKong). In his very first batch was a young brilliant Chinese lad named Sun Yat
Sen. Teachers of old like some teachers today loved their students like their own
children (following the Hippocratic oath the relationship is akin to a Father-
Son/Daughter bond).

It was also in HongKong that he did his seminal work on the liver. Years later when
Dr Sun Yat Sen the revolutionary was kidnapped by the Manchurian regime in
London and almost definitely headed back home for his head to be separated from
his neck, it was Sir James Cantlie that Dr Sun turned to for help and he organised a
press campaign and much publicity that ultimately pressured the Manchurian
authorities to release Dr Sun, once again proving the power of the pen vs the sword.

Dr Sun subsequently went on to be the Father of Modern China.

http://www.ncbi.nlm.nih.gov/pmc/articles/PMC2826664/
Thomas M van Gulik and Jacomina W van den Esschert

Abstract
As early as 1897, Sir James Cantlie published a series of observations of
extraordinary significance in the face of how we now look upon portal blood supply
and the pre-resectional use of portal vein ligation or embolization to induce
hypertrophy of the part of the liver we intend to preserve. In the Proceedings of the
Anatomical Society of Great Britain and Ireland, he describes performing an autopsy
on a patient in which the right side of the liver was reduced to a mass of fibrotic
tissue whereas the left side of the liver showed a marked hypertrophy.1* He noted
that the hypertrophy of the left side joined with the atrophied right side, at a line
drawn through the fundus of the gallbladder to the center of the inferior vena cava
at the back of the liver. He assumed that an abscess had destructed the right lobe of
the liver, and that this resulted in a compensatory hypertrophy of the contralateral
part of the liver. Hence, he concluded that the line connecting the fundus of the
gallbladder with the centre of the inferior vena cava indicated the mid-line of the
liver, unlike common opinion at that time which considered the umbilical fissure as
the division of the right and left liver lobes.

He corroborated his observations by performing experiments in which he injected


the right and left divisions of the portal vein with coloured dyes showing that the
injected areas met along a line connecting the fundus of the gallbladder with the
spot where the inferior vena cava grooves the back of the liver. This line we still
refer to as Cantlie's line (Fig. 1) indicating the anatomical mid-line of the liver and
defining the border between the right and left liver segments in the plane of the
middle hepatic vein. As he realized that the right and left liver were perfused by two
separate streams of the portal vein, he recognized the potential this phenomenon
could have for hepatic surgery.

Figure 1
Cantlie's line represents the anatomical mid-line of the liver connecting the fundus
of the gallbladder with the centre of the inferior vena cava.
He perceived the consequences the watershed between the right and left liver lobes
could have for trauma of the liver by writing The liver, when split or fissured by a
blow, as between the buffers of railway-carriages, splits along the mid-line of the
liver in preference to any other. He also foresaw that this would not necessarily
result in major bleeding as haemorrhage has less to be dreaded as the liver is
incised or torn in the neighborhood of that line (i.e the mid-line). Indeed in blunt
abdominal trauma, the liver may be completely fractured along Cantlie's line
without any major bleeding from that plane. We were able to confirm this message
recently in a patient admitted after blunt abdominal trauma who had fractured his
liver along Cantlie's line (Fig. 2) and who had been successfully managed by
conservative treatment without the need for any blood transfusion.

Figure 2
Contrast enhanced abdominal computed tomography (CT) scan of a patient
admitted after blunt abdominal trauma showing a fracture of the liver along
Cantlie's line, running between the medial borders of segment IV and segments V/
VIII.
Coming back to his initial observation at the autopsy, he noted the almost
elephantine hypertrophy of the left side of the liver at the expense of a greatly

atrophied right side which looked like, and practically was, a mere appendage to the
left side of the organ. On dissection of the liver he found the veins, artery and duct
of the right side of the liver to be obliterated whereas those to the left side were
proportionally increased in diameter. From this observation he conceived that by
eliminating blood supply to one side of the liver, a functional advantage for the
spared half of the liver could be created resulting in hypertrophy of that part of the
liver. He then wrote It is theoretically possible to tie the vessels of one side at the
gate of the liver, supplying an abnormal growth in one or other of the liver lobes,
leaving the other side to do the work. Realizing the importance this phenomenon
could have for resecting the liver, he continued I commend this subject to all those
who are working at the surgery of the liver; and I believe that if, in the hands of
future observers, the statements I have made receive closer investigation, the
surgery of the liver will be advanced a step. The foresight he had was amazing, with
the first formal right hemihepatectomy being performed 55 years later in Beaujon
Hospital in Paris and the first clinical portal vein embolization being performed in
Japan 85 years after his report.
Sir James Cantlie (Fig. 3) was born in 1851 in Banffshire, Scotland. After finishing his
medical studies at Aberdeen University, he trained as a surgeon at Charing Cross
Hospital in London. He became a fellow of the Royal College of Surgeons in 1877 and
went on to work as a surgeon at Charing Cross. Interestingly, in 1887 he moved to
Hong Kong where he became a co-founder of a new medical school, the Hong Kong
College of Medicine for Chinese, the forerunner of the Faculty of Medicine of the
University of Hong Kong. In this institution, of which he led the surgical department,
Cantlie carried out the autopsy described above. One of his students was SunYat Sen
who would later become the first provisional president of the Republic of China.
When this Chinese leader was detained at the Chinese Legation in London in 1896,
Cantlie played a key role in his release. In 1897 Cantlie returned to practice in
London.

Figure 3 Sir James Cantlie (18511926)

The division of the portal vein into a right and left branch at the liver hilum was
already reported by the anatomists of the 17thcentury. Francis Glisson (15981677)
in his textbook Anatomia Hepatis described cannulating the portal vein and making
casts of the portal venous system. Cantlie, however, showed that by the separate
portal vasculature, the liver could be functionally divided into an anatomically
distinct right and left half. The potential of one half of the liver to hypertrophy when
the other half is deprived of its blood supply was further confirmed in experimental
studies by Rous and Larimore in 1920 and Schalm and colleagues in 1956. The latter
authors from Arnhem, the Netherlands, made reference to Cantlie's work and ideas
on unilateral occlusion of the portal vein. Surprisingly, portal vein occlusion found its
way to clinical application only in 1982, when Makuuchi and later, Kinoshita,
published their first experiences with portal vein embolization in patients. Hence,
although the credit for the first clinical portal vein occlusions goes to these
colleagues in Japan, it should be remembered that in 1897, James Cantlie from
Scotland had already laid down the concept of pre-operative portal vein occlusion.
YL: Cantlie's Line delineates the SURGICAL anatomy of the liver, a line connecting
the fundus of the GB and the IVC, separating the liver into the surgical Left and
surgical Right lobe. while, the Falciform Ligament we see from the anterior aspect of
the liver is just the anatomical line, nothing to do with its function nor help with
surgery.

You often look for the hepatic veins on USG while placing your probe subcostally. It's
the one with thin walls as opposed to the thicker walls of the portal veins. The
middle and right hepatic vein join the IVC, showing us the 'Playboy Sign'. A thin
playboy if its normal. When you see lesions suspicious of HCC/mets, if they are on
one side of the Cantlie's line, there is hope of tumour resection for the patient. if it
crosses both lobes, unfortunately we will have to prepare the patient for the worst.

4) ON THALASSEMIA
Dear yin ling,

Yesterday a student told me that her mother has heavy menses and is chronically
anaemic. Low ferritin confirmed. The student herself looked a bit pale too.
I asked if she has any family history of Thalassemia. And she replied that Screening
for Thalasaemia for her mother was done at the same time. And it is 'normal'.

What are your concerns as a consultant on knowing this and what will you advise?
YL: Thalassemia cannot be screened when there is concurrent iron deficiency
anemia.. HbA2 will be FALSELY NORMAL. We have to correct the iron deficiency
first. Repeat ferritin- normal range only then we can do a thalassemia screening.
Prof : Iron deficiency is common in adult women. It is a serious potential
complicating factor when testing for a thalassaemia carrier state. Both iron
deficiency and a thalassaemia carrier state may result in a low MCV and MCH. But
the MCV in thalasaemia is generally very low. Erythrocytosis is more likely to be
caused by thalassaemia, but it is not a diagnostic finding.
The diagnostic criterion for beta thalassemia trait (BTT) is elevated Hb-A2 levels.
Iron deficiency anemia (IDA) reduces the synthesis of Hb-A2, resulting in reduced
Hb-A2 levels, so patients with co-pathological conditions BTT with IDA, may have a
normal level of Hb-A2.
Many socio-economic factors like worm infestation, poor diet, multiple pregnancies,
doctor unawareness result in interpretation of these subjects as simply iron
deficiency anaemia or worse as normal.

PROF : What is the danger of just giving iron to patients undiagnosed or
misdiagnosed as iron deficiency? Tell me about Hepcidin and its role in iron
physiology. What happens in Thalasaemia carriers?
YL : Hepcidin regulates the iron absorption in our body. Hepcidin inhibits iron
absorption. Low hepcidin encourage iron absorption and vice versa. When there is
increase erythropoiesis.. hepcidin will be low so that absorption of iron is increased,
and vice versa. During inflammation, hepcidin also an acute phase reactant is high,
iron absorption is less so contributing further to anemia of chronic diseases.



Beta thal carriers do not have severe anemia but they have increase in
erythropoiesis and low hepcidin levels. They can have iron overload from all the
increased iron absorption.
When we have not diagnosed iron deficiency anemia, giving iron to pt who has
other diseases such as beta thal trait may cause iron overload! More so with their
tendency to have an increased iron absorption. We are doing more harm than good
to them
PROF : It is uncommon to transfuse patients with BTT unless their Hb is
unacceptably low. Most of them adapt very well to the chronic anaemia.
Furthermore blood transfusion also transfuses iron besides the risk of blood borne
diseases. Having said that I am aware of some BTT female patients who require
blood transfusion. They likely have beta thalasaemia intermedia
The term "thalassemia" is derived from the Greek root words for "anemia" and
"sea" because the thalassemia syndromes were initially believed to be restricted to
populations around the Mediterranean Sea
In most adults, 97% of the hemoglobin produced is hemoglobin A (HbA), which has
two alpha-globin chains and two beta-globin chains. The remaining 2% to 3% of
adult hemoglobin is hemoglobin A2, which is composed of a pair of alpha-globin
chains and a pair of delta-globin chains. In some adults, fetal hemoglobin HbF,
which is composed of two alpha- and two gamma-globins, may continue to be
produced, but it does not typically exceed 2% of the hemoglobin production

Thalassemia intermedia is a term to describe patients with beta-thalassemia in
whom the clinical severity falls between the minor and major forms. The minor
forms, tend to be clinically mild, if not asymptomatic

There are two beta-globin genes controlling the production of beta-globin, one on
each copy of chromosome 11. The beta-globin gene may have a mutation that
results in the production of no beta-globin, noted as [beta]0, or it may have a
mutation that results in reduced production of beta-globin, noted as [beta]+. Beta-
thalassemia trait occurs when a person acquires a normal beta-globin gene and a
thalassemic beta-globin gene, or two thalassemic beta-globin genes that still
produce minimal to moderate amounts of beta-globin chains

Survival advantage
The RBC in a patient with beta-thalassemia trait is more rigid and dehydrated than a
normal RBC.

Due to similarities in the distributions of malaria and the thalassemias, it was
hypothesized that the development of thalassemia traits offered some protection
in malarial infection. Alpha-thalassemia may offer some general protection against
severe malaria. Beta-thalassemic patients may be protected from malaria by an
enhanced phagocytosis of the early intraerythrocytic form of malaria, called rings, in
beta-thalassemic cells.

Thalassemia trait may be manifested by pallor, fatigue, or other nonspecific
complaints associated with anemia. There may be a family history of anemia; often
it has been mistakenly diagnosed as iron deficiency. The family's ethnic origin may
be suggestive of a thalassemia trait if they are from the Mediterranean region, or
Southeast Asia. There is frequently a marked microcytosis, with a LOW mean
corpuscular volume, and mild to moderate hypochromia of the RBCs. The
characteristic RBC count index findings for beta-thalassemia trait are a high RBC
count, mild anemia, and microcytic, hypochromic cells. The mean corpuscular
hemoglobin concentration (MCHC) tends to remain in the normal range of values.
Mild splenomegaly may be found in people with beta-thalassemia trait

Diagnosing the thalassemia traits can be difficult because the lab values may mimic
iron deficiency; there may even be concurrent iron deficiency. If a patient has a low
MCV, a serum ferritin level should be obtained.

If the ferritin is low, the iron deficiency should be corrected, and the MCV
reinterpreted afterward. If the ferritin is normal, a hemoglobin electrophoresis will
identify hemoglobins A, F, and a number of other hemoglobin variants, along with
the estimation of the HbA2 level. People with an elevated level of HbA2 along with
hypochromic, microcytic RBCs have a beta-thalassemia trait . If the HbA2 is

borderline high to normal, it is likely that they have alpha-thalassemia or a


combination of alpha-beta thalassemia. Alpha-thalassemia trait is often regarded
as a diagnosis of exclusion because the hemoglobin electrophoresis does not
definitively prove it is an alpha-thalassemia trait. The definitive testing for alpha-
thalassemia traits would be genetic testing that can determine the exact number
of deletions of the alpha-globin genes. These tests, however, are expensive and
not readily available.

Please always remember that Iron deficiency may obscure the results of the
hemoglobin electrophoresis if thalassemia trait is present as it falsely normalize
the level of HbA2
Dear Yin Ling

are you aware of experimental treatments in Thalassaemia offering a possible cure.
The experimental treatments include inducing the body to produce fetal
hemoglobin again as opposed to adult hemoglobin, and the introduction of gene
therapy. The treatments currently being tried are bone marrow or umbilical stem
cell transplants.

Fetal hemoglobin changes to adult hemoglobin later in childhood. It was noted that
infants born with sickle cell anemia showed no signs of sickling until the fetal
hemoglobin was replaced. It was extrapolated that if a child with thalassemia could
have their adult hemoglobin replaced by fetal hemoglobin once again, this would be
beneficial. A search began to find a drug that could reverse the neonatal switch in
hemoglobins. This would need to be a drug that changed gene regulation called a
hypomethylator.

A drug was found (5-azacyidine) that worked well but caused severe side effects. We
await new drugs.

It is hoped that eventually thalassemia will be treated by replacing the defective
globin gene with a normally working gene

Presently the treatment of choice is stem cell transplant. Healthy stem cells sources
include bone marrow and umbilical cord blood from healthy donors. Umbilical cord
blood is a more readily available source of stem cells. The recent movement of
umbilical stem cell collection at births is building up banks of possible tissue
matches. The goal of stem cell transplantation is similar to gene therapy, ie replace
the defective blood cells found in thalassemia with healthy normal blood cells.

As far as exams is concerned, no discussion will be complete without mention of


treatment BEFORE conception. This is particularly important in high prevalence
regions.

Both parental genetic screening and preimplantation diagnosis from a single
blastomere in in-vitro fertilization have totally changed the way of life in some
countries with previously high numbers of children with thalassemia. Throughout
the Mediterranean, education campaigns have taken place for entire countries,
especially targeting secondary school teens. Education and free thalassemia clinics
provide choices about birth control, mate selection, adoption, fetal testing, artificial
insemination by donor, and abortion. Even the conservative Greek Orthodox Church
agreed to require that all couples applying for a marriage certificate be pretested for
thalassemia carrier trait. Abortion, which was previously illegal, has become legal in
pregnancies diagnosed with thalassemia.

Needless to say such steps have their proponents and opponents.

5) Yin Ling,
A 25-year-old Chinese woman was referred for review of her anaemia. She is
pregnant about 12 weeks gestation at the time of presentation. G1P0
Asymptomatic. Planned pregnancy.

She was noted by her GP to be pale when she went for her UPT. She does not
recall any family members who are pale or require regular transfusion. Her
menses prior to pregnancy was unremarkable. She eats like a true Malaysian with
a "see food, eat food" diet. No bleeding noted.

Clinically she appears pale but no koilonychia or glossitis is noted. She is not
jaundiced or sallor or have any abnormal facies. No splenomegaly.

Her initial full blood count from her GP showed a microcytic, hypochromic
anaemia (MCH, 25.6 pg/cell, MCV 68) with a haemoglobin of 10.2 g/dL. The GP had
started ferrous sulphate tablets at 200 mg three times daily for 4 weeks and asked
for a repeat full blood count when the tablets had finished. The repeat full blood
count results show a deterioration of the anaemia as below.

Haemoglobin 9.8: Platelets: 384
White cell count: 10100 MCH: 26 MCV 65

At this point the GP asked for a consult.



Questions
Why did the iron tablets not help?
What is the significance of this result?
How might you explain the different results for the two full blood counts?

What will you do now for this patient?
Prof : Dear yin ling,
dietary iron comes in different forms, the percentage of dietary iron absorbed
depends on the type of food we eat and what other foods are being eaten at the
same time. For example, iron from meat is easier for the body to absorb than iron
from vegetable.
In addition, iron absorption can be greatly increased or decreased by various factors.
Chemicals called polyphenols in tea, coffee, cocoa, spinach inhibit iron absorption as
well. Eating more ascorbic acid, which is common in fruits, vegetables and fortified
foods, can improve iron absorption. But Calcium inhibits the absorption of iron by an
unknown mechanism. This is probably why there is a correlation between high milk
intake and iron deficiency.

If indeed this patient or any patient has Fe def, then they should have iron
supplementation both to correct anaemia and replenish body stores. This is
achieved most simply and cheaply with ferrous sulphate 200 mg three times daily
although ferrous gluconate and ferrous fumarate are as effective.


Elemental iron is the iron available in the supplement for absorption

Ascorbic acid enhances iron absorption and can be given together.

Patients often ask for injections instead! But Parenteral iron should only be used
when there is true intolerance to oral preparations. It is painful (when given
intramuscularly), expensive, cause the bum to have a BLACK spot and may cause
anaphylactic reactions. AND The rise in haemoglobin is no quicker than with oral
preparations!!! So why take the risk! The haemoglobin concentration should rise by
2 g/dl after 34 weeks.
Iron from meat, poultry, and fish (i.e., heme iron) is absorbed two to three times
more efficiently than iron from plants (i.e., non-heme iron).
The amount of iron absorbed from plant foods (non-heme iron) depends on the
other types of foods eaten at the same meal. Thats why we eat a mixture of
dishes.Foods containing heme iron (meat, poultry, and fish) enhance iron absorption
from foods that contain non-heme iron (e.g., fortified bread, spinach).

Foods containing vitamin C also enhance non-heme iron absorption when eaten at
the same meal. Fruits at end of meal is important.
Substances (such as polyphenols, phytates, or calcium) that are part of some foods
or drinks such as tea, coffee, whole grains, legumes and milk or dairy products can
decrease the amount of non-heme iron absorbed at a meal. Drink Chinese tea in
small amounts during your meal, good tea is sipped right?! Not gulped.
Calcium decrease the amount heme-iron absorbed at a meal. However, for healthy
individuals who consume a variety of food, the amount of iron inhibition from these
substances is usually not of concern.

Vegetarian diets are low in heme iron.
Medicines for peptic ulcer disease and acid reflux taken long term like some poor
teachers here for chronic stress induced gastritis reduce the amount of acid in the
stomach and the iron absorbed and cause iron deficiency. Thats the price we pay for
teaching!
This case scenario is complicated by the facts that

1. She is pregnant
2. She is not menstruating

Because of rapid growth, infants and toddlers need more iron than older children. A
student above is worried about the parasite called Ancylostoma duodenale. I am
more concerned about a MUCH2 bigger parasite called FOETUS! Women who are
pregnant have higher iron needs because of this. To get enough, most women must
take an iron supplement in pregnancy.

Serum iron. This test measures the amount of iron in the blood. BUT The level of
iron in the blood may be normal even if the total amount of iron in the body is low.
For this reason, a serum iron test is not adequate.

Serum ferritin. Ferritin is a protein that helps store iron in the body. Its a BIG magnet
that sticks many Fe molecules but its also an inflammation associated molecule.
Remember Cytokine storm in Dengue and what we must measure? Se FERRITIN!!!!
A measure of this protein in the serum helps find out how much of the body's stored
iron has been used or left. Se ferritin reflects the total body ferritin values.

Transferrin level, and total iron-binding capacity. Transferrin is a protein that carries
iron in the blood. Its like the lorries that carry this heavy metal round and round
without it falling off and damaging all the roads. Total iron-binding capacity
measures how much is the total capacity of the transferrin in the blood is there to
carry iron. If the patient has iron-deficiency anemia, he/she will have a high level of
transferrin, a high Total Fe carrying capacity aka TIBC that has ironically no iron.
(hehe IRONically)

So what should we use for screening, and what do we use when trying to diagnose a
cause of anaemia?
For GP screening, se ferritin is easily available and affordable. For diagnostic work
up, the iron studies PLUS Folic acid levels PLUS Vit B12 and TSH is needed. The
most useful Ix at workup is actually a PBF read by a competent haematologist. The
answer is almost always there.
Hemosiderin is an abnormal microscopic pigment composed of iron oxide and can
accumulate in different organs in various diseases. Iron is toxic when not properly
stored. Humans store iron within ferritin. The form of iron in ferritin is Iron(III) oxide-
hydroxide. By complexing with ferritin, the iron is made water soluble!

Several diseases result in deposition of Iron(III) oxide-hydroxide in tissues in an
insoluble form. These deposits of iron is hemosiderin! These deposits often cause no
symptoms, but they lead to organ damage.

Hemosiderin often forms after bleeding into an organ. When blood leaves a
ruptured blood vessel, the hemoglobin of the red blood cells is released into the
extracellular space. The macrophages phagocytose the hemoglobin to degrade it,
producing hemosiderin and porphyrin. The iron in haemosiderin cannot be released
for use. Its stuck!

YL: after a month's treatment with iron supplement and assuming she is taking her
iron tablets, she is still having microcytic hypochromic anemia with an MCV of 65,
MCH of 26, low normal MCHC, normal RBC and normal RDW! We don't know the Se
Ferritin and TIBC nor serum iron so far.

RDW is a quantitative measurement of anisocytosis on PBF. it tells us that the RBC
production is haywired and a high RDW reflects a nutritional anemia. RDW helps us
differentiate thalassemia and IDA for microcytic anemia; and vitamin B12/folate
deficiency (megaloblastic anemia) from other macrocytic but NON megaloblastic
anemia, hence its importance. in thalassemia the production of RBC is generally still
OKAY, although they are abit small, hence the normal RDW.

In this patient, a normal RBC and low normal MCHC, coupled on with a normal RDW,
we really couldn't strike thalasemia off, moreover she's pregnant!

So in view of a microcytic hypochromic anemia that is not correctable and also a FBC
components which suggest something other than IDA, i would run iron panel tests
to correct whatever iron deficiency anemia there is, and proceed to work her up for
thalasemia. Beta Thalassemia trait can be easily pick up by Hb electrophoresis, while
alpha thal needs a molecular diagnosis (as all Hb has an alpha component, on
electrophoresis one low all low!)
it is important to pick up alpha thal, a disease that is caused by gene deletion. if this
patient happened to be a carrier/ minor, it is even more important to screen her
husband too! The scariest thing about alpha thal is for the mother to have an aa/- -
gene makeup which have the likelihood to couple up with a similar aa/- - gene
makeup carried by her husband.............and produce a hydrops in her pregnancy.


Prof : The normal physiological increase in plasma volume in pregnancy causes
haemodilution and can give an artificially low haemoglobin level. However,
haemoglobin levels should not fall below 11.0g/dL and less than 10.5g/dL is
abnormal.

We often ASSUME that a microcytic, hypochromic anaemia esp in pregnancy is
caused by a lack of iron. However, this patient is not iron deficient. Her se ferritin
when tested is normal. Thalassaemia carriers are well, normal looking people but in
females with heavy menses or during pregnancy are often anaemic, sharing the
features of iron deficiency of microcytosis and hypochromia. A careful look beyond
the FBC at the PBF will tell us the answer because the PBF is very characteristic.
Another distinguishing element is the erythrocyte count which is reduced with
iron deficit, but often increased in thalassaemia carriers.

Her partner/husband/boyfriend needs urgent testing for haemoglobinopathies in
order to estimate the risk to the fetus. If the partner tests positive the couple can be
offered fetal testing to determine how the child might be affected, as for the fetus
there is a one in two chance of being a carrier of haemoglobinopathy and a one in
four chance of being either affected by the disease or free of the genetic disposition.
Excluding the possibility that the babys father is a carrier of a haemoglobinopathy

allows everyone to relax.



A very low MCV/MCH must trigger testing for haemoglobinopathies.


KEY POINTS
Watch out: not all microcytic, hypochromic anaemias as seen in the FBC is caused
by iron deficiency.

Test results in pregnancy have a different significance compared with tests for
those not pregnant.
For one you the doctor is dealing with a few lives!

Prof : The pathogenesis of anemia of chronic disease is multifactorial and is related
to hypo-activity of the bone marrow, with relatively inadequate production of
erythropoietin or a poor response to erythropoietin, as well as slightly shortened
red blood cell survival.

The hallmark ferrokinetic profile of anemia of chronic disease is decreased serum
iron level, decreased transferrin level, or normal or elevated ferritin levels, all of
which result in iron being present but inaccessible for use.

Do not forget Endocrine deficiency states, including hypothyroidism, adrenal or
pituitary insufficiency, and hypogonadism, which may cause secondary bone

marrow failure because of reduced stimulation of erythropoietin secretion.


Hyperthyroidism may also cause normocytic anemia.

Anemia occurs in acute and chronic renal failure. The anemia is usually normocytic
but may be microcytic. In renal failure, anemia occurs in part because uremic
metabolites decrease the lifespan of circulating red blood cells and reduce
erythropoiesis.

Anemia secondary to uremia is characterized by inappropriately low erythropoietin
levels, in contrast to the normal or high levels that occur with most other causes of
anemia. To further confuse the presentation, serum iron levels and the percentage
of iron saturation are often low. Furthermore, the serum creatinine level and the
degree of anemia may not correlate well.

And never forget abt RETICULOCYTES! ! They falsely increase mcv.

Remember that the role of the consultant is TO BE CONSULTED WHEN OTHERS
HAVE DIFFICULTY SOLVING A PUZZLE. So one must think laterally. When the HO or
MO is stuck it is usually because they are thinking along one line and NOT SEEING
THE FOREST. JUST THE TREE
Prof: Yin Ling,

You know that in thalassemia minor/trait, patients tend to have elevated RBC
count (>5.5) with low mcv and mch.

What is the pathophysiology behind this? What does this lead to clinically?
When a patient has a type of thalassemia, there is an excess production and
accumulation of globin chains produced by genes that are not effected by the
thalassemia deletion. This is a compensation mechanism that the body utilizes to
maintain hemoglobin production.

For eg In alpha thalassemia, the body can produce excess gamma chains as a
compensatory mechanism. This can lead to the production of gamma chain
tetramers (hemoglobin Bart's) in the unborn child and as beta chain tetramers
(hemoglobin H) in adults. This subsequent tetramer accumulation in response to
thalassemia often leads to red blood cell damage and hemolytic anemia.

The normal or high RBC count results from MASSIVE FACTORIES in bone marrow
producing RBCs desperately in response to chronic anaemia.
Together with the decreased MCV this are indicators of ineffective erythropoeisis.

The RBC morphology shows hypochromic microcytosis with schistocytes, target
cells, anisopoikilocytosis and basophilic stippling. Schistocytes form by several
mechanisms, one being the removal of RBC inclusions.

The splenomegaly is consistent with increased RBC destruction.
Yin ling,
Pls remember the following guides:
(1) Thal trait rarely causes anemia of less than 10 g/dL.

(2) The RBC count in TT is more than 5.0 x 106/L (5.0 x 1012/L) and in IDA is less
than 5.0 x 106/L (5.0 x 1012/L).

(3) The RDW in IDA is more than 17% and in Thalassaemia Trait is less than 17%.

BUT THE MOST IMPT AND SIMPLE TEST IS A PBF!

In Thalassaemia Trait it looks like a ZOO with all the various cells noted above. In
IDA the RBCs are like Minions.... ALL SMALL PALE AND CUTE. As iron-deficiency
anemia progresses, and the patients serum iron drops lower and lower, each
successive wave of new red cells gets smaller and smaller.

So there are some kind of small cells, and some really small cells! Smaller and
smaller minions as each batch is released. The red cell distribution width (RDW) is
high in iron deficiency anemia because there is now a wide variation in red cell
size.

In mild thalassemia (alpha or beta), the red cells are all about the same size; while
they are weird looking they are in every batch similarly weird! ! Hence there is
virtually minimal variation. So the RDW is normal.

This difference in RDW is helpful when youre trying to differentiate IDA and
thalassemia; if you have a microcytic, hypochromic anemia, the next thing youd
do is look at the RDW (and pls3 look at the blood smear).

If the RDW is normal (the cells are mostly the same size), then its probably
thalassemia. If the RDW is high (the cells vary a lot in size), then its probably iron
deficiency anemia.

6) on GROSSLY DAMAGED SMALL JOINTS OF THE HANDS
Dear yin ling,

There is an elderly lady who came to me with grossly damaged small joints of both
hands. Symmetrical. Mark ventral subluxation of both hands at the wrist joints is
seen. There is also prominent ulnar deviation in both hands and her fingers look
small and flail.

There is no warmth or swelling by the time I saw her.
Her fingernails have onycholysis. The rest of her skin is normal.
Her knees, feet and hips are normal. Her elbows cannot be fully extended.
Her blood tests do not show any evidence of inflammation. RA factor negative.
What are your thoughts?
Does ROADS mean anything?
Prof : Psoriatic arthritis usually occurs with skin psoriasis. People with psoriasis may
also have changes in their fingernails and toenails, such as nails that become pitted
or ridged, crumble, or separate from the nail beds. Signs and symptoms of psoriatic
arthritis include stiff, painful joints with redness, heat, and swelling in the
surrounding tissues. When the hands and feet are affected, swelling and redness
may result in a "sausage-like" appearance of the fingers or toes (dactylitis). In most
people with psoriatic arthritis, psoriasis appears before joint problems develop.
Psoriasis typically begins during adolescence or young adulthood, and psoriatic
arthritis usually occurs between the ages of 30 and 50. However, both conditions
may occur at any age.
In a small number of cases, psoriatic arthritis develops in the absence of noticeable
skin changes.

Psoriatic arthritis may be difficult to distinguish from other forms of arthritis,
particularly when skin changes are minimal or absent. Nail changes and dactylitis
are two features that are characteristic of psoriatic arthritis, although they do not
occur in all cases.

Psoriatic arthritis is categorized into five types:


(D)distal interphalangeal predominant,
(O)asymmetric oligoarticular,
(R)symmetric polyarthritis,
(S)spondylitis, and
(A)arthritis mutilans.
Now yin ling pls makes a word with these capital letters like how PNEIS is made into
SPINE. ROADS!

The distal interphalangeal predominant type affects mainly the ends of the fingers
and toes like SLE. Nail changes are especially frequent with this form of psoriatic
arthritis.

The asymmetric oligoarticular and symmetric polyarthritis types are the most
common forms of psoriatic arthritis. The asymmetric oligoarticular type of psoriatic
arthritis involves different joints on each side of the body, while the symmetric
polyarthritis form affects the same joints on each side like RA. Any joint in the body
may be affected in these forms of the disorder, and symptoms range from mild to
severe.

Some individuals with psoriatic arthritis have joint involvement that primarily
involves spondylitis. Symptoms of this form of the disorder involve pain and stiffness
in the back or neck, and movement is often impaired. Joints in the arms, legs, hands,
and feet may also be involved.

The most severe and least common type of psoriatic arthritis is called arthritis
mutilans. Fewer than 5 percent of individuals with psoriatic arthritis have this
form of the disorder, UNFORTUNATELY this patient has this. Her clue was in her
extensive nail changes not seen in RA.
Arthritis mutilans involves severe inflammation that damages the joints in the hands
and feet, resulting in deformation and movement problems. Bone loss (osteolysis) at
the joints may lead to shortening (telescoping) of the fingers and toes. Neck and
back pain may also occur but thankfully she was spared.



How common is psoriatic arthritis?

Between 5 and 20 percent of people with psoriasis develop psoriatic arthritis.
Some suggest a figure as high as 30 percent. Psoriasis itself is a common disorder,
affecting approximately 2 to 3 percent of the population worldwide.

Exam Q.... What genes are related to psoriatic arthritis?

The specific cause of psoriatic arthritis is unknown. Inflammation occurs when the
immune system sends signaling molecules and white blood cells to a site of injury or
disease to fight microbial invaders and facilitate tissue repair. When this has been
accomplished, the body ordinarily stops the inflammatory response to prevent
damage to its own cells and tissues. Mechanical stress on the joints, such as occurs
in movement, may result in an excessive inflammatory response in people with
psoriatic arthritis. The reasons for this excessive inflammatory response are unclear.
Changes in several genes that may influence the risk of developing psoriatic arthritis.
The most well-studied of these genes belong to a family of genes called the human
leukocyte antigen (HLA) complex. The HLA complex helps the immune system
distinguish the body's own proteins from proteins made by foreign invaders. Each
HLA gene has many different normal variations, allowing each person's immune
system to react to a wide range of foreign proteins. Variations of several HLA genes

seem to affect the risk of developing psoriatic arthritis, as well as the type, severity,
and progression of the condition.

Now in the OSCE situation, after you had diagnosed what assessment must follow?
This is the critical part. Any doctor worth his her salt can diagnose but the MRCP
winner must have this crucial next step. I will fail the candidate if this is not done
properly.
When the MCP PIP and DIP are all involved think of psoriatic arthropathy
When there is arthritis and funny nails think of it too.
When the hands are literally destroyed think of it too. RA destruction has the
classic features. This looks like its nuked.
Dear yin ling
Your answer is good for membership exam but still incomplete.
Remember I tried so very hard to teach the 4 aspects of diagnosis. What are they?
Uncle kian seng added the 5th aspect which is CRUCIAL to flying at MRCP exams!

This is the emotional toll of psoriatic arthritis! On top of evaluation of function.
While most chronic illnesses increase stress, the emotional toll of psoriatic arthritis
can very high. Not only do patients with psoriatic arthritis feel embarrassed because
of the skin psoriasis, but the joint pain, stiffness, and fatigue make it near impossible
to feel positive and be active.
NHS guidance on psoriatic arthritis tells doctors to screen patients for emotional
problems! !! Never forget these 2 things; FUNCTION AND EMOTION. In the ideal
world this of course would apply for all patients of all illnesses.
YL : 4 aspects of diagnosis is Anatomical, pathological, aetiological and functional.
5th one would be emotional.
Prof: Dr David KL Quek reports a patient worth a lesson to all of us
A few years ago a 74 yr old man with extensive psoriasis but moderate polyarthritis
presented with STEMI and acute pulmonary edema. We treated his acute heart
failure then stented the culprit artery of his 3 vessel CAD. Then we got on to talking
about his skin and joint disease and was shocked to hear that he'd not ventured out
of his home for 5 years because of shame over his previously untreatable and 'ugly'
skin condition! We got a dermatologist in who finally managed to get his psoriasis
under control!
Here for MRCP candidates to remember is that someone with a chronic
inflammatory disorder can be predisposed to a higher risk of CAD. Methotrexate

when used in treating psoriasis also predisposes the patient to higher thrombotic
events. So you absolutely right that we need to more routinely gauge assess the
emotional and psychological components and needs if our patients!


Dr Ng Kian Seng: Yin Ling ...look at the nails, onycholysis and pitting are the lesions
we usually talk about but Patients with Psoriasis have many other nail clues...Beau's
line, Salmon patches, leukonychia, spotted lanula....


Kianseng Ng One diagnostic aid, to help differentiate RA and PsoA, RA swelling is
fusiform, like in this image. The swelling in PsoA may be like this...sausage shaped,

Dactylitis

Kianseng Ng ...and of course in PsoA, you have this...enthesitis, so always examine


the Achilles tendon, if u fail to, the enthsitis becomes your Achilles tendon, now
have I confuse you with words

ROADS is useful but somtimes the patterns may change over the course of time
(where PsoA is concerned)


and always remember inverse Psoriasis where the skin lesions may be hidden...hair
line, cleft between gluteals, umbilicus, genitals, inguinal regions and when you see
the lesions this may help you diagnose Psoriasis


Yin Ling: Dont miss this, fourth toe Dactykitis and hair line Psoriasis



Dear Yin Ling: Can you comment on the risk of GI bleed with anti inflammatory
painkillers?
A combination of low dose aspirin with any NSAID increases the risk of GI bleed 15
times. Celecoxib does not affect platelet function in contrast to NSAIDs.

Breaking a Myth! Osteoarthritis has Low Grade inflammation besides degeneration. I


think every doctor who examines an OA joint will have this impression. Is it better to
leave patients on low dose cox2 inhibitors or using prn in OA? FOR Frequent FLARES,
low dose continuous Rx is better.
With ref to Pain relief both cox2 inhibitors and NSAIDs are equally effective. The
difference is in SAFETY. There is no medication that is risk free.
The Risk of GI bleed is present with SSRI! ! PLS NOTE THIS. A combination of SSRIs
with NSAIDs is common and dangerous.
The risk of GI bleed exponentially increases above the age of 65 with any drug that is
associated with GI bleed.
Highest risk of GI bleed is with naproxen, Followed by diclofenac.
Meloxicam is not associated with a lower risk.
Celecoxib combined with PPI is not associated with increased risk of GI bleed.
The use of PPI for more than 2 years is associated with fractures and sudden death
from hypomagnesaemia. COMBINING AN NSAID WITH A PPI DOES NOT ELIMINATE
THE RISK OF GI BLEED.
Even cox2 inhibitors have negative effects on renal function but less in degree THAN
NSAID
Hpt patients taking NSAIDs and cox2 inhibitors have increase in BP. LESSER WITH
COX2 INHIBITORS
Be aware that PCM in high doses inhibits COX1 as well. The highest daily dose of
PCM is now 2.4gms! At doses more than 3gms cox1 is inhibited and the risk of GI
bleed is present
With regards to AMI ROFECOXIB HAS THE HIGHEST RISK Followed BY DICLOFENAC
Ibuprofen IS associated with the highest risk of stroke! !
Patients taking long term warfarin poses a big problem when in need of painkillers....
the safest approach is likely SHORT TERM celecoxib WITH a PPI



7) on BAD TONSILS!
Dear Yin Ling,

A 17-year-old school girl came with her mother. One week previously she had
been seen by a GP and was diagnosed with acute tonsillitis. The doctor noted a
history of sore throat, malaise, muscle aches all over the body, shivering, swelling
around the eyes and headaches. Her mother was very anxious as she wants her
well asap to study!

The examination then revealed a temperature of 39.6C, pharyngitis, red, purulent
and enlarged tonsils, and cervical lymphadenopathy. The doctor prescribed a 5-
day course of ampicillin and advised the patient to increase her fluid intake, rest
and take regular panadol.
The mother is concerned because her daughter has not recovered. She angrily
related that at first the antibiotics seemed to help and the temperature, sore
throat and headaches improved. But after 4 days a faint non-itchy red rash
developed.
While the pain in the throat is better, the patient complains of more pains in her
neck and the recurrence of headaches. She feels total lack of energy and mother is
concerned that her daughter sleeps 18 hours a day. Mother is concerned about not
having the energy to study for her upcoming exams.

On examination she looks pale, unwell with a slightly yellow sclera. Her
temperature is 37.6C. There are multiple swollen and tender lymph nodes in the
anterior and posterior cervical regions, bilaterally. Her throat is still inflamed, and
the tonsils enlarged, but without white patches. Some petechiae is seen on the
soft palate. There is a faint red macular rash all over her trunk spreading to the
limbs. The tip of the spleen is palpable on deep inspiration, the liver 2 FB palpable.

What is the likely diagnosis and differential diagnosis?


What tests would you order?
What will you do while awaiting the blood tests?
Prof: Often the cervical LNs is BIG and parents worry about a malignancy. If the
tonsils look as though a madman had painted whitewash on it, I am reasonably
confident but if tonsils are mildly inflamed and with LNs and hepatosplenomegaly,
I too worry about an Acute Haematological pathology. A PBF is very useful, often
an experienced Pathologist will pick up Infectious Mononucleosis.
IM aka Glandular fever is the most likely clinical diagnosis as many of you all
realised. Acute tonsillitis is seen far more commonly in primary care and would have
been a disease susceptible to antibiotic treatment although the rash may have been
triggered by the antibiotic or may be from the illness itself. The further list of
potential diagnoses includes acute Human immunodeficiency virus (HIV) infection,
old friend diphtheria, cytomegalovirus and leukaemia.



The PaulBunnell reaction (Monospot) heterophile immunoglobulin M (IgM)
antibodies agglutinating sheep erythrocytes is the most commonly used screening
test for glandular fever. It can be falsely negative, especially in young patients, or
falsely positive, for example owing to cytomegalovirus; so its not very good.

A full blood count should show a leucocytosis between 10000 and 20000
cells/mm3, thrombocytopenia (often), and on the blood film many atypical
activated T-lymphocytes (mononucleosis cells). More than 20 per cent atypical
lymphocytes or more than 50 per cent lymphocytes with at least 10 per cent
atypical lymphocytes on blood film make the diagnosis very likely.



There are more specific immunological tests for EpsteinBarr virus available which
can be useful if the PaulBunnell test is negative and we still suspect the disease but
its academic because by the time the results are back its already recovery time.

Glandular fever (infectious mononucleosis, or kissing disease) is an infection of the
B-lymphocytes by the EpsteinBarr virus. It is a self-limiting disease. The virus is
secreted in the saliva and can be transmitted through kissing or sharing utensils
(cups, cutlery, towels).

The organism may also be shed from the uterine cervix, implicating the role of
genital transmission in some cases and possibly Oral Sex! On rare occasion, EBV is
spread via blood transfusion.
The incubation period is 48 weeks. Most patient recover within 2 weeks with
some residual tiredness for another week. However, a significant minority go on to
suffer with tiredness for much longer.

It is reassuring to the doctor, patient and relatives to have the diagnosis confirmed.
She needs to avoid contact sport because of the potential for damage to her
swollen spleen, for 12months. She should see herself as infectious while she is
feeling unwell, avoiding sharing utensils and close bodily contact.
KEY POINTS
Early presentations of disease can be deceptive, with the correct diagnosis
emerging later in its progression.
Proper diagnosis and sensible advice are reassuring for the patient and their
family, whether or not there is effective medical treatment as the differentials are
not too nice folks.

Dear Yin Ling,



Circulating B cells spread the infection throughout the entire reticular endothelial
system (RES), ie, liver, spleen, and peripheral lymph nodes. Thats why they are
Enlarged!! Easy to remember.

EBV infection of B lymphocytes results in a humoral and cellular response to the
virus. The humoral immune response directed against EBV structural proteins is the
basis for the test used to diagnose EBV infectious mononucleosis. However, the T-
lymphocyte response is essential in the control of EBV infection; natural killer (NK)
cells and predominantly CD8+ cytotoxic T cells control proliferating B lymphocytes
infected with EBV.

In HIV patients we can well understand why the EBV goes berserk!

The T-lymphocyte cellular response is critical in determining the clinical expression
of EBV viral infection. A rapid and efficient T-cell response results in control of the
primary EBV infection and lifelong suppression of EBV.

Ineffective T-cell response may result in excessive and uncontrolled B-cell
proliferation, resulting in B-lymphocyte malignancies (eg, B-cell lymphomas).

The immune response to EBV infection is fever, which occurs because of cytokine
release consequent to B-lymphocyte invasion by EBV. Lymphocytosis observed in
the RES is caused by a proliferation of EBV-infected B lymphocytes. Thats why the
lymphocyte count goes up and you have funny looking cells. At least to me la)

Pharyngitis observed in EBV infectious mononucleosis is caused by the proliferation
of EBV-infected B lymphocytes in the lymphatic tissue of the oropharynx. Now you
know why the tonsils which is effectively 2 Big LNs look like a warzone.

Pearls:
fatigue is almost invariable! Pls note this symptom.

Pulmonary involvement is not a prominent feature of EBV infectious
mononucleosis. If the patient is badly hacking his/ her lungs away, ITS NOT IM!

Jaundice occurs because of the hepatitis.

Pls note yin ling that a POSITIVE THROAT C&S for Strep may mean NOTHING! as
approximately 30% of patients with EBV infectious mononucleosis have group A
streptococcal carriage of the oropharynx. The unwary physician may incorrectly
conclude that a throat culture for group A streptococci in a patient with infectious
mononucleosis represents streptococcal pharyngitis.

EBV infectious mononucleosis is characterized by early and transient bilateral
upper-lid edema but this is not a common sign. The external eye involvement of
EBV infectious mononucleosis is characterized by bilateral upper-lid edema. This is
referred to as Hoagland sign.
Hoagland sign may be detected when patients look in the mirror early in the course
of their illness or when the astute physician notices this early in the clinical
presentation. Hoagland sign is present for only the first few days of illness and
should not be sought later in the course of the infectious process.

EBV is the main cause of malignant B-cell lymphomas in patients receiving organ
transplants. The Immunosuppression is Heaven for the virus!
Depending on the intensity, rapidity, and completeness of the T-lymphocyte
response, malignancy may result if EBV-induced B-lymphocyte proliferation is
uncontrolled. Hodgkin disease and non-Hodgkin lymphoma (NHL) may result. Other
EBV-related malignancies include oral hairy leukoplakia in patients with HIV
infection.
Leiomyomas and leiomyosarcomas in immunocompromised children,
nasopharyngeal carcinoma, and Burkitt lymphoma are among other neoplasms
caused by EBV.



8) on METFORMIN
Dear yin ling,

you asked me yesterday to viva you on Metformin and its usage in various clinical
scenarios. I post here a few common Qs on Metformin.



This chart is impt as it shows the relationship between stress causing HIGH Glucose
which cannot be cleared by low Insulin!! and with increase of shunting to lactate
now made worse by poor clearance.


Dear YL,

We no longer use Phenformin. But it has unjustifiably given Metformin a bad name.
Metformin is actually a very safe drug when used mindfully.

Under what conditions does Lactic acidosis RARELY occur? You run marathons, it is
impt that you understand this. Can you describe how you felt like when lactic
acidosis occured?
Do you recall not having much appetite despite running such a long distance,
anorexia is common. Were you nauseated, perhaps even vomiting? You felt
deliriously happy when you finished but this altered level of consciousness could be
due to metabolic cause!!! Yes you had hyperpnoea, abdominal pain and thirst. And I
recall you telling me that you were anuric and had DARK urine! Please dun do it
again!!!

Red blood cells produce lactic acid as a byproduct of the regeneration of ATP during
anaerobic glycolysis but cannot use lactic acid. Take home that when you have a
patient with tissue underperfusion and hypoxia coinciding because of illness or
surgery, avoid Metformin!

Lactic acidosis is a broad-anion gap metabolic acidosis caused by lactic acid
overproduction or underutilization. The ABG and simple maths will tell you this.
Overproduction of lactic acid occurs when the body must regenerate ATP without
oxygen (tissue hypoxia). Circulatory, and pulmonary disorders are commonly
responsible.

Underutilization involves removal of lactic acid by oxidation or conversion to


glucose. Liver disease, inhibition of gluconeogenesis by metformin!! are common
causes. Poor renal function also makes excretion poor.

Approximately 1400 mmol of lactic acid are produced daily, which are buffered by
1400 mmol of HCO3 to form sodium lactate. The liver is responsible for oxidizing
lactate to restore this amount of HCO3. The role of the liver in lactate homeostasis
is considerable.

The kidneys contribute to lactate removal in three ways: excretion,
gluconeogenesis, and oxidation.
Malignant cells produce more lactate than normal cells, even under aerobic
conditions. This phenomenon is enhanced if the tumor outstrips the blood supply!!


Yin ling,

Pls note the word STABLE! Problem starts when Renal function is deteriorating or is
expected to deteriorate but NOT in the awareness of the HO.

No one will use it when eGFR is less than 30 or you are walking a thin line.

Tell me what common conditions can lead to a clinical deterioration of renal
function?
Ninety percent of metformin is excreted unchanged by the kidneys and lactic
acidosis typically occurs in patients with renal insufficiency. Even mild renal disease
increases the risk of lactic acidosis.

Prof: A metformin dosage of 850mg twice a day, or 500mg three times a day,
usually gives good diabetic control. There is not much point giving beyond 2000mg
a day.

Caution is needed when increasing the daily dosage beyond this, especially in the
elderly and those with mild renal disease.
Significant mortality ( as high as 50%) is associated with biguanide-induced lactic
acidosis and attention should be focused on prevention through awareness of the
risk factors.


Heart failure and metformin. It is not anymore dangerous than usual. Again the
word is STABLE vs UNSTABLE.

A patient who comes in with fluid overload and low O2 sat is different from one who
is comfortable at home well maintained on medicine and stable.

Someone who comes in with mild illnesses like a URTI is obviously different from
another with severe pneumonia.

Someone coming for excision of an in grown toenail is different from an elderly
woman going for a total hip replacement.

What will you do if you are refered such diabetic patients as above?

YL: For the short term stay in hospital for unstable patients, sliding scale will be a
good choice. Until we know patient is more stable and taking orally well, we will
keep them on either sliding scale or regular insulin first

PROF: Can you pls elaborate on the Sliding Scale used here?

YL: We give pt subcut short acting insulin injection based on their blood glucose
level every 4 hourly. The ranges can be 2 units of insulin if glucose is 5 to 10, 4 units
for 10 to 15 and so on. Sliding scale can be augmented if glucose is hard to control.

Lactic acidosis is an uncommon but potentially fatal adverse effect. The reported
frequency of lactic acidosis is 0.06 per 1000 patient-years, mostly in patients with
predisposing factors.

Examples of metformin-induced lactic acidosis scenarios include:

A 69-year-old man, with renal impairment and cardiac failure, was prescribed
metformin due to failing glycaemic control on glibenclamide monotherapy. He was
well for six weeks, then developed lactic acidosis and died within 3 days.
Tell me what should have been done if we can turn back the clock?

An elderly man had a total hip replacement. Post-surgical lactic acidosis caused the
death of this 70-year-old man whose metformin was not withdrawn at the time of
surgery.
If you are the resident, what would you have done instead?

A 56-year-old woman, with no predisposing disease, died from lactic acidosis
following major abdominal surgery for Ca Colon. Metformin was withdrawn only for
the day of surgery.
What should you have done instead if asked to see this patient?
The risk factors for metformin-associated lactic acidosis include sepsis, high
dosage, increasing age, and DEHYDRATION. The last is often forgotten! In
situations predisposing to dehydration such as FASTING for surgery, or contrast
radiography, metformin should be ceased at least 48 hours prior to the procedure
(or on admission for an emergency procedure). It is not restarted until the patient
has fully recovered and is eating and drinking normally. The glucose levels of
patients in CATABOLIC states, e.g. sepsis or in the post-operative period, should be
monitored and Short-term insulin therapy is strongly advised for Mx.

Try not to use above 70 years old, and if using be very careful with renal function.
remember that by the time biochemistry is abnormal, the renal function is already
significantly affected bec of renal reserve
YL: For the first scenario, his renal impairment already stop us fr using metformin.
Pairing up with a heart failure causing increase tissue hypoxia, lactic acidosis is likely.
Glibenclamide is such a long acting SU, hypo episodes is risky. And glibenclamide
which shouldnt be use in renal impairment. We can use gliclazide.. a newer gen SU,
lesser hypo episodes, lssser weight gain and can be used in mild renal impairment
with caution. He will also be a good candidate to start insulin. When he is being
admitted for heart failure, temporary use of sliding scale is warranted if hes
unstable.
PROF: AS a rule, pls do NOT use glibenclamide in the elderly. Its long acting and
even its metabolites are ACTIVE. Glicazide is excreted by the liver so its relatively
safer even in renal impairment.
Despite the presence of many unique classes of drugs to treat hyperglycemia in
patients with type 2 diabetes, metformin remains the Drug of Choice.
Metformin caused less weight gain compared with either the thiazolidinediones or
sulfonylureas. Metformin decreased low-density lipoprotein levels compared with
pioglitazone, sulfonylureas, and DPP-4 inhibitors.

Patients taking sulfonylureas had a fourfold higher risk of mild or moderate
hypoglycemia compared with metformin alone. This is a tremendous advantage of
the drug.

Most importantly as far as evidenced based medicine is concerned, Metformin is
unique in being not only as effective as any other oral antidiabetic therapy in
controlling blood glucose, but also having an unparalleled clinical database relating
to improved clinical outcomes in pre-diabetic subjects, and patients with
established type 2 diabetes.

9) on DIABETES!
Prevalence is a shocking 20.8% in Msia!
Non Pharmacological Treatment
Exercise: how much?
Sedentary behaviour is less than 1.5met. Sex is 5 mets only

Sedentary behaviour increases all cause mortality.


The more educated the person, the less phy activity. The more we sit, the faster we
have cvd. Television time increases risk of dm, cvd and all cause mortality
So how much exercise do we need?
Regular. 3x a week
Resistance training. 2x a week
Intensity: irrespective of duration, the more intense the greater reduction in total
deaths.
High intensity exercise at regular intervals 3x a week may be better than moderate
intensity continuous training. But many people cannot do this because of phy
limitations.
What abt low volume interval training?
3x a week of 3 mins of hard work. But Can it work? We do not know yet.
Diet: less than 6g of salt
No evidence with supplements.
How much carbo? We do not know!! But A low GI diet is needed.
Basmati rice has low GI.
Jasmine rice is close to glucose!
Table sugar is Sucrose.
Fructose is actually better than glucose when both are in small amounts.
Carbo in combination with high fibre is prefered.
Protein: we do not know the amount to take. But a High protein diet is associated
with higher risk of albuminuria.
Low protein diet appears to slow progression to renal failure.
Fat: also no data. But Avoid trans fats. Plant stenols can reduce total and LDL chol
61% of dialysis patients in Msia are Diabetics
There is a J curve in DMs with renal impairment. Hba1c of 7 to 8% is at the bottom
of the curve. In DM with renal impairment, lowering the Hba1c below 7% is
associated with faster deterioration.

Obesity: contributes to renal impairment. Second only to proteinuria. Both direct


and indirect effects on the kidney. BMI the higher, the higher the risk.
Surgery: med Rx was cf to gastric bypass and Sleeve gastrectomy. In obese patients
gastric bypass and Sleeve gastrectomy is superior to medical Rx.
Vegetarian diet appears to help DM patients. STOP SMOKING. Higher risk of renal
dysfunction with smoking.
Poor oral hygiene: increase risk of chronic renal dysfunction. Chronic inflammation
of the oral cavity contributes.
Monitoring: Pls do not forget Post Prandial Glucose. Fasting glucose monitoring is
inadequate. Both needs control.
Glucagon secretion is not suppressed in DM as beta cell function declines. This
increases glucose esp in fasting states.
Intensive sugar control clearly reduces microvascular complications.
Modern insulins are ultra fast acting eg Lispro, Aspart and can be given at meals
without a need to wait.
Early AGGRESSIVE treatment in newly diagnosed young DM is needed.
This reduces long term complications. The data from UKPDS post trial follow up Is
clear on this. Macrovascular benefits are seen but only on long term follow up.
In DM incretins levels are relatively low and hence a method for Rx approach.
This help control PPG. Reducing alpha cell glucagon output is another. This help
reduce FBG
While it is very important that we keep the A1c level as close to normal as we
safely can, this MUST vary according to the patient's clinical circumstances, age,
risk for hypoglycemia, social background and many other factors.
Which patients will you be happy to have A1c levels at 10%, which, for
complicated reasons, you can't get them lower or do not want it lower?
And who will you want at target of < 7% and even in some < 6.5%?

For a patient who is symptomatic and has a blood sugar level of >11.1mmol/l he
clearly has diabetes. However can you diagnose with
Measurement of the hemoglobin A1c level;
Measurement of the fasting glucose level;
and what are its limitations?

Which is MORE Reliable; MGTT or the hemoglobin A1c level?



Please note that blood glucose levels like any laboratory assay can vary even in
perfectly normal people.
We do one test to see whether a person has diabetes, and then unless the values
are clearly abnormal, we need to repeat the same test to verify whether that value
is true. Because lab errors can occur, its impt to repeat the same test.

The treatment of DM is more than controlling blood sugars, in any real life and exam
situation, a global evaluation is essential. Blood pressure: what is the present
targets?

Every patient who has diabetes and who is older than age 40 years is somebody
who will need to be on statin therapy! The occasional man who swears that the
statin had caused ED is also a difficult situation as I am not sure its the DM or the
drug. What is the role of fenofibrate if any?

The role of ACEI and ARBs is another common exam Q. Will you use angiotensin-
converting enzyme inhibitors or angiotensin receptor blockers for the prevention of
nephropathy in patients who do not have elevated blood pressure levels?
How will you advise your patient regarding alcohol intake?
YL : we should individualised our HbA1c target according to a patient's age,
comorbiditis and general condition. For a young patient say 40-50 years old having
T2DM, we would like his HbA1c to be tightly control, targeting a 6.5-7% to delay
the onset of microvascular and macrovascular disease.
However if we are looking at an elderly, frail, 70 year old diabetic patient, we
would be happy if his HbA1c is around 8-10%, tightening the glucose control equals
expecting more hypoglycemic events which is detrimental to the old.
DM can be diagnosed when a patient has a HbA1c of >6.5%. HbA1c of 6-6.5% is
considered Pre Diabetes. we can also diagnose DM by fasting blood glucose of >7.0
if there is symptoms. 2 readings is needed if there is no symptoms. IFG is when FBG
ranging 5.6-7 and IGT when 2 hours post OGTT BG 7.8-11.0

Be careful when using HbA1c to diagnose in patients who have high RBC turnover
eg thalasemia, any hemiglobinopathies, renal disease and etc.
i would like to think that HbA1c is more reliable as it measures glucose control for
over a longer period of time.


BP target is at 130/80 for patients with DM, we no longer use the 125/75 target
anymore. there is no benefit in lowering BP to such level in this group of patients.

Anyone above 40 who has DM deserves a statin. after starting statins and
controlling the blood sugar, if TG is still high, fenofibrate is warranted. Fenofibrate
can also be used in patients who cannot tolerate statins eg from the myopathy.

ACE-i and ARB is beneficial for DM patient and we are taught to use them as first
line in all DM patient due to the renal protective effect. ACE can help prevent
nephropathy and reduce proteinuria. these patients commonly have deranged renal
function from diabetes, it is not entirely contraindicated to use ACE/ARB as long as
renal profile are monitored, stop if there is >30% raised of creatinine during a repeat
renal profile in more than 2 weeks. we would not use ACE - i once creatinine is >200.

We are commonly taught to use ACE-i as first line and if patient cannot tolerate ACEI
mostly due to an ACE induced cough we would use an ARB instead.
Yin Ling: Why ACE-I first? ACE OR ARB in HPT (Up to 80% of Diabetics are
hypertensive!)

Evidence for ACE I


-HOPE trial compare RAMIPRIL with placebo while EUROPA trial uses PERINDOPRIL
8mg to placebo.
Both shows Reduction in cv death and MI. These two trials show BP independent
effect of ACEi to reduce endpoints. The benefits ACE confers is more than its Bp
lowering effect
Meta analysis of ACE shows at NO BP reduction, it still reduce MI and death by
12%- A Bp independent effect
HOWEVER, ARB DOES NOT HAVE THE SAME BP INDEPENDENT EFFECT
-meta analysis shows at NO BP REDUCTION, there is a paradoxical increase in MI
incidence!
-No trial on ARB reduce mortality like in EUROPA AND HOPE
-Meta regression Analysis shows RRR of ACE vs ARB by 15%
SO WE ASK WHY?? WE NEED TO GO BACK TO BASIC SCIENCES
One concept to learn : Endothelial cells turnover is about 3 months. When mismatch
of apoptosis and regeneration occurs, atheroma formation happens, plaque
instability happens and ACS risk increases, MI risk increases.
ACE converts Angiotensin-I to Angiotensin-II, and angiotensin-II can act on either
AT1 receptor or AT 2 receptor
The fx of AT 1 receptor
Vasoconstriction
Water retention
Proliferation
Anti apoptotic
The Fx of AT 2 receptor
Vasodilatation
No water retention
Anti proliferation
Pro apoptotic
ARB IS AN AT1 receptor blocker, which allows angiotensin-II to act on AT2 receptors,
which is anti proliferation and pro apoptotic---> endothelial dysfunction and
increase atherosclerosis and plaque rupture may occur.
Pt with CAD has increased endothelial turnover

Why ACE fares better than ARB ?


1. Angiotensin II has toxic effect on organs, independent of BP.
ACE REDUCES angiotensin 2 and then Reduces TNF alpha which is pro apoptotic
2. maintain BRADYKININ level which increases NO levels, which maintain healthy
endothelial, potentiate ischemic preconditioning (withstand ischemia without
producing v fib)
ARB DO NOT maintain the amt of bradykinin and do not decrease Angiotensin-II
levels which allow toxic effect on Organs by causing endothelial dysfunction.
Even more so, When ARB inhibit AT1 receptors, negative feedback will stimulate the
increase of angiotensin-II which causes endothelial dysfunction and plaque rupture.
2. ASCOT TRIAL shows that when PERINDOPRIL and amlodipine was used as
combination, the mortality benefit goes beyond BP lowering in the former
combination of Beta blockers and diuretics.
Why??
Vessels endothelium are kept dilated by ACEi which increases bradykinin and
decreases AT2, and vessels smooth muscle was kept dilated by CCB - by decreasing
Ca acting on voltage operated and receptor operated calcium channels; both are
synergistic.
4. Canada still keep their Bp threshold at 130/80, 140 systolic is still too high for
diabetic
5. CONCLUSION: comparing ACEI AND ARB
BOTH EXACT SAME IN REDUCING BP AND STROKE
BOTH REDUCE NEW ONSET DM AND AFIB
BUT DIFFER AT RISK OF MI AND DEATH

Alcohol : cut off units for men is 21 and women 14 per week. but of course taking
many units in one setting is bad. encourage moderate alcohol intake.
Yin Ling:
1. Postprandrial blood glucose is more sensitive than fasting BG. Encourage to do
PPG in clinic for follow up monitoring rather than FBG
2. DM is of polygenic inheritance. Hence Patient's response to anti diabetic drugs is
variable; some patients may require only low doses to act while others respond
poorly even to multiple drugs.
3. All drugs besides metformin are team players; they work best in combination with
metformin.
4. There's a window period before the diagnosis of DM for intervention to reverse
onset of DM. This 'prediabetes' phase provides us a Window of opportunity to
prevent Diabetes from occuring. Weight loss, exercise, a low calorie diet, and even
Metformin or DPP4I can help. Remember that the diagnosis of DM is based on
arbitrary values set on figures above which complications are common.
Patient with end organ damage eg retinopathy has DM despite him having normal
values or Pre Diabetic values.
3. Insulin resistance is static throughout a diabetics lifetime once it has plateaued
while insulin secretion will continue to decrease with time.
4. HOON study
Impaired fasting glucose confers a 33% risk of DM in the next 5 years
Impaired glucose tolerance another 33%
With Both IFG and IGT there is a marked 66% DM risk in the next 5 yrs
5. Postprandrial TG if raised suggest insulin deficiency even before DM is diagnosed.
It's the earliest simple marker we have for Insulin deficiency. insulin is needed for
lipid metabolism. Hence raised postprandial TG let us know early on that there is a
risk of impending DM and we have a Window of opportunity to intervene before full
fledged DM sets in
6. UKPDS shows at the time of dx of DM, 50% of beta cells are already lost.
Progressive lost of 5-10% each yr
Most diabetics Require insulin after 10-20 yrs due to natural progression of beta
cells loss
7. Insulin is a direct antiinflammatory agent for the beta cells in DM pt

8. Change of nomenclature
Incretin LEVELS does not decrease with time because of diabetics. It's the incretin
EFFECT that is progressively diminishing with time BECAUSE Incretin acts on beta
and alpha cells - hence due to beta cells progressive dysfunction, incretins effect
also decrease with time. There is however a slow reduction of Incretin production
with aging.
9. MODY
Young type 2 DM but with No anti beta cells antibodies. Autosomal dominant with
strong family history and presenting young typically before 25 years old.
10. Metformin also increase GLP1 levels. It's our current first line agent for diabetes
11. Second line any antihyperglycemic drugs including insulin can be combined with
metformin.
12. Incretin works on alpha and beta cells, Causing insulin secretion and also
inhibiting glucagon driven gluconeogenesis.
Question: how significant is the role of incretin effects on ALPHA cells causing
inhibition of gluconeogenesis that lowers glucose level? It is quite significant and
patients with beta cell exhaustion on Insulin treatment have a significant reduction
of Hba1c when a DPP4I is added. Here it can only act via the suppression of glucagon
release.
11. GLP1 analogs act at a supraphysiological level. Acts at the neurological level
suppressing appetite, causing satiety, delaying gastric emptying and even nausea.
Used for wt loss in the west. While DPP4I results in GLP1 values that acts at a
physiological level
12. Only oral glucose can stimulate incretin effect, not IV glucose (stimulate beta
cells directly)
13. Glicazide : 99% excreted in liver. Can be used in renal impairment
14. Remember DPP4I is a team player, do not use as monotherapy as it is weak and
only offers a reduction in Hba1c of 0.5%. It may be used as monotherapy during the
window of opportunity before DM is dx ( PREDIABETES).
15. ACE-I is still superior to reduce cvs risk and is has better renal protection
compared to ARB. Low doses of ACEi only inhibit the circulating ACE in the blood and
may be adequate to lower BP in Hypertensives. But it is inadequate to block ACE in
tissue. We need to block tissues ACE to have the full benefit of ACEI hence high
doses of ACE-I is needed. Remember to uptitrate pt's ACEi

16. ARB Is superior to ace in pt with proteinuria


17. Metformin reduces the risk of cancer in diabetics. 56% risk reduction for breast
cancer, 60% RR for pancreatic cancer
Stop metformin when eGFR < 30
Dear yin ling
Hba1c is better than MGGT. A1C captures chronic hyperglycemia better than two
assessments of fasting or 2-h oral glucose tolerance test of plasma glucose

Diabetes has been diagnosed for decades with fasting plasma glucose or, with an
oral glucose tolerance test (OGTT). Hyperglycemia as the biochemical hallmark of
diabetes is unquestionable. However, fasting and 2-h MGTT gauge just a moment of
a single day. Many2 factors affect this sample which Hba1c overcomes.

BUT Hba1c testing lacks standardization and different labs provide different values
for even a same sample. This on top of haemolytic anaemia or lack of EPO or bone
marrow problems. So we need to interprete mindful of these.

When there is IHD, hypoglycaemia is DANGEROUS. it can ppt arrthymias and even
ACS. Hence in proven DM CHD, the target shd NOT be lower than 7%.

Serum fructosamine was something I used to test but its unreliable. In thalassaemia
trait patients bld sugar profiles remain our bedrock. Same for renal failure patients.

Top secrets:

Fbs, Hba1c and 2HPP values are all correlated to retinopathy, the most specific
complication of dm
Fbs is poorly correlated to CHD
BUT 2hpp and Hba1c is well associated.

Bld sugars start dropping in value by 5% every hour after venesection
So if the specimen sits on a bench while the technician rests, you are not going to
get anything close to reality. Worse in HOT WEATHER.

Hba1c is not affected in this way.

Dear Yin Ling,
Concerning DM Treatment and Hypoglycaemia
Insulin was discovered in 1921. That is the most potent hypoglycemic agent
available. Why do we need any other med for DM?
Because the risk of hypoglycaemia is common and real.

Chlorpropamide is a potent hypoglycaemic agent and if you had used it YOU are
OLD! It has been banned in most countries because of severe and prolonged
hypoglycaemia.
The incretins act on BOTH ALPHA and BETA CELLS; it Suppress Alpha cell activity and
stimulate Beta cells activity.
When Beta cell exhaustion is present we use Insulin perhaps with Metformin.
The Use of DPP4I or GLP1 RA will work on Alpha receptors inhibiting Glucagon
secretion.



GLP1 is rapidly degraded. Only 1 to 2 mins half life on humans.
GLP 1 Receptor Agonists is available but is in injection form. They are analogs and
NOT recognisable by DPP4I hence its long half life and duration of action.
DPP4I prevent natural GLP1 from breaking down BUT the limiting factor is twofold;
1. GLP1 secretion, and there is a natural decline with aging and
2. in DM, GLP1 EFFECT is reduced as beta cell production of Insulin is reduced
despite the GLP1 stimulation because of declining beta cell function.


Individual response to drugs is Real because of genetic differences. In Elderly
patients with normal renal function, start with metformin then plus DPP4I then add
low dose SU cautiously.
DPP4I : Caution when in patients with
MILD RENAL Impairment eGFR 60 TO 90
Moderate RENAL IMPAIRMENT 30 to 59
SEVERE 15 to 29
As Drug doses adjustment is needed.


Metformin is STILL the first line drug in Type 2 DM as it has many advantages which
includes a reduction in risk of malignancy in Diabetics.

HPT and DM
70 to 80% of patients with DM are also Hypertensives. In Msia 90% of DM are
hypertensives.
Major CV events DOUBLE when both present.
Tell all patients about NEWS: Nicotine Exercise Weight Salt must be addressed.
3 to 9% wt loss equals 3 to 6mmhg decrease
Walking Jogging as Exercise reduces by 13 systolic 18 diastolic
Msian guidelines to start Rx at >140 /80
Target <140 80 and in yg patients <130 systolic
Preference for RAS blockers esp if micro albuminuria or proteinuria is present.
There is evidence of better renal protection with ARB in patients with proteinuria.
BUT Do not combine ARB and ACEI
When compelling indications are present eg CHD then the appropriate drug is used
or avoided.
Reducing to abt <140 90 in elderly appears to achieve most benefits
In yg people consider <130
Average systolic reduction is 9mmhg with any one drug.
Doubling the dose DOES NOT DOUBLE the effect.
Diastolic reduction is 5.5mmhg with a single drug.
2/3rd of patients require at least 2 drugs
Diuretics have low compliance because of side effects
Single tablet Combined pills have higher compliance

Dear YL,

Summarising ARB vs ACEI

In choosing between ACEI and ARB therapy for patients with type 2 diabetes
diabetic nephropathy, you have to consider evidence of proven renal protective
benefit for ARB treatment versus evidence of a mortality benefit for ACEI
treatment shown in patients without established diabetic nephropathy.

ARB appears superior to delaying progression to renal failure.


But IN NON PROTEINURIC PATIENTS, ACEI has better results with regards to
mortality.

ACEI in general HPT patients has better outcome for CHD.

There is no routine role for combining both
You recall OnTarget trial. It was a huge trial testing Telmisartan vs Ramipril and its
combination. The angiotensin receptor blocker telmisartan was "noninferior" to
the ACE inhibitor ramipril in patients with vascular disease or high-risk diabetes in
this landmark trial

However, the combination of the two drugs was associated with more adverse
events without an increase in benefit.



Yin Ling asked
Why do we use normal saline in dka when many patients develop hyperCl as a
result?
Normal Saline 0.9% contains 150 mmol/l of sodium and chloride.

Hartmann's solution contains 131 mmol/l of sodium, 111 mmol/l of chloride, 29


mmol/l of bicarbonate (as lactate), 5 mmol/l of potassium, and 2 mmol/l of calcium.
Yes there is less chloride here.
Although Hartmann's solution has a lower chloride concentration and higher pH, its
routine use in diabetic ketoacidosis could be argued against for several reasons.
Firstly, people with diabetic ketoacidosis already have a high lactate to pyruvate
ratio, and the 29 mmol/l of lactate in Hartmann's solution could potentially
exacerbate this and lead to more adverse outcomes.
Secondly, Hartmann's solution raises plasma lactate and generates more glucose
from the lactate.
Thirdly, giving a solution containing even 5 mmol/l potassium to a patient who may
be hyperkalaemic could lead to potentially fatal cardiac arrhythmias, such as
bradycardia and asystole.
Fourthly, bicarbonate is not recommended for patients with pH greater than 7.0
because it could worsen the acidosis. Giving bicarb may paradoxically cause
intracellular acidosis due to diffusion of CO2 inside the cell.

Finally, because low serum sodium at presentation is a risk factor for developing
cerebral oedema, initial treatment with a relatively hypotonic fluid could be
harmful.
Thus, Hartmann's solution does not seem to be optimal for use here.

10) on Dyspnoea
Dear YL,

A 56 years old lady was brought by her daughter to see you. She has many years of
breathlessness and wheezing.
She used to smoke when she was young. She is now hypertensive and diabetic for
10 years. She has been coughing badly with purulent sputum the last 2 weeks. She
wakes up at 5am coughing and breathless.

She is on glicazide and metformin. She Is on an ACEI for HPT.

SHE USE A LADA PLUS Steroid INHALER And takes theophylline tablets.
Her dr gave her a macrolyte antibiotic for ' bronchitis' .
What will you do now?
Yin Ling: Thanks Prof. This lady is having an exacerbation of her obstructive airway
disease evident from the increase breathlessness/purulent sputum and cough.

What strikes me first is the macrolide antibiotic being given to her when she's taking
theophylline. i wonder which macrolide is it. did she bring it with her? We wouldnt
want to give her any erythromycin/clindamycin for the fear of theophylline toxicity
which will cause arrhythmia and seizure. Azithromycin is said to interact less with
theophylline and we commonly see this combination in GH.

Retake a thorough history to diagnose asthma, COPD and rule out sinister problems.
ensure no sx showing theophylline toxicity- GI problems, palpitations. assess her
recent control of her disease. is the LABA and inhaled steroid enough?

Examine her properly. is there any clubbing, cyanosis, her oxygen saturations, the
lungs. Any signs of pneumonia on examination, crepitations, wheezing. any signs of
heart failure. does she needs to be admitted.

A CXR is warranted. CBC and U&E would be helpful. ECG as well.

Take off her metformin for the time being and start her on a short acting B2 agonist,
a combination of B2 agonist and anticholinergics would be best if i diagnosed her as
having COPD. She can take it every 6 hourly for the time being.

She is on a LABA and inhaled steroids. i assumed she was being treated as asthma by
her doctor, based on her diagnosis and control, this preventer regime can be
augmented with a LAAC if her she is having COPD instead of asthma. A small dose of
theophylline will be helpful too.

A short course of oral steroids will be helpful. About 5 days.

As for antibiotics a respiratory quinolone would be better. Not ciprofloxacin. i would
use augmentin but why risk the GI side effects.

I will ask her to come back in 3-5 days or earlier is she's doing worse.
Dear Yin ling,

Be sure its not chronic heart failure tipped OVER with a LRTI. She's DM and HPT so
risk is high. Pls note she is NOT on a statin, does it matter what her value is?
She may not have chest pain bec of DM and yet have critical chd. A baseline ECG is

needed and further investigations for CHD may be needed..



Portable O2 sat measurement is available at abt Rm400. It shd be measured if
available. Ask her to blow out a match at 1 feet.

Old people may NOT have fever so that is NOT A GOOD Indicator of sepsis. ASK FOR
PREMORBID STATE FROM THE DAUGHTER!

It is often forgotten that smoking induces cytochrome P450 (CYP) 1A2, resulting in
altered concentrations and required doses of drugs metabolized by this route. Pls
name me some common drugs metabolized by this.

Conversely, upon cessation of smoking, concentrations of these drugs can rise to
toxic levels unless appropriate dose adjustments are made.

Doctors in Internal Medicine need to be aware of this, if potential iatrogenic harm to
patients is to be avoided.

The Inducers of P450
Carbemazepines
Rifampicin
Alcohol (chronic)
Phenytoin
Griseofulvin
Phenobarbitone
Sulphonylureas
Theophylline is metabolised by cyp 2a1.
Smoking induces cyp 2a1 while drugs like cimetidine, ciprofloxacin, erythromycin
and clarithromycin inhibits it and cause increase risk of toxicity
The cytochrome P450 enzymes are found primarily in the liver, although some (eg,
CYP3A4) are also found in substantial amounts in the intestine. They are involved in
the metabolism of most medications and are the mechanism by which most
pharmacokinetic drug interactions occur. Cytochrome P450 3A4 (CYP3A4) is the
superstar; it gets attention because a majority of drugs are metabolized by CYP3A4.

With regards to Asthma


1 in every 250 deaths worldwide! !
Prevalence in Msia is 4.53% in adults.
Selangor has the highest incidence followed by Johor!
10% of Msian adult asthmatics would have had an admission.
Phenotypically there are many variations eg Exercise induced asthma, aspirin
induced asthma, noctural asthma, etc
Patients must be taught to be Good Observers to note what precipitated their
asthma.
STOP EXPOSURE
REVERSE INFLAMMATION
TREAT BRONCHOCONSTRICTION
Anti inflammatory drugs and beta agonist drugs are the bedrock of treatment.
Inhaled steroids is the foundation of treatment to treat, and prevent recurrances
and to avoid fibrosis. Steroids increase the number of beta 2 receptors and enhance
the effect of beta agonists. Hence combination is synergistic.
Do NOT use LABA by itself!
Meta-analyses have shown that LABAs are associated with increased risk of overall
death when used as monotherapy.
The use of LABAs concomitantly with inhaled corticosteroids significantly reduces
asthma hospitalizations and is not associated with life-threatening events and
asthma-related deaths.
The evidence appears to support the use of LABAs plus inhaled steroids in a single
inhaler device for patients with moderate to severe asthma.

Yin Ling, pls tell me about Maintanance or Preventive treatment ie Asthma control
Patients may not know about 'maintainance' treatment in asthma. For most people
they take medicines when they are sick not when they are well.
Any patient that requires emergency treatment is poorly controlled.
Low dose theophylline potentiates ICS effects and hence has a role.

ICS plus LABA improves function and delays the time to relapse and is more effective
than increasing the dose of ICS alone. Using ICS alone takes time for symptom relief.
LABA plus ICS improves lung function while Increasing ICS alone delays
exacerbations but patient would not feel the symptomatic improvement that he
seeks. The LABA ALSO helps us use a lower dose of ICS.
With combination ICS and LABA therapy, increasing the ICS dose will further
improve lung function if symptoms persist.
The LABA should not be further increased for maintainance therapy.
Lipophilic LABAs will accumulate and may cause arrhythmias.
Hydrophilic LABAs has less accumulation.
It takes at least 3 months of ICS suppression for controlling and restoring the
bronchial epithelium to normal. Hence ICS must be given uninterruptedly for 3
months at a fixed dose.
After that the maintainance dose may be reduced and titrated as per patient. The
ICS may be reduced by 50% while continuing the LABA. LOW DOSE TREATMENT is
needed for maintaining the absence of airway inflammation.
Whether to stop the LABA is debatable.
Typically 2 weeks before a relapse the PFR will start to decline and the beta
receptors down regulated. Oral and ICS up-regulates beta receptors And that is why
it is so effective.
Oral prednisone or iv hydrocortisone takes about half an hour to act. Patients who
are still unwell after 3 doses of the Reliever medication must seek medical attention
and while on the way, take their emergency allocation of prednisolone.
20 puffs of the Reliever MDI In a spacer device is equivalent to a single dose of
nebulised beta agonist.

11) on GRAVES DISEASE IN PREGNANCY


Dear yin ling
Happy CNY to my 'liver' daughter.
A young lady of 25 has Graves Disease. She is newly married and not on
contraception. Presently she is on carbimazole 10mg daily and is euthyroid. She has
been on treatment for the last 1 year. She has mild proptosis and a moderate size
goitre.

At follow up you attend to her. What will you do for her?
When hyperthyroidism is not controlled in pregnancy, complications include
miscarriage, pregnancy-induced hypertension, premature birth, low birth weight,
intrauterine growth restriction, stillbirth, thyroid storm, and maternal congestive
heart failure. Therefore, proper diagnosis and treatment ofGraves' disease and
hyperthyroidism during pregnancy is essential. Your answer MUST start with pre
pregnancy treatment of women who have Graves' disease, they should conceive
only after they are euthyroid. Pls strongly recommend contraception until this is
achieved, and recommend that physicians offer counseling to women regarding the
implications of treatment on conception plans.

TRAb levels tend to rise after Radiodine and remain elevated. For this reason Mx
on pregnancy post RAI is more problematic and in Msia we do not usually use this
in young women. Our follow up treatment is not so great and patients often default.
If RAI is performed, a pregnancy test should be done 48 hours prior to the RAI
administration. When she says NOT PREGNANT, better TEST!
For antithyroid drugs, women should be informed about the risks associated with
propylthiouracil (PTU) and Carbimazole, and if these drugs are used, PTU should be
used in the first trimester of pregnancy. Carbimazole poses risks to the fetus if used
in the first trimester as you noted.
Kiv discontinuation of PTU after the first trimester and switching to carbimazole,
to decrease the risk of liver disease associated with PTU.

Hence, The primary treatment for hyperthyroidism during pregnancy is antithyroid
drugs. Because antithyroid drugs cross the placenta, care needs to be taken with the
use of antithyroid drugs during pregnancy. In particular, the main concern is the
ability of carbimazole to cause congenital malformations eg aplasia cutis - these
complications are not associated with the use of PTU.

PTU, however, carries a risk of liver toxicity, and hence let PTU be used in the first
trimester, and that patients switch to carbimazole in second semester.
PTU crosses the placenta and breast milk less.

Beta blockers are not typically recommended during pregnancy, as they are
associated with intrauterine growth restriction, low fetal heart rate, and
hypoglycemia in newborns.

All woman taking antithyroid drugs during pregnancy MUST undergo regular
monitoring of Free T4 and TSH, so that the Free T4 values remain at, or just above
the upper limit of normal, while taking the lowest possible dose of antithyroid
drugs. Free T4 and TSH should be measured every two to four weeks at the start of
treatment, and every four to six weeks after, to achieve the target blood levels.
PLS remember the limitations of TSH tests on pregnancy.

Typically, because hyperthyroidism often normalizes during pregnancy bec of
higher requirements and volume of dilution, antithyroid drugs can be discontinued
in the third trimester in as many as 20% to 30% of patients.

For women who have high TSH receptor antibody (TRAb) levels, they may need to
continue with antithyroid drug treatment until delivery bec these Abs may cause
fetal hyperthyroidism as you pointed out.

Thyroidectomy for Graves' Disease During Pregnancy
If a woman is allergic to antithyroid drugs, requires high doses to control
hyperthyroidism, or is not following her drug therapy, thyroidectomy -- is needed,
the optimal time is during the second trimester of pregnancy.At the time of
surgery, the TRAb levels should be measured to assess the potential risk of
hyperthyroidism in the fetus.

There are a number of risks to the fetus including:
fetal hyperthyroidism
neonatal hyperthyroidism
fetal hypothyroidism
neonatal hypothyroidism

The factors that can affect fetal risk include: poor control of hyperthyroidism
throughout pregnancy, which can cause transient central hypothyroidism in the
fetus, high doses of antithyroid drugs, which can cause fetal and neonatal

hypothyroidism, high levels of serum TRAb which can cause fetal or neonatal
hyperthyroidism.
Fetal and neonatal hyperthyroidism occurs in between 1% and 5% of all pregnant
women with an active or a past history of Graves' hyperthyroidism.

In a pregnant woman who has an active or past history of Graves' disease, TRAb
should be measured by 20 to 24 weeks of gestation. A value that is more than
three times the upper limit of normal is considered a marker of risk to the fetus.
Keypoints:
Patients who are receiving treatment should receive pre-conceptual advice with a
view to optimal preparation prior to pregnancy. This includes ensuring they are
euthyroid prior to conception and altering medication to PTU which is felt to be
superior to carbimazole during pregnancy, especially in the first trimester due to
reduced incident of aplasia cutis.

Current evidence suggests that following organogenesis, carbimazole or
methimazole should be re-introduced due to a possible increased risk of hepatitis
with PTU.

Those on a block and replace regime should also be swapped to PTU alone as
thionamides will cross the placenta but levothyroxine will not, thus increasing the
risk of foetal goitre and hypothyroidism.

Pregnant patients on treatment should have frequent TFTs throughout pregnancy
(monthly) and the dose reduced to the lowest possible to maintain euthyroidism
with T4 at the upper limit of the reference range.
Doses are reduced in the latter stages of pregnancy, and not infrequently stopped
altogether as the condition undergoes remission. If hyperthyroidism is secondary to
Graves Disease (or patient has had previous definitive treatment such as surgery or
RAI) then TSH receptor antibodies should be measured as high titres can indicate
intrauterine or neonatal thyrotoxicosis.


With Dr Donald Alexander, Reader in Medicine, University of Glasgow. This is the
Giant who did much research on the radio assays of the Thyroid and who advocated
the Block and Replace method of treatment. He took me under his wings as a Post
Graduate Fellow in Internal Medicine.

Yin Ling: Fetus thyroid is underdeveloped in early pregnancy ie first trimester and
rely on mothers transplacental T4 which is Important for neurodevelopement.
Thyroid stimulating antibodies can cross the placenta.
The fetal thyroid begins concentrating iodine at 10-12 weeks of gestation and is
controlled by pituitary TSH by approximately 20 weeks of gestation. Fetal serum
levels of TSH, TBG, FT4, and free triiodothyronine (FT3) increase throughout
gestation, reaching mean adult levels at approximately 36 weeks of gestation.
Maternal Thyroid stimulating hormone does not cross the placenta, and only small
amounts of thyroxine (T4) and triiodothyronine (T3) cross the matured placenta. In
neonates with congenital hypothyroidism, enough maternal thyroid hormone
crosses the placenta to maintain cord blood thyroid hormone levels at 25-50% of
normal. However, thyrotropin-releasing hormone (TRH), iodine, and TSH receptor
immunoglobulins do cross the placenta, as do the thioamides propylthiouracil (PTU)
and carbimazole/methimazole
During pregnancy, there is increase in Thyroid binding globulin from estrogen
stimulation, increase loss of Iodine in urine and increase deionisation by the
placenta. There is also stimulation of bHCG on the TSH receptor. Thyroid hormone
requirement in early pregnancy is increased. That is why in early pregnancy T4 need
to be increase by about 50%. The fetus is relying 100% on mothers thyroxine for its

early development. T4 cannot cross the placental barrier freely after it has been
well formed by 11-12 weeks. Only thyroid antibodies can. (Previously it was thought
that T4 and T3 cannot cross the placenta).
Serum levels of thyroid stimulating hormone (TSH), which is produced by the
pituitary gland, tend to decrease in the first trimester of pregnancy; during the first
trimester, serum TSH drops to undetectable levels in up to 15% of normal
pregnancies. By the onset of the second trimester, however, serum TSH levels return
to normal. This suppression in serum TSH values in the first trimester is secondary to
negative feedback from elevated levels of hCG, which binds to thyroid gland TSH
receptors. High levels of hCG also can occur when there is an abnormal pregnancy,
such as in cases of hydatidiform mole, and when a woman suffers from pernicious
vomiting of pregnancy or hyperemesis gravidarum (HG).
The rise in hCG may cause high elevations of free thyroxine (FT4) concentrations, as
seen is thyrotoxicosis. The thyroid gland enlarges slightly in normal pregnancy


Close monitoring with free T4 and TSH will guide our treatment but be aware of the
delay in pituitary response when looking at TSH. Propranolol can be used to treat
the symptoms but in pregnancy we wouldn't want to use a beta blocker because of
the Side effects of SGA and IUGR. Cord blood must be sent for t4 and TSH and G6PD.
Results must be reviewed before discharge.

Block and replace regimes are not commonly used in pregnancy. It requires higher
doses of antithyroid drugs which can cross the placenta and causes hypothyroidism
in the fetus. Only if the patient has a large amt of thyroid antibodies which can cross
the placenta and causes hyperthyroidism in the fetus that we can give PTU to the
mother in the hope that it too will cross the placenta and being down the thyroid
levels. And then we replace the iatrogenic hypothyroidism in the mother with
thyroxine.
Dr Hu Mung Chee: During pregnancy, reference ranges for thyroid-stimulating
hormone (TSH) are lower because of the cross-reactivity of the alpha subunit of
human chorionic gonadotropin with the TSH receptor. Changes in serum-binding
protein levels can influence measurements of free thyroxine (FT4) that rely on
estimates rather than direct measurements, resulting in inaccurate reported values.
Physicians should know the limitations of locally available assay methods. When
preferred FT4 assay techniques are unavailable, a serum TSH level is a more
accurate assessment of maternal thyroid status, and measurements of total
thyroxine and the FT4 index can be used instead.

~ American Family Physician. 2014 Feb 15;89(4):273-278
In spite of the limited value of serum TSH levels, which are normally low in the first
trimester, serum TSH determination is the most practical screening test for
evaluation of thyroid function. With rare exceptions, a high TSH value is consistent
with the diagnosis of primary hypothyroidism, while a low TSH suggests
hyperthyroidism

Drugs: Note that about half the patients with Graves' disease who are given
antithyroid drugs are still in remission one year after treatment is stopped. There is
now a large body of work which shows that thyrotrophin receptor antibody levels,
central to the aetiology of Graves' hyperthyroidism, fall during antithyroid treatment
and that remission may be related to this fall in a fashion which is dependent on the
dose and duration of treatment. This immunosuppressive effect is supported by
experimental data and antithyroid drugs may modify the natural history of Graves'
disease and contribute to the remission which occurs in a proportion of treated
patients.

12) on BASICS
Dear Yin Chok,
& To Ian my nephew reading Medicine in Canberra,

If I were teaching you Ian, I would had been VERY Hard on you. I would have ZERO
Tolerance for errors and omissions. I would have been doubly demanding of you
compared to other students bec you are my nephew.
I have been hard on yin ling too for my wife and I treat her like a daughter. As for my
other students, Modern propriety demands that I tone down 3 gears.

A generation ago when Msian candidates sat for the MRCP exams in UK, we were
not afraid of the clinical cases as we were well exposed to many clinical problems in
our daily work. But theoretical questioning can be a problem as we may not be
familiar with the FASHION of questions in voque.

A good friend was asked. 'What Is the chemical test used to determine the Hb level?'
for his viva. Yr1 biochem. Did the test ourselves but Good heavens what's the name
of the damn chemical? He failed.

I did this experiment in year1 biochem and in yr4 obs posting we the minion med
student had to check the patients Hb before they can be discharged. So it's lasered
into our brainstems.

The patient's blood is mixed with Drabkin's solution, a solution that contains
ferricyanide and cyanide. (Risked my life studying medicine!) The ferricyanide
oxidizes the iron in the hemoglobin, thereby changing hemoglobin to
methemoglobin. Methemoglobin then unites with the cyanide to form
cyanmethemoglobin. Cyanmethemoglobin produces a color which is measured in a
spectrophotometer. The color relates to the concentration of hemoglobin in the
blood which we obtain by comparing with a standard chart.
Today I am going to ask you a series of basic questions.

1. Your patient has ESR 115. What are your thoughts?
YL: There's only a few causes for three digits esr. Multiple myeloma, GCA(temporal
arteritis), advanced malignancy, TB and SLE (connective tissue disease) and also in
severe sepsis.

Prof : And we must not FORGET causes of a LOW ESR too!



Lower-than-normal levels can occur with:

Congestive heart failure
Hyperviscosity
Hypofibrinogenemia
Low plasma protein (due to liver or kidney disease)
Polycythemia
Sickle cell anemia
The rate of fall of red cells in anti-coagulated plasma is influenced by a number of
factors. Sedimentation is influenced greatly by the extent to which the red cells form
rouleaux or large clumps of cells. This rouleaux formation is related to the plasma
proteins which overcome the negative surface charge on red cells.

The haematocrit also influences the ESR. Anaemia accelerates sedimentation and
polycythaemia retards it.

Are you all aware that Aging alone elevates it? Perhaps some low grade
inflammation kills us all eventually. Upper limit of Normal for males is Age divide by
2 and for Females is Age PLUS 10 divide by 2!

Are you aware that merely being Diabetic also increases it? Maybe the same reasons
cause that.
With increasing age after 50 years, the ESR rises and, in the elderly, many apparently
normal subjects have increased readings. But it is NOT 3 figure values.

Physiological increases in the ESR occur in pregnancy and the puerperium.
The ESR is a measure of the presence and severity of inflammatory, auto immune
and other morbid processes.

NOTE THAT a normal ESR cannot be taken to exclude disease

Most acute or chronic infections, neoplastic diseases, collagen diseases, renal or
other diseases associated with changes in plasma proteins lead to acceleration of
sedimentation. It is also useful in monitoring disease activity in certain disorders. It
has been used for this purpose in tuberculosis, rheumatic fever, rheumatoid
arthritis, Hodgkin's disease and non-Hodgkin's lymphomas, myeloma and
macroglobulinaemia.

The investigation of a very high ESR involves full clinical assessment of the patient
which will usually reveal the cause. There are a number of well known associations
with a high ESR which include what had been stated and the following:

Recent respiratory tract infection and anaemia due to mycoplasma infection
Cardiac bacterial endocarditis is impt as it may be low grade and missed. Be
careful in drug addicts.
Multiple myeloma is THE CLASSIC cause never to be missed when grilled. Or come
back next year.

2. The LFT came back. As HO you review the results. AST more than ALT by 3 times.
Thoughts and why?

ALT more than AST 2 times. Thoughts and why?

AST equal to ALT. Thoughts?
YL: AST is mainly in the mitochondria while ALT is cytosolic. Inflammation to
hepatocytes that break down cell membrane will release much more ALT compare
to AST. Eg hepatitis, drug induced inflammation. Cirrhosis will cause hepatocytes
death which release more AST likewise with MI.

AST 2 times more than ALT is specific for alcoholic liver disease as alcohol induces
AST.

AST and ALT equivalent can mean liver cell death or can be seen in v chronic liver
disease when there is not much increase in both transaminases.


3. Your patients Urea is elevated but Creatinine is normal. Thoughts and why? ?

4. Se calcium is tested below normal. Patient Is well with normal ECG. What are
your thoughts?
5. Se Alk phos high and se calcium high

6. Se Alk phos normal and Se calcium high

7. Se calcium normal and se Alk phos high

Dear yin ling,



there are some unique aspects of elevated ALKALINE PHOSPHATASE

Body sources of alkaline phosphatase include neo-osteogenesis of bone, (NEW
BONE FORMATION IS NEEDED) intestines, liver, placenta, and white blood cells.

Often elevated levels are associated with complications of the liver or gallbladder.
Gamma glutamyl transferase (GGT) will determine hepatic versus other origins of
alkaline phosphatase elevation.

In the presence of liver disease, increased levels of Gamma glutamyl transferase is
seen but in persons who have recently ingested even minute amounts of alcohol it is
elevated and is therefore a less specific test. Isoenzyme testing is also an option to
determine the source of alkaline phosphatase elevation.

An elevation of hepatic alkaline phosphatase usually implies biliary tract pathology
with resultant cholestasis, but this elevation also occurs with infiltrative diseases of
the liver, masses or abscesses of the liver,primary biliary cirrhosis and primary
sclerosing cholangitis.

Where alkaline phosphatase elevation is secondary to bone disease, its elevation is a
marker of osteoblastic activity. In bony disease, the alkaline phosphatase elevation
is usually associated with hypercalcemia. Common bone diseases include metastatic
bony lesions, Pagets disease, secondary hyperparathyroidism of renal disease,
osteomalacia, rickets, primary hyperparathyroidism, and even hyperthyroidism. I
had patients with Graves who presented with high Alk phos which normalised on
treatment.

BUT the exception is multiple myeloma where hypercalcemia is secondary to
osteoclastic bone disease and the alkaline phosphatase level is normal. I found this
fascinating as a student.

Clinically, pathologic fractures of the sternum are considered pathognomonic of
multiple myeloma, and bone pain of the ribs or back that is exacerbated by
movement is considered characteristic. Add raised se calcium, high esr, normal alk
phos and raised globulins and I will knock your head if you cannot diagnose.

During pregnancy, maternal alkaline phosphatase increases. This is the result of

placental alkaline phosphatase production, and the elevation resolves


puerperium.
Dear Yin ling,

I had only seen 1 patient with very high Direct Bilirubin and Normal Alk phos.
This is in a patient with a leaking T tube. Direct conjugated bilirubin leaked into the
peritoneum and was absorbed back into the circulation

8. Patient is clinically suspected to have malaria. Consultant asked for BFMP X 3
The HO dutifully on 3 days took 3 samples and all 3 were negative. What could be
wrong?
Dear Yin ling,

Taking blood films for malaria parasites shd be done when parasitaemia is highest.
This is when fever Is rising. So it is not the HO merely taking blood from a vein when
it is convenient to him.

Up to 3 specimens can be taken in the same day if fever is noted to be rising. Hence
for simplicity this is done via a finger prick.
A thick film for screening and a thin film for identification is made each time. HOs
must know these techniques as malaria screening is common in Msia.

Pls make sure that the alcohol used to clean the finger has DRIED before you prick
the finger at the side of the distal phalanx. To ensure that the thick film is not too
thick pls apply the blood to the slide from BELOW ie the underside of the glass
slide. You can control it easier this way instead of dropping the blood.

The usual horror story when BFMP X 3 is ordered is that the HO takes a syringe and
draws a venous specimen at end of ward rounds and proceed to make 3 sets of
slides.


9. Patient had features of filiariasis. Consultant asked for blood films for the
parasite. The HO took 3 specimens on his shifts. What is wrong?

Dear Yin ling,



What is the brand of the watch you wear on your wrist?
Because I will like to know how the filiaria worm tells time!

Bancroftian and brugian filariasis tend to show nocturnal periodicity, so it is
recommended that samples be collected between 10:00 pm and 2:00 am. Yup its a
midnight show.

So the dear HO who pokes a vein after rounds Is not going to find anything.

Its also a fingerprick capillary specimen and similar Thick and thin films but done
literally by a Vampire HO at the stroke of midnight.

13) on FEVER
Dear YL,
What is fever?
What is the mechanism behind fever?
Why can some people with no obvious illness have fever?
Why do some people with illness have NO fever?
I want to EMPHASIZE strongly that the elderly may not have the ability to develop
Fever when ill. Absence of fever does not mean anything
Chee Yong Chuan: Fever is defined as the elevation of core body temperature above
normal. In normal adults, the average oral temperature is 37 degrees Celsius.

The febrile response is a complex physiologic reaction to disease involving a
cytokine-mediated rise in body temperature and generation of various acute phase
reactants. Fever is thus considered a hallmark of IMMUNE SYSTEM ACTIVATION,
resulting in a regulated rise in body temperature. So what happens is that
exogenous pyrogens from infectious agents, toxins, tumours induce production of
pro-inflammatory cytokines, such as interleukins, TNF which subsequently enter the
hypothalamic circulation and stimulate release of local prostaglandins(this is where
our antipyretics and NSAID exert their effects). The body will then react to rise the
temperature to this new thermal set point(manifested by chills and shivering).

Why do some people with illness have no fever? I can observe that response to
fever varies with age. Elderly patients especially are unable to regulate their body
temperature to the same degree as young adults. Prof Wong had reiterated many
times that older patients with serious infections have substantial prevalence of a
pyrexia or LOWER febrile responses! Don't be surprised to see hypothermia instead
in full blown sepsis in this group of patients. Fever is also considered to be an
important host defence mechanism, hence in those who are immunocompromised
i.e HIV, patients receiving steroid therapy, neutropenic patients, due to the inability
to mount an adequate immune response, might not give you the textbook febrile
response that you would have expected. I can think of a few examples where
patients who are well but developing fever.

1) Transfusion associated fever. Again, due to activation of the immune system
against antigen on the donor blood
2) Drug induced, probably affects the ability of the body to dissipate heat, or

through immune system activation, serum sickness, allergy


3) Factitious fever

Prof : difference between fever and hyperthermia



Prof : Dear Yin Ling,

I hate all these definitions as my RAM is simply too small to process them. I think of
FEVER as a problem but you are sitting for exams and hence stuck in the system.

DEFINITIONS
FUO>38.3C [>101.8F], duration >3 weeks, diagnosis uncertain after 3 days in
hospital or "three outpatient visits"!! This is close to our old definition.

NOSOCOMIAL FUOhospitalized patients, >38.3C [>101.8F], diagnosis uncertain
after 3 days and infection not present or incubating on admission

IMMUNE-DEFICIENT (NEUTROPENIC) FUO >38.3C [>101.8F], >3 days, neutrophil
count <500/mm3.

HIV-RELATED FUOHIV patients, >38.3C [>101.8F], duration >3 weeks for
outpatients or ">3 days for inpatients"
The era of high technology has changed the goalposts considerably, note inpatients

now are THREE days to a diagnosis. With every scan thrown in from PET CT to MRI, 3
days in the IDEAL hospital appears enough.

FEVER, NYDpersistent fever that has not yet met the definition for FUO.

In the HISTORY
-the pattern and duration of fever,
-the associated symptoms (cough, dyspnea, hemoptysis, chest pain, diarrhea,
abdominal pain, dysuria, urethral discharge, hematuria, neck stiffness, headache),
-any rash (palpable purpura, exanthem),
-any exposure (food, water, plants, animals, insects, infected human secretions),
-weight loss, night sweats,
-travel history, sexual history, HIV risk factors, immunizations,
-past medical history (rheumatologic disorders, malignancy, alcohol),
-medications
are ALL CRUCIAL.

PHYSICAL exam
vitals (tachycardia, tachypnea, hypotension, fever, hypoxemia),
oral ulcers, lymphadenopathy,
nuchal rigidity,
respiratory and cardiac examination (murmurs),
temporal artery,
abdominal examination (hepatosplenomegaly),
prostate examination,
skin lesions (morphology, distribution), insect bite marks,
joint examination

Always think of:

INFECTIONSTB (pulmonary, extrapulmonary, miliary),
abscess (liver, splenic, perinephric, psoas, diverticular, pelvis), osteomyelitis,
endocarditis

NEOPLASTIChematologic (lymphoma, leukemia, multiple myeloma,
myelodysplastic syndrome), solid tumors (renal cell, hepatoma)

COLLAGEN-VASCULARvasculitis (giant cell arteritis, Stills disease, polyarteritis

nodosa, Takayasus arteritis, Wegeners granulomatosis, mixed cryoglobulinemia),


lupus, rheumatoid arthritis

DRUGSantimicrobials (sulfonamides, penicillins, nitrofurantoin, antimalarials),
antihistamines, antiepileptics (barbiturate, phenytoin), NSAIDs/ ASA,
antihypertensives (hydralazine, methyldopa), antiarrhythmics (quinidine,
procainamide), antithyroid, iodides, quinine, illicit (cocaine)

UNCOMMON CAUSES OF FUO
endocrine (hypothalamic dysfunction, hyperthyroidism, pheochromocytoma,
adrenal insufficiency),
infections (dental abscess, leptospirosis, psittacosis, melioidosis, syphilis,
gonococcemia, hereditary periodic fever syndromes (familial Mediterranean fever,
alcoholic hepatitis, hematoma, factitious fever

Pls do not forget the good old CLASSIC DEFINITION (1961)>38.3C [>101.8F],
duration >3 weeks, diagnosis uncertain after 7 days of investigation in hospital

Prof : Malignancies have now superseded infections as the most common cause of
fever of unknown origin (FUO). Did you know this?
In the past infectious diseases were the most common etiology of FUO, and
neoplasms constituted the second most frequent category. This shift from infectious
to malignant etiology as the most frequent cause of FUO is related to several
factors.

Firstly, due to the widespread introduction of computed tomography (CT) and
magnetic resonance imaging (MRI), many intra-abdominal causes of infection are
diagnosed early and therefore do not meet the definition of FUO. Sub phrenic
abscesses, pelvic pathologies and even sinus infections well hidden from the
clinician Is not exposed clearly.

Secondly, radionucleotide imaging studies, that is, indium scans, gallium scans, and
bone scans, have been useful in identifying occult malignancies undetectable by
other means.

Now PET CT has made detection of both well hidden infections and malignancies
much easier.

Prof : lymphoma is the most common etiology of neoplastic fever of


underdetermined origin. The pathophysiology of tumor-induced fever may be due
to several mechanisms of which release of cytokines from tumor cells or tumor
necrosis factor and interleukin-1; necrosis of tumoral tissue; all contribute.
I cannot resist asking you, What is the Naproxen challenge in FUO? ?

THE NSAID naproxen Is very effective in suppressing tumor fever and this property
may be useful in elucidating suspicion of cancer in patients with prolonged,
undiagnosed fever.

Naproxen IS THE classically touted agent for suppressing tumor fever due to its
unique ability to suppress tumoral cytokines in preference over infectious
cytokines.

While the naproxen challenge may be useful in evaluating prolonged fever
suspected to be of neoplastic origin, it must be utilized in the context of a thorough,
clinically-driven assessment.

14) NMS
A 66-year-old male was hospitalized for increasingly aggressive behavior. He had no
prior psychiatric admissions. On the day of admission after he sustained a fall, a CT
scan of the brain revealed a subarachnoid hemorrhage at the right superior sulcus
and a possible hemorrhagic contusion at the left frontal lobe.

Over the course of hospitalization, the patient had a series of CT scans showing
resolution of the hemorrhage.

He was started on olanzapine for intermittent agitation. Olanzapine was titrated to
7.5 mg daily.Ten days later the patient became abruptly somnolent with body
temperature reaching 39.7 C and severe muscle rigidity in both upper and lower
extremities. He had severe diaphoresis and fluctuation of blood pressure and pulse.

Laboratory data revealed elevation of white blood cells to 14800 K/L, creatine
phosphokinase to 2800 U/L (normal < 174 U/L), and mild elevation of serum alanine
and aspartate aminotransferase. MRI of the brain, CSF studies, and chest radiograph
were unremarkable.

What is the diagnosis?


What will you do now?
Pathophysiology fr Medscape

'' The most widely accepted mechanism by which antipsychotics cause neuroleptic
malignant syndrome is that of dopamine D2 receptor antagonism. In this widely
accepted model, central D2 receptor blockade in the hypothalamus, nigrostriatal
pathways, and spinal cord leads to increased muscle rigidity and tremor via
extrapyramidal pathways.

Hypothalamic D2 receptor blockade results in an elevated temperature set point
and impairment of heat-dissipating mechanisms. Peripherally, antipsychotics lead to
increased calcium release from the sarcoplasmic reticulum, resulting in increased
contractility, which can contribute to hyperthermia, rigidity, and muscle cell
breakdown.

Beyond these direct effects, D2 receptor blockade might cause neuroleptic
malignant syndrome by removing tonic inhibition from the sympathetic nervous
system. The resulting sympathoadrenal hyperactivity and dysregulation leads to
autonomic dysfunction. This model suggests that patients with baseline high levels
of sympathoadrenal activity might be at increased risk. While this has not been
proven in controlled studies, several such states have been proposed as risk factors
for neuroleptic malignant syndrome.''
Diagnostic features of NMS:

Neuroleptics within 1 to 4 weeks.
Hyperthermia (above 38).
Muscle rigidity lead pipe.
Five of the following:

Changed mental status.Tachycardia.Hypotension or
hypertension.Tremor.Incontinence.Diaphoresis (excessive sweating) or sialorrhoea.
Increased creatine phosphokinase (CPK) or urinary myoglobin.
Metabolic acidosis.Leukocytosis.
Exclusion of other illnesses (neuropsychiatric, drug-induced, systemic).

Prof : Have you heard of 'an extreme Parkinson's crisis' ?



It may be possible that NMS is as an extreme parkinsonian crisis resulting from
overwhelming blockade of dopamine pathways in the brain.

In this view, NMS resembles the parkinsonian-hyperthermia syndrome that can
occur in Parkinsons disease patients following abrupt discontinuation
or loss of efficacy of dopaminergic therapy, which can be treated by reinstituting
dopaminergic agents.

Evidence to support this view includes:

Parkinsonian signs are a cardinal feature of NMS

. Withdrawal of dopamine agonists precipitates the syndrome.

All triggering drugs are dopamine receptor antagonists.

Risk of NMS correlates with drugs dopamine receptor affinity.

Dopaminergic agonists may be an effective treatment.

Lesions in dopaminergic pathways produce a similar syndrome.

Patients with NMS have demonstrated low cerebrospinal fluid concentrations of
the dopamine metabolite homovanillic acid.

Yin ling: this patient is likely having what we know as a Neuroleptic Malignant
Syndrome due to the Olanzapine that was just started ten days ago.

Neuroleptic malignant sydrome is caused by drugs causing Dopamine receptor
antagonists (commonly antipsychotics) and also withdrawal of Dopamine agonists
(commonly anti Parkinsonian drugs). temperature regulation became haywire and
patient presents with classic tetrad of hyperthermia which is unable to be brought
down by antipyrogens, RIGIDITY, mental confusion/somnolence/coma, and
autonomic dysfunction (perfused sweating, tachycardia, labile BP and etc). it can
occur after a few days of taking the drugs up to years after that. classically it occurs
within a few weeks.

Labs wise: increase total white and v high CK, deranged transaminases and decrease
in se iron is classical for NMS.

Stopping the culprit drug is first and foremost. Secondly bring down the patients
temperature with methods like cooling blanket, hydration, and medical therapy with
dopamine agonists like bromocriptine can help. Dantrolene a muscle relaxant is
used too but not available in many hospitals. Supporting patient's ABC is of course
vital

NMS has been claimed to have some similar genetic profile as Malignant
Hyperthermia that occurs when anaest drugs classically succinylcholine is
administered. MH is autosomal dominant and a drug history will help us with
diagnosis.

It is often difficult to differentiate Serotonin syndrome and NMS. even more so
when serotonin containing drugs will also have dopamine antagonist effect.

Serotonin syndrome is due to a high level of serotonin in the body caused by giving
two serotonin containing drugs concurrently, giving these drugs tgt with CYP 2D6/
3A4 inhibitors OR giving a Long Acting drugs which increase serotonin eg
Prozac(fluoxetine). classical drugs that causes SS are antidepressants like SSRI, SNRI,
MAOi-A (MAOi-B that we are giving to our parkinsons disease patients won't cause
that effect. i don't know why). other common drugs are PAIN CONTROL drugs eg
Tramadol, fentanyl, meperidine; ANTIEMETRICS eg Maxolon, granisetron(Kytril),
ondansetron; TRIPTANS, LITHIUM.

Pathophysiology of NMS and SS is different because SS does not cause
hyperthermia by altering the hypothalamus setpoint. it is due to the overall
hyperexcitability.

The few ways to differentiate:
1) Serotonin syndrome causes neuromuscular excitation : hyperreflexia, myclonus,
clonus, increase in bowel sounds, pupils dilatation, as opposed to LEAD PIPE
RIGIDITY in NMS.

2) Lab reuslts of increase total white and CK and transminases and decrease se iron
will point to NMS. SS less likely

3) SS can occur immediately after serotonin containing drugs is given while NMS

might occur after a few weeks.



Mx of SS : diazepam for the hyperexcitability, intubation and sedation may be
needed. anti serotonergic drug like cyproheptadine can be given.

Dr Hu : Neuroleptic malignantlike syndrome (NMLS)

Withdrawal or dose reduction of levodopa in patients with Parkinsons disease (PD)
has been reported to precipitate a potentially fatal syndrome closely resembling
neuroleptic malignant syndrome (NMS). This syndrome is referred to as neuroleptic
malignantlike syndrome (NMLS) and parkinsonism-hyperpyrexia syndrome by
some authors.

Clinical features of NMS and NMLS are very similar and include high fever, marked
rigidity, altered consciousness, leukocytosis, autonomic dysfunction, rhabdomyolysis
with elevated serum creatine kinase (CK) levels, and renal failure. NMS and NMLS
share a similar pathophysiologic mechanism consisting of an acute reduction in
nigrostriatal and hypothalamic cerebral dopaminergic transmission. In the case of
NMS, the primary mechanism occurs when neuroleptics block D2 dopamine
receptors, while the cause of NMLS is the withdrawal of exogenous dopamine. This
usually occurs in the form of its precursor, levodopa. Clinical conditions that may
contribute to the occurrence of NMLS include disturbance in the gastrointestinal
tract resulting in poor absorption of levodopa, intercurrent infection, dehydration
due to heat, and poor adherence to medications.

It is well recognized that high dietary protein intake can impair the absorption of
levodopa, leading to loss of efficacy and PD symptom fluctuations. The amino acids
of the protein may compete with levodopa for absorption in the gut and for
transport through the blood-brain barrier. (Health-care professionals should be
aware of the interaction between levodopa and protein content of enteral
nutrition to avoid the occurrence of NMLS in patients with PD.)

The treatment of NMLS due to levodopa withdrawal consists of supportive
measures, hydration, and Levodopa reintroduction or increase in dose can be
effective.

(The Annals of Pharmacotherapy 2010 September, Volume 44)

15) on Cardiac Tamponade


Yin Ling,
A Patient was stabbed in 4th intercostal space, left to sternal border, he appears
anxious, has marked difficulty breathing, with pain radiating down left arm, and the
veins in his neck distend upon inspiration.

What is the DIAGNOSIS?
YL: Kussmaul's sign for Cardiac Tamponade
Prof : Yes it is a Cardiac TAMPONADE
-this Results from accumulation of fluid/blood in the pericardial cavity-this
compresses the heart chambers
Because of poor venous return the Neck vein has distension= Resulting in
Kussmaul's sign.
- DECREASED venous return and REDUCED cardiac output



16) on THE ELDERLY
Dear Yin ling,

Elderly patients often come to the Physician with abd pain. Are you aware that the
Classical history and signs are often not seen in the elderly? Multiple factors
contribute to the diagnostic difficulty and high incidence of complications seen in
elderly patients.

Immune function tends to decrease with advancing age. Many elderly patients have

underlying conditions such as diabetes or malignancy, further suppressing immunity.



To make it worse Elderly patients often have underlying cardiovascular and
pulmonary disease, which decreases physiologic reserve and predisposes them to
conditions such as abdominal aortic aneurysm (AAA) and mesenteric ischemia.

The exam patient typically has multiple morbidities cutting across many disciplines.
Elderly patients also have a high incidence of asymptomatic underlying pathology.

Up to one half of elderly patients have underlying cholelithiasis something i see so
often, one half have diverticula, and 5-10% have AAA. Remember you see me
screening for this in all over 50

Understanding that elderly patients may present very differently than their younger
counterparts also is important. Elderly patients tend to wait much longer to seek
medical attention than younger patients as they are very money conscious, and they
are much more likely to present with vague symptoms and have nonspecific findings
on examination.

Many elderly patients have a diminished sensorium, allowing pathology to advance
to a dangerous point prior to symptom development.

Painless infarcts for eg is common. Elderly patients with acute peritonitis are much
less likely to have the classic findings of rebound tenderness and local rigidity. If they
come to see me THEY ARE SICK.

They are less likely to have fever, leukocytosis, or elevated C-reactive protein
level. In addition, their pain is likely to be much less severe than expected for a
particular disease.

Because of these factors, many elderly patients with serious pathology initially are
misdiagnosed with benign conditions such as gastroenteritis or constipation.
Have a high degree of suspision and a low treshold for Ix when managing them.
The Elderly HPT
65 TO 74. YOUNG OLD
75 TO 84. OLD OLD
85 TO 100 REALLY OLD
> 100 MIRACLE

90% of people more than 65 yrs old will develop hpt.


Increased thickness of intima and media as we age leads to arterial stiffness.
In the yg the artery is distensible so that the outflow is relatively steady.
In the old the wave travels faster bec of arterial stiffness with high systolic pressure
and low diastolic pressures. A high pulse pressure is seen. Isolated systolic hpt is
typical of elderly patients.
Active treatment in elderly is effective with reduction in CV mortality and stroke.
In the very elderly of > 80 with sbp > 160 sig event reductions is still seen with
treatment.
In trials systolic bp has not been lowered to <140 hence target is between 140 to
150
Diuretics and ccb are preferred esp in ISH but any class of drugs can be used

17) on BP AND MAP
Dear Yin Ling,

over dinner just now you sat in between as the Consultant Anaesthetist and I
chatted. The discussion was whether MAP is better or equivalent to Systolic diastolic
pressure in monitoring patients?

You did not say anything but what would you had said if this was asked of you in
your exam? You will have recalled that I contributed my understanding of its
physiology. Always the basic sciences.

MAP is considered to be the perfusion pressure of the organs in the body.
Hence I asked her in what context she was refering to and when she replied in
septicaemia; it is then obvious that MAP would be excellent as we are worried about
multi organ failure in septicaemia.

Typically when the MAP is greater than 60 mmHg, that is enough to sustain the
organs of the patient. That is why the Consultant referred to that figure and used 65
as her example.

The MAP is normally between 70 to 110 mmHg

If the MAP falls below this for an appreciable time, the organs will not get enough
Oxygen perfusion, and will become ischemic.
The MAP is the average over a cardiac cycle and gives us a slightly better idea of
perfusion when compared to looking at the systolic diastolic pressures alone.
Because diastolic pressure is important you will note that in our typical calculation,
the diastolic is multiplied by 2.

I also explained that in the non ICU setting, we look at systolic diastolic pressures
instead because the circumstances are different. In the clinic following up HPT
patients for example, we are concerned about CVS outcomes. We know that the SBP
or PP predicts CVD among older men. As I shared in my talk, the stolic BP and its
variability is very important. But as for MAP, we are unsure of its correlation with
outcomes unlike SBP and DBP which is highly correlated and better predicts CVD.

MAP is an alternative and preferable measurement to systolic blood pressure in
monitoring patients at risk for hypotension and the detection of organ
hypoperfusion. This is a fundamental clinical use which differs. In sepsis for eg,
tissue hypoperfusion is the pathophysiologic endpoint of low blood pressure, and
MAP, rather than SBP, is the physiologic driving force behind blood flow to organs
and tissues. This discussion over dinner could well be critical to your understanding
and hence I wrote to ensure that you do understand.

As BP progressively drops, a SBP of 80 mmHg or less becomes less sensitive and
physiologically less appropriate measurement of hypotension than MAP. The MAP
provides an objective assessment of hypotension that may precede hemodynamic
decompensation.

18) on BP Variability

1) In the clinic setting we are just taking a snapshot of the patients overall BP in 24
hours. How accurately does this reflect the 24hours BP? We are moving towards
home BP monitoring and ambulatory BP monitoring esp to detect masked
hypertension and white coat hypertension

2) Masked hypertension : group of patients with normal BP in clinic but in their daily
lives their BP are in the high side. easily missed group of patients hence high CVS
risk!

3) Normal healthy person has a dip of their BP when they are sleeping at night.
When we lose this dipping phenomena (patient who are diabetic, renal failure,
autonomic neuropathy, taking cyclosporin etc) OR WORSE, have an early morning
BP surge, have higher CVS risk!

4) Rothwell wanted to answer a question : whether high BP or BP variability poses
the risk for stroke. Results : BPV.

5) Hence we need a drug that can control BP for over 24 hours. a long acting
antihypertensive.

6) ASCOT trial compared a beta blocker (atenolol) and a CCB, in which CCB shows
greater benefit in lowering stroke. Of course, amlodipine was a long acting drug
compared to atenolol

7) JNC8 guidelines : for all patients we aim for BP target of <140/90 (this including
diabetic and CKD patients more than 18 years old) , except older patients >60 years
old, threshold is higher at 150/90mmHg.

8) Older ppl will often has a higher systolic BP and lower diastolic BP.

9) Older teaching : in CKD patients keeping BP <140/90 is our target, and for patient
with proteinuria, <135/85.

10) Most patient require more than 1 drug use to control BP : best is CCB and an
ACEi.

11) Amlodipine of 10mg OD will often cause ankle oedema, ACEi which causes
efferent vessels vasodilatation can counter this effect.

12) ACEi trumps ARB in lowering all cause mortality for all diseases that requires BP
lowering except in nephropathy. OnTARGET trial shows us ARB is as good as ACEi. In
nephropathy, ARB has more benefit.

13) Remember to keep BP around 160-180 post stroke, and Do NOT LOWER the
DIASTOLIC to <110mmHg. We need to save the penumbra.

14) Early morning BP need to be monitored before the patient takes their
medication. Early morning surges are associated with ACS, CVA.


15) Atrial fibrillation we CANNOT measure the BP using dynamap, inaccurate!!
even pulse rate will show a wrong reading.

16) Wrist monitoring BP is not as accurate as the prox arm BP moniroting because of
smaller vessels, except in one situation. when the elderly patient has obvious
sclerosed brachial vessels.

The HPT elderly has after some time a shift in the cerebral autoregulation to the
right. Hence when we treat them for HPT we have to bear this in mind as
overzealous reduction will bring the BP to the left of the auto regulation curve and
cerebral perfusion drops. We often hear the treated elderly complain of light
headedness, unsteadiness or confusion: be careful that we are not doing more harm
here.

Hence in elderly patients, I am happy if their BP is abt 150 or so and watch for
symptoms like a hawk.
The ASCOT-BPLA arm shows that antihypertensive therapy based on a "newer"
regimen of the calcium channel blocker amlodipine and the angiotensin-
converting enzyme (ACE) inhibitor perindopril confer significant advantages over a
"traditional" regimen of a beta-blocker, atenolol, and thiazide diuretic,
bendroflumethiazide (BFZ), in terms of effects on both cardiovascular mortality
and all-cause mortality. The amlodipine/perindopril regimen is also associated
with significantly fewer heart attacks and strokes.



Prof : I was asked

1. why is the 1 to 2pm BP most representative of MAP?

2. To make sure patients dont get into trouble is it not better to check the Morning
BP surge and to as it were "treat" that and maintain that to near normal rather to
maintain the 1 to 2 pm BP at normal?
What is the early morning surge of BP seen in hypertensive patients; how do we
define it?
By defination a systolic BP >55mmhg of the ''highest morning systolic BP minus the
lowest nighttime systolic BP'' IS EARLY MORNING SURGE.


Ambulatory blood pressure monitoring (ABPM) has shown us that 24-hour average
blood pressure values bear a limited correspondence with office BP values. The
correlation coefficients between office systolic BP or diastolic BP and the
corresponding 24-hour average values are rarely >0.50

Pls note that this is Average BP which differs from MAP.

The 1 to 2 pm measurement approximates to the Average BP, this has nothing to do


with the MAP figures.

The organ damage accompanying hypertension is more closely related to 24-hour
average than to office BP. This is the case regardless of whether the damage is in the
heart (left ventricular hypertrophy or dysfunction), in the kidney (proteinuria), or in
the brain (cerebral lacunae or white matter lesions.
The end-organ damage of hypertension is similarly related to daytime, nighttime
and 24-hour average ABPs.

Your second Q is something I wished no one would ask!! This is because this is
where EBCP is a headache.

BP falls during the night because of the reduction of sympathetic activity (and the
increase in vagal drive). The early morning surge in BP "Matches" with a peak
incidence of myocardial infarction, sudden death, and stroke in the morning hours
which is why an enhanced morning BP rise is widely regarded as an adverse
phenomenon that needs to be counteracted by the BP lowering effect of treatment.
Sounds rational right? I think so too.

However, no other evidence exists that morning BP rise is risky; NO Interventional
TRIALS to boot, therefore, at present, its relevance for the peak morning incidence
of cardiac and cerebrovascular events is only of observational data and speculative.

Furthermore, several other phenomena potentially dangerous for the heart and the
brain (heart rate, fibrinolytic activity, platelet aggregability, circulating
catecholamines, etc), also show peaks in the morning that may make the morning
BP rise a pathophysiological cofactor.
Our treatment hence is to homogeneously lower the whole 24-hour BP profile, and
thus also without any adverse interference with the physiological morning BP rise
unless it is unacceptably high. Perhaps nature has intended a rise for function.

The nocturnal fall in BP can vary widely among individuals, which has led
hypertensive subjects to be classified into 2 categories, ie, those whose nighttime
average BP falls more than 10% of the average daytime value and those in whom it
falls less.

With the use of this classification, in hypertensive subjects in whom nocturnal BP

falls less than 10% (known as nondippers), organ damage is much greater than in
those in whom it falls more than 10% (known as dippers), and that this is the case
also for the organ damage progression.

Yin Ling: Hypertension
1. One Third of the population above 18 years old is hypertensive
2. 20% not dx and treated
3. 65% treated but not controlled!
Overall only 14 % is controlled
4. COMBINATION therapy can be started in pt with stage one HPT and with high cvs
risk factors or in stage 2 HPT
Most anti hpt drugs would have reached its max bp reduction effect by 4 weeks.
Start with half the max dose and at 4 weeks if not controlled increase to maximal
dose.
By 8 weeks hence the max bp lowering effect by any single agent is known.
But any single agent can only reduce systolic bp by typically 16mmhg only.
Hence if baseline bp is high, combination drugs is needed from start as no single
drug will bring it to target.
Combination drugs acting synergistically will overcome the body's reflex response to
increase the bp back to baseline.
Fewer side effects is seen with combination treatment s lower doses is used in
combination. A single pill will also have better compliance.

1. Cvs risk doubles with each 20/10 mmhg increment
2. 2mmhg reduction in systolic Bp reduce 7% risk in ihd mortality and 10% reduction
in stroke mortality. Rewarding to treat HPT
3. Nocturnal Bp is predictive of MI and stroke
4. Resistant HPT is Bp remaining above goal in spite of 3 anti-HPT of difference
classes ( one of them should be a diuretic and all at optimum doses)
5. Proper way for Bp measurement: Pt seated 5 mins, arms at heart level, cuff
encircle 80% of arm! check both arms. Note that caffeine and tobacco increase the
Bp transiently

6. Use home and ambulatory Bp to sort out white coat HPT when suspected
7. Target value of home Bp monitoring is 5mmhg lower than clinic Bp (135/85)
8. Meds that increase Bp : NSAIDS, aspirins, cox 2, decongestant, EPO, cyclosporine,
licorice, amphetamine, cocaine, ecstasy, OCP
9. Recommended triple regimen : ACE or ARB, CCB and diuretics
10. If still resistant, kiv add spironolactone, or change to loop diuretics if CrCl<30
11. Diuretics therapy v useful as nuclear studies shows fluid overload in pt who does
not seem to have clinical overload.

Characterization of resistant hypertension: association between resistant
hypertension, aldosterone, and persistent intravascular volume expansion.
Gaddam KK, et al.
Journal: Arch Intern Med. 2008 Jun 9;168(11):1159-64.
doi:10.1001/archinte.168.11.1159.
BACKGROUND: Resistant hypertension is a common clinical problem and greatly
increases the risk of target organ damage.
METHODS: We evaluated the characteristics of 279 consecutive patients with
resistant hypertension (uncontrolled despite the use of 3 antihypertensive agents)
and 53 control subjects (with normotension or hypertension controlled by using
<or=2 antihypertensive medications). Participants were prospectively examined for
plasma aldosterone concentration, plasma renin activity, aldosterone to renin ratio,
brain-type natriuretic peptide, atrial natriuretic peptide, and 24-hour urinary
aldosterone (UAldo), cortisol, sodium, and potassium values while adhering to a
routine diet.
RESULTS: Plasma aldosterone (P < .001), aldosterone to renin ratio (P < .001), 24-
hour UAldo (P = .02), brain-type natriuretic peptide (P = .007), and atrial natriuretic
peptide (P = .001) values were higher and plasma renin activity (P = .02) and serum
potassium (P < .001) values were lower in patients with resistant hypertension vs
controls. Of patients with resistant hypertension, men had significantly higher
plasma aldosterone (P = .003), aldosterone to renin ratio (P = .02), 24-hour UAldo (P
< .001), and urinary cortisol (P < .001) values than women. In univariate linear
regression analysis, body mass index (P = .01), serum potassium (P < .001), urinary
cortisol (P < .001), urinary sodium (P = .02), and urinary potassium (P < .001) values

were correlated with 24-hour UAldo levels. Serum potassium (P = .001), urinary
potassium (P < .001), and urinary sodium (P = .03) levels were predictors of 24-hour
UAldo levels in multivariate modeling.
CONCLUSIONS: Aldosterone levels are higher and there is evidence of intravascular
volume expansion (higher brain-type and atrial natriuretic peptide levels) in patients
with resistant hypertension vs controls. These differences are most pronounced in
men. A significant correlation between 24-hour urinary aldosterone levels and
cortisol excretion suggests that a common stimulus, such as corticotropin, may
underlie the aldosterone excess in patients with resistant hypertension.
1. watch out for postural hypotension in elderly >20mmhg is significant
2. Below age 50 diastolic Bp is major predictor of ihd risk; while above age 60,
systolic Bp more significant
3. Pulse pressure is an independent risk factor in the elderly, it reflects
atherosclerosis of vessels. (>60 is not good)
4. Beta blockers did not fare too well in trials in stroke reduction as it only reduces
peripheral Bp but not the central aortic pressure significantly
5. Reduce Bp v gently in older patients to maintain cerebral perfusion pressure as
CPP is shifted to the right in the older population with hypertension
6. Aldosterone promote fibrosis in the heart and contribute to diastolic stiffness.
Aldosterone antagonist reduces this effect. RALES study shows benefit in pt with ccf
7. Diastolic dysfunction patients do well with ace, arb, diuretic and ccb. Beta
blockers are important to control the heart rate to maximise diastolic filling period
8. Theoretically not to lower diastolic Bp below 55-60 to maintain coronary
perfusion pressure, a J curve?
9. Always SCREEN pt's renal function when Bp starts to get uncontrolled and glucose
becomes better controlled!!
10. In renal disease patients, loop diuretics is preferred to thiazides if creatinine
>200
11. ARB might be slightly better than ace-I in proteinuria. Otherwise ACEi has an
upper hand.
12. Careful to start ace or arb in child bearing age women as ace and arb is
Contraindicated in pregnancy

13. Pt who became HPT after taking OCP, have to be monitor over the yrs as they
have higher risk for hypertension
14. Peptic ulcer disease- CCB not a good drug as increases risk of GI bleed
15. Avoid beta blockers in major depression
16. In the event of an ischemia stroke, only lower Bp if Bp >220/120. This is To
maintain perfusion to the penumbra.
But if there is presence of end organ damage like AMI, CHF or aortic dissection, only
lower Bp by no more than 10-20% in 24 hours
J curve
A Continuous relationship with stroke and chd is seen with rising bp.
Prevention is with bp lowering.
HPT is an ARTIFICIAL construct as the relationship is CONTINUOUS.
BENEFIT IS INDEPENDENT OF THE STARTING BP AND PROPORTIONAL TO THE
QUANTUM OF REDUCTION.
But how low do we go? In health bp at night is LOW and can be VERY low!
IN CHD there appears to be a J curve in contrast to healthy people but the data is
epidemiological and small. The J curve seen may NOT be due to the low diastolic bp
but to other factors
No nadir is seen with systolic bp.
Controlling bp well is more important than worries about J curves

Prof: Yin Ling, tell me your thoughts about the young hypertensive.
Firstly who is The Young Hypertensive?
43% of those more than 30yrs old are hypertensive in Msia.
Less than 40 yrs old is Young HPT.
Hence a sig number of our patients are Young HPT
1. Ensure correct diagnosis
2. 2. Teenagers with hpt.. think of Renal causes esp if symptomatic. GN is most
likely Or reflux uropathy occasionally.

Coarctation occasionally seen in Asymptomatic patients. MRI angio to


diagnose.

3. At Older Middle ages 20 to 40 yrs think of Renal Artery Stenosis esp if renal
function deteriorate with treatment. MRI angio help diagnose.
Obviously symptomatic patients think of Hyperthyroidism with high systolic BP.
VAGUE symptoms think of Hypothyroid with high diastolic BP.
For suspected Conn's we need to screen Aldosterone levels. Be aware that
hypertensive drugs like beta blockers and methy dopa, Dihydropyridine ccbs and
diuretics can affect the measured levels. They have to be stopped before
measurement.
If obese pls think of Sleep apnoea.
If eating Supplements or TCM, think of exogenous steroids, licorice.
Phaechromocytoma is rare. 24h urine cathecholamines for screening.

A recent trail result gave us new insights:
People with systolic blood pressure of 90 to 114 mm Hg and diastolic blood pressure
of 60 to 74 mm Hg had the lowest risk for cardiovascular disease in each age group.
(Lets keep our BPs low, be as slim as possible).
There was no evidence of a J-shaped increased risk for cardiovascular disease at
lower blood pressures. (Cheers!! One less worry!)
The strongest associations with high systolic blood pressure were for intracerebral
hemorrhage (hazard ratio [HR], 1.44; 95% confidence interval [CI], 1.32 - 1.58),
subarachnoid hemorrhage (HR, 1.43; 95% CI, 1.25 - 1.63), and stable angina (HR,
1.41; 95% CI, 1.36 - 1.46). (as expected with our present understanding).
The weakest association with high systolic blood pressure was for AAA (HR, 1.08;
95% CI, 1.00 - 1.17). (Hmmmmm, thats new).
Elevated systolic blood pressure had a greater effect on angina, MI, and PAD than
did diastolic blood pressure.
However, elevated diastolic blood pressure had a greater effect on AAA than did
elevated systolic pressure. (Thats new!)

Pulse pressure was inversely associated with AAA (HR per 10 mm Hg, 0.91; 95% CI,
0.86 - 0.98). (OK gota remember that!)
Pulse pressure was directly and most strongly associated with PAD (HR, 1.23; 95% CI,
1.20 - 1.27).
Lifetime risk of overall cardiovascular disease at age 30 years was 63.3% (95% CI,
62.9% - 63.8%) in people with hypertension (blood pressure 140/90 mm Hg or
treatment with blood pressurelowering drugs) compared with 46.1% (95% CI,
45.5% - 46.8%) for those with normal blood pressure.
People with hypertension developed cardiovascular disease 5.0 years earlier (95%
CI, 4.8 - 5.2 years) than those with normal blood pressure. (Years of life lost!!)
For persons having hypertension from index age 30 years, stable and unstable
angina accounted for 43% of the cardiovascular diseasefree years of life lost. (Its
more than Cholesterol!).
For persons having hypertension from index age 80 years, heart failure and stable
angina each accounted for 19% of years of life lost.
The investigators concluded that their findings do not support conventional wisdom
that blood pressure has strong associations with all cardiovascular diseases across a
wide age range, and that diastolic and systolic associations are concordant.
They also note the substantial lifetime burden of hypertension despite modern
treatments, as well as the need for new blood pressurelowering strategies.
Lancet. 2014;383:1861, 1866-1868, 1899-1911, 1912-1919. Rapsomaniki Abstract
Falaschetti Abstract Commentary Editorial

19) on HISTORY Taking
Dear Yin Ling,

the history taking skills are best learnt from repeatedly talking to patients with
different complains and forming well trodden thinking paths in the mind as we listen
and enquire. I try so hard every year to teach this to my students and sometimes it is
exhausting.

A good history is truly the Conductor of the orchestra of diagnosis, the many
physical signs and investigations the various players of varied instruments.
Sometimes it is a humble Quartet with a few signs and minimal investigations to

reach a diagnosis but still nevertheless beautiful like the '4 seasons'.

Sometimes it requires a full orchestra, with many complicated signs and a long list of
high tech scans. Nevertheless it still requires the Conductor to DIRECT it all or the
many musicians will be like headless chickens running about in no particular
direction. Many of the innocent naive histories and Phy Examinations presented by
my year3s are like that; a history that has NO thinking path and completely random
plus a Phy Examination like a wild man shooting in the dark hoping he will hit
something! Even their investigations do not follow any thinking sequence for the
Conductor of the History has gone schizophrenic!!

For my kiddies out there pls read this and understand well. The History when well
taken is a Master Conductor like Herbert von Karajan, Sir Georg Solti, Leonard
Bernstein, who directs his Orchestra with finesse, NOT A SINGLE INSTRUMENT OUT
OF TUNE or TIME!

The History when poorly taken leads to a whole sequence of laughable events which
may have serious consequences; Res Ipsa Loquitur!!

20) on ANGINA
Dear YL,
Why is pain of heart attack perceived in LEFT arm as well as in thorax?
Chee Yong Chuan : William Heberden M.D was the first to describe angina pectoris
and subsequently presented his findings to the Royal College of Physician. It was a
pleasure reading the original description of angina, and many of his astute
observations still hold true until today.

Prof : It is impt to realise that angina does not have a uniformly bad prognosis, that
the perception of the degree of pain does not correlate well with pathology.
The young student might think that he she that tells of minimal pain on the scale
could be dismissed as minor but this is certainly not so as described above. But a
SEVERE PAIN SO BAD THAT THE PATIENT GRASPS YOUR ARM AND NOT LET GO
would make one consider a dissection highly likely.

CYC : The visceral afferent fibers travel within the sympathetic cardiac nerves to the
sympathetic trunk on their way to the spinal cord, which they enter through dorsal
roots at levels T1-T5. These visceral afferents are thus in the same dorsal roots as
the somatic afferents returning from the medial border of the left upper limb and
left side of the chest wall (look at any standard dermatome chart). For reasons not
well-understood, the visceral pain (originating in the heart) is referred to the
cutaneous region innervated by nerve fibers that enter the spinal cord at the same
levels.
Prof : Dear YL,
-The Afferent 'pain' sensation FROM the heart is carried to spinal cord via
SYMPATHETIC nerves. That is why in conditions with autonomic neuropathy, there
may be NO pain perceived at all.
- These axons enter spinal cord at T1-T4 (maybe T5) on the left side.
-The Pain of angina is referred to left side of the chest and arm (supplied by T1
nerve) because these areas send sensory impulses to same segments of spinal cord
that receive cardiac afferent fibers. Pls note that there is much variation in the area
of pain.

Prof : Which is the Most common site of coronary artery block?


Which is the 2nd most common site of coronary artery block?
CYC : The major vessels of coronary circulation are the right coronary artery(RCA)
and the left main coronary artery(LMCA). The LMCA branches into the left anterior
descending artery(LAD) and left circumflex artery(Lcx). The RCA on the other hand
branches into the marginal branch and posterior inter ventricular branch. A quick
walk in the CCU/ED will reveal that anterior and inferior STEMIs are by far the most
common! (Makes sense as they are the major source of blood supply to the
myocardium). STEMI involving the high lateral wall, isolated posterior STEMI, left
main occlusion are not so common! Referring to the GUSTO-1 trial that recruited
almost 41 thousand patients with STEMI, anterior and inferior STEMI constitute 97%
of all STEMI

Prof : Well done. A true son of Aesculapius. Now you realise why the ECG changes of
Inferior MI and V1 ST abnormalities, the concept of RV infarcts , and Anterior MI
changes with possible Wellens preceding if high is SO much emphasized in my
sessions with you all.
Unrecognized myocardial infarction in patients aged 55 and older is not uncommon.
A high incidence of unrecognized myocardial infarction is due to atypical clinical
presentation of the infarction with increasing age.

Such an atypical presentation includes the absence of chest pain and the presence
of non-chest pain, particularly localized in the neck, back, jaw, or head, followed by
non-pain symptoms such as weakness, sweating, nausea, dyspnoea, or cough.

Women are a subgroup with a greater likelihood of atypical presentation.
The symptomatology of myocardial infarction, including both pain and non-pain
symptoms, may be affected by risk factors, such as smoking, hypertension, diabetes,
and hypercholesterolaemia.

There is also a lower frequency of chest pain among those evolving non-Q than
among those evolving Q-wave infarction.
Finally, isolated infarctions of inferior or lateral site more often have atypical
presentation compared with anterior infarctions.



Prof : Dear YL
what is the SHOULDER-HAND SYNDROME in relation to CHD?
YL: "SHS is a complex regional pain syndrome causing a painful UL due to the
disturbance of the sympathetic nerve supply,(reflex stimulation) commonly seen
after hemiplegia / MI. clinically patient will have painful shoulder with limited
motion with swelling, colour changes and stiffness.elbow is rarely involve. there will
be colour changes of the hands, followed by flexion deformity,and trophic changes
of the skin"

Prof: Dear yin ling,
Tell me the importance of lead avR in ECG interpretation?

Yin Ling: aVR lead is the only lead looking into the chamber of the heart. A normal
avR will usually not show any STE.
If there is ST elevation in aVR and widespread ST depression, it is suggestive of left
main stem occlusion or a high LAD occlusion. If there is no ST elevation in aVR, we
can almost certainly exclude LMCA.

Hence, in the setting of a patient having SYMPTOMS suggestive of ACS and you see
STE in aVR in the presence of other signs of Ischemia, you are dealing with with 1 of
three things : LMCA Occlusion, Proximal LAD Occlusion or Triple Vessel disease.

Dr Kianseng used to teach that
STE in aVR + STE in aVL = LMCA occlusion
STE in aVR + STE in V1 = LMCA or Proximal LAD Occlusion
STE in aVR > STE in V1 = LMCA Occlusion

When there is STE in aVR, KIV avoid Aspirin and Clopidogrel because it indicates
LMCA Occlusion and there is the probability he will need CABG.
In acute pericarditis, avR will be the only lead to be allowed to have ST depression.
Finding any ST Depression in any other leads, it is UNLIKELY acute pericarditis. In
acute pericarditis, there may also be PR segment elevation in aVR lead as well.
Other roles of avR - in the hospital setting, seeing an UPRIGHT avR tells me I amy be
looking at a wrongly placed limb lead in the ECG.

in the setting of MCQs, an upright avR with tachycardia in the correct clinical setting
is 'TCA toxicity'.

21) on LIVER DISEASE
Dear YL,
A 41-years-old man has a history of drinking 1 to 2 liters of toddy per day for the
past 20 years. Life is hard in the estates.

He had numerous episodes of nausea and vomiting in the past 5 years. He now has
prolonged vomiting, followed by massive hematemesis.

On physical examination his vital signs are:

T 36.9C, P 110/min, RR 26/min, and BP 80/40 mm Hg lying down.



What does this basic info tell you?
What is the pathophysiology behind this?
What will you immediately do?

Pulse is regular and no murmurs noted and his lungs are clear to auscultation. There
is no abdominal tenderness or distension and bowel sounds are present.

PR showed normal stools

What is the most likely diagnosis?
Prof : In liver disease there is Decreased activation of vitamin K-dependent enzymes:
Vitamin K is fat-soluble requiring bile secretion (for fat emulsification in the
intestine) for absorption.

Cholestatic liver disease can result in a lack of vitamin K, with decreased production
of vitamin K-dependent enzymes (factors II, FVII, X and X) and a vitamin K-
responsive coagulopathy. Remember also that important inhibitors of the
coagulation cascade, Protein C and protein S, are also vitamin-K dependent.

Production of abnormal factors: Abnormal fibrinogen molecules (dysfibrinogenemia)
is a feature of some liver diseases, such as hepatomas and acute and chronic liver
disease. The abnormal fibrinogen cannot form fibrin or cannot polymerize, resulting
in inadequate clot formation and hemorrhage
A therapeutic trial of Vit K is often given after blood is taken for Coagulation profile.
But there are no randomised clinical trials on the benefits or harms of vitamin K for
upper gastrointestinal bleeding in patients with liver diseases. There is no evidence
to recommend or refute the use of vitamin K for upper gastrointestinal bleeding in
patients with liver disease.
Dr Hu : Most emergency physicians are less aware of Wernicke encephalopathy,
and always misdiagnosis as posterior circulation stroke.

Wernicke encephalopathy, the consequence of thiamine deficiency, is characterized
by disorders of ocular motility, gait ataxia, and disturbances of consciousness. The
same triad of signs also encountered commonly in occlusion of the rostral basilar
artery.

22 ) ON BETA BLOCKERS
Dear Yin LING,

A 75 year old portly female has been on follow up for years with mild HPT and DM
controlled with oral medication. She has dyspnoea on exertion and left ventricular
systolic dysfunction is noted on ECHO. Her angio showed mild non critical lesions.
She was progressively dyspnoeic on climbing stairs but not at rest. Her renal
function is normal.

The patient is already on Ramipril and Furosemide.
Which drug/s may improve the patients symptoms and prognosis?

Amiodarone
digoxin
diltiazem
Bisoprolol
Isosorbide dinitrate
Vasteral
Coenzyme Q10
YL : 1) Beta blocker in heart failure. The sympathetic pathway and the
neuroendocrine pathway are the two culprits in the clinical course of heart failure.
The neuroendocrine pathway can be halt by an ACE- I and beta blockers can reduce
the sympathetic activation on the heart and slows down the progression of heart
failure. Major trials are MERIT-HF and CIBIS-II trial. The patients will often feel worse
before they feel better. Uptitrated during every follow up is important as long as
patient is able to tolerate.

2) beta blockers are still used in rate control for atrial fibrillation/SVT. It can also
stabilise a prolonged QT preventing Torsades.

3) Propanolol commonly used as prophylaxis for esophageal varices.

Prof : Dear Yin Ling,
besides what you had mentioned above, pls also remember
Anxiety
Glaucoma
Migraine
Thyrotoxicosis and its associated Periodic Hypokalaemic paralysis


Prof: The cornerstone of Rx for heart failure is ACEI OR ARB, and Beta blockers.
Diuretics removes symptoms and once that is achieved the above is THE Rx as
diuretics merely relieves symptoms without altering the pathophysiology.
ARB is used when ACEI is not tolerated, ACEI is first choice.
But Which Shoud we start first ? No difference between starting beta blockers or
ACEI first
We need to uptitrate ACEI OR ARB TO HIGH DOSES to achieve tissue blockade.
Beta blocker Dosage is not clearly correlated with mortality but reduction in heart
rate is. We aim to use target recommended doses. Only bisoprolol, cardivelol and
metoprolol can be used.
We need to start ACEI OR ARB AND BETA BLOCKERS early in THE course of
treatment for the best outcome.
DUAL BLOCKADE with ACEI AND ARB has many adverse effects on renal function,
hyperkalaemia and BP. It is not used.
Mineralcorticoid antagonists added to ACEI OR ARB is better.
In THE elderly be careful with ACEI or ARB because of renal impairment. A reduced
dose is advised on starting and careful follow up of renal function is needed.
With aldosterone be careful of hyperkalaemia esp in the elderly.
23) ON PALPITATIONS
Dear Yin ling,
A common clinical problem that we see in clinics is the young person presenting
with complain of 'palpitations a few days ago' .

An 18 year old woman had repeated episodes of breathlessness and palpitations, no


giddiness, but lasting about 20 minutes and resolving gradually.

Commonly on examination there are no abnormal physical signs. Nothing!
No click murmur, no long fingers. No vitiligo, no goitre.

How do you think out as to the most likely cause of these features?

Is it possible drug abuse by young spoiled adults?

What about panic disorders?

Hpw and when do we suspect paroxysmal supraventricular tachycardia?

Or it possible that a personality disorder is the cause?

And how about thyrotoxicosis sans goitre?
YL : yes palpitations is a common complaint we often see during our daily work and
we also often hear of our young friends having appts for a Holter due to recurrent
palpitations too.

Click murmur will suggest a Mitral valve prolapse, also know as Barlow's disease,
occurs more in young female in which the there's doming/ prolapse of usually the
posterior cusp of the MV back into the atrium during systole. the tensing of the
mitral valve during this cycle caused the Click! followed by a mid or late systolic
murmur. atrial fibrillation or other SVT can occur due to this- causing the complaint
of palpitations the is uusally self limiting.

Long fingers will suggest Marfan's syndrome, which is also associated with MVP. AR
rarely causes arrhythmias.

Vitiligo is due to an autoimmune process, another autoimmune process i can think
of that causes palpitations is graves disease, common in the young. they would
present with or without a goiter as well. thyroid antibodies, T4 and TSH are
important investigations as palpitations cannot be cure without taking care of the
thyrotoxic state.

Drug abuse is common; students taking amphetamines, caffeinated products to stay
awake to study, energy drinks saturated with glucose and caffeine, sports or health

product which promises vitality, thyroxine tablets taken by young girls for weight
loss...v common, causing thyrotoxic sans goiter i supposed?
PROF : Drug abuse can cause palpitations but there will be other symptoms. Young
girls pop amphetamine like drugs to lose weight and palpitations is common but
there is anorexia, insomnia as well. Look at the IC picture. Any mark change? Drugs
of abuse similarly has other features eg dehydration is common. Speak to parents.

Personality disorders tend to be chronic like me la and will not have a short history.
Many aspects of life will be affected by personality disorders.

Hyperthyroidism in young people typically have many features unlike the elderly
where AF may be the only finding. Weight loss, increased appetite, menstrual
abnormalities will be expected.

SVT has sudden starts and stops. That's not seen here.

Panic disorder is likely and this would be the working diagnosis while we exclude the
other possibilties as noted.

24) ON CLUBBING
Dear Yin Ling,

Pls know EVERYTHING about clubbing before your exams.

Finger clubbing may occur, without evident underlying disease, as an idiopathic form
or as a Mendelian dominant trait. Digital clubbing may be symmetric bilaterally, or it
may be unilateral or involve only a single digit.

The specific pathophysiologic mechanism of digital clubbing remains unknown!! The
common factor in most types of clubbing is distal digital vasodilation, which results
in increased blood flow to the distal portion of the digits.
Serric Suthesh: Currently the most accepted HYPOTHESIS is that it is caused by
Platelet derived growth factor; megakaryocytes travel to the lungs to fragment and
become platelets, if this process doesnt occur, they get trapped in the peripheral
vessels, secrete PDGF and cause proliferation of cells and fibroblast leading to
clubbing,
This hypothesis actually helps explain why cyanotic heart disease and lung disease

which disrupts normal pulmonary circulation can lead to this,


In Patients with lung cancer however they found Vascular endothelium growth
factor that is involved with hypertrophic pulmonary osteoatropathy as well.
Alterations in size and configuration of the clubbed digit result from changes in the
nail bed,
1. beginning with increased interstitial edema early in the process. Look for nailfold
Bogginess.

2. As clubbing progresses, the volume of the terminal portion of the digit may
increase because of an increase in the vascular connective tissue and change in
quality of the vascular connective tissue, this leads to Schamroth's sign

3. and subsequently an increase in the girth of the finger at the nailbed till the cross
sectional measurement is now more than that at the level of the DIP joint.

4. Finally the classic Drumstick or Parrot beak appearance is seen.
(Occasionally there are spurs of bone on the terminal phalanx.)

Clubbing usually develops first in the thumb and forefinger, and occurs in the other
fingers later. It may be difficult to recognize unless the student observes the hands
(and feet) carefully. Two signs are characteristic of early clubbing: the "floating nail"
sign and the "profile" sign.

In Grade 1 the nail feels as if it is floating on a cushion. I asked you all about the
angle seen from the side many times; with clubbing, proliferation of tissue under the
nail plate causes this angle to increase to more than 160 degrees. In fact, the angle

may be entirely lost and the nail plate and skin lie in a straight line (180-degree). As
clubbing progresses, the angle exceeds 180 degrees. Because this can be easily
detected and precisely defined, the student should rely on this sign when uncertain
if a patient's fingers are truly clubbed. We may debate over how Boggy is Boggy!!!
Han Sheng: MNEMONIC for causes of CLUBBING:

C.L.U.B.B.I.N.G, with the L having an extension of A.B.C.D.E.F

C - Cyanotic heart disease
L - Lung disease

Abscess
Bronchiectasis
Cystic Fibrosis/ carcinoma
Dont say COPD
Empyema
Fibrosis

U - Ulcerative Collitis + Inflammatory Bowel Disease(Crohn's Disease)
B - Biliatry cirrhosis
B - Birth Defects
I - Infective Endocarditis
N - Neoplasm(eg. Lung cancer or mesothelioma)
G - Gastrointestinal malabsorption syndrome(Coeliac disease)

Acropachy is an alternative term for clubbing. Acquired clubbing is often reversible
when the associated condition is treated successfully.


25) AUSCULTATORY PERCUSSION
Dear Yin Ling,
Tell me what do you know about Auscultatory Percussion on phy exam?
Chest and abdominal examination is greatly clarified once you master this
technique.

PROF : Auscultatory percussion is a method of physical examination which consists
in tapping lightly the manubrium sterni with the distal phalanx of the middle finger

while listening over the chest wall posteriorly with a stethoscope; a decrease in
sound intensity is usually attributed to lung abnormalities. The basic principle is that
Auscultation performed at the same time that percussion is made, combining both
clinical methods.

Auscultatory percussion is a sensitive and specific way to detect pleural effusion,
even when fluid amounts are small. It is useful in detecting the upper level of the
effusion before a tap.
It can also be used in localising the edges of the abdominal organs.

Interestingly it can be used in detecting fractures of the shaft of the femur,
humerus, and clavicle; telling the presence of a complete fracture, the relative
position of the fragments, and, during the postoperative course, the development of
bony union. The stethoscope bell and the percussing finger is applied over bony
prominences on either side of the fracture and the sound so elicited compared with
that produced by the same procedure on the normal side. Sound alteration
constitutes the criterion of the test. Pitch and quality changes result from free
vibration of the separate fragments and, accordingly, signify complete fracture or
incomplete union. Appreciable diminution in sound intensity indicates poor
conduction and reflects absence of end-to-end contact.


Pls see attached paper in JAMA Aug2 1941

26) ON SHOCK
Dear YL,

I tried very hard to teach you all today regarding the pathophysiology of
Hypovolemic shock and recognising it early. Impending shock is characterized by a
raised heart rate caused by catecholamine release in an attempt to maintain cardiac
output when stroke volume decreases due to diminished venous return.

Remember the rapid BUT thready Pulse!! Looking and feeling the hands tell
volumes!
C olour
C appilary return
T emperature
V olume of pulse
R ate of pulse
U rine output
Everyone of these signs tells of compensatory mechanisms taking place!
The Skin changes are there for all to see if only you observe.
There are signs of decreased skin perfusion in early shock due to peripheral
vasoconstriction. Just hold and feel the hands; a cold clammy hand is DANGER
manifested by a prolonged capillary refill (>2 seconds), cool mottled extremities and
pallor.

(Bradycardia is however a preterminal sign caused by hypoxia and acidosis. That is
near end game and too late! We see this in patients about to die. )

Hypotension (systolic blood pressure less than 80 or 90 mm Hg) is a very late sign of
shock. The body will do its best to compensate to ensure that cerebral and cardiac
perfusion is maintained.

A reduction in stroke volume may not cause a decrease in blood pressure if there is
an adequate increase in systemic vascular resistance and heart rate that can
compensate for the blood loss. In early shock and in spite of having a normal blood
pressure, organ and tissue perfusion may be compromised.

As shock advances, the compensatory mechanisms fail and profound hypotension
then occurs. That is when the nurse calls you!!

Pls always remember the formula for BP and you will understand the mechanism for
compensation. You must also remember the Shock Index to pick up the impending
shock early.

The clinical signs are easy to recall once you have the BP formula in your mind.

Background:
Compensatory Stage of Shock
During the compensatory stage of shock, the body tries to overcome the effects of
blood loss. Physiological, neural, hormonal, and biochemical reactions are used to
correct the imbalances. One of these mechanisms is hyperventilation to help
improve oxygen flow to the cells in order to neutralize the newly acidic conditions.

Another mechanism used is the catecholamine response to increase the bodys
heart rate to increase blood pressure.

A third mechanism is the renin-angiotensin response. During this response, a
hormone called vasopressin is released into the bloodstream to retain fluid and
triggers vasoconstriction.
Another hormone participating in this rescue mechanism is vasopressin, it is
secreted by the posterior pituitary gland in response to 2 factors, primarily
osmolarity and secondly blood pressure. The plummeting blood pressure is detected
by the pressure receptor e.g carotid sinus and aortic body which signal to the
hypothalamus' paraventricular nucleus and also the supraoptic nucleus to secrete
ADH/Vasopressin

There is 2 subtypes of vasopressin receptors: V1 is located at the smooth muscle
which cause vasoconstriction (hence increase the TPR and maintain the BP) and V2
at the kidney distal convoluted tubule and collecting tubule which increase
reabsoprtion of water. It was first named vasopressin because of its effect in
vasoconstriction.


Renal function will be sacrificed on the altar of cerebral perfusion!
Oliguria (urine output < 0.5 ml/kg/ h in adults ) reflects a decreased glomerular
filtration rate due to afferent arteriolar vasoconstriction in response to sympathetic

stimulation and renin-angiotensin system activation when volume depletion occurs.


Make sure that your patient has at least 1ml a min of urine output.
Urine output is absent in severe shock. ZERO!

The Brain is King. The brain, and heart have their blood supply protected at the
expense of decreased blood supply to other organs.

In early stages there are no changes in mental status. But as shock progresses, the
compensatory mechanisms fail and signs of decreased cerebral perfusion such as
agitation, confusion, drowsiness and coma appear.

Progressive Stage of Shock
If shock progresses, damage to the body is more severe and may even be
irreversible. Cellular function deteriorates, anaerobic metabolism leads to an
increase in metabolic acidosis, and the compensatory mechanisms are not able to
maintain the balance required to protect the organs.

Refractory Stage of Shock
If the cause of shock cannot be fixed, the body will inevitably enter the last stage of
shock. During this stage, the organs fail and lead to death. One of the most
important factors in recognizing the different stages of shock is to prevent
progression to this final stage.

27) ON CT SCANS
Dear Yin Ling,

CT scans have been a tremendous advance in our diagnostic kit.
It is however not entirely without risks.

Can you elaborate on the contraindications and risks involved?
I want all students to be aware of this. Pls see carefully the CXR equivalents. Then
we will be more mindful when ordering CTs. If a patient needs it, by all means do it!
But not if you are doing it for your own interest



Ask yourself honestly, does your patient require a CT and is the outcome beneficial
to him vs the long term risk? Then decide on the investigation
Another way of helping us understand is to see it from the equivalent of a year's
background radiation.
Radiation is measured in units called millisieverts (mSv). Different types of CT scan
involve different amounts of radiation:
CT scan of the head 1.4 mSv, which corresponds to seven-and-a-half months'
worth of background radiation
CT scan of the chest 6.6 mSv, or three years' worth of background radiation
CT scan of the whole body 10 mSv, which corresponds to four-and-a-half years'
worth of background radiation.

Newer machines with better software uses less radiation to obtain the same image,
how old is your hospitals CT scanner makes a difference
We are constantly exposed to radiation from a number of sources, including
radioactive materials in our environment, radon gas, and cosmic rays from outer
space. This is called background radiation and it varies across the countries.

The average man is exposed to about 3 mSv (millisieverts) of radiation from natural
sources over a year. Much of this exposure is from radon, a natural gas.

The earths atmosphere blocks some cosmic rays, thus living at a high altitude
increases a persons exposure a 10-hour airline flight increases cosmic ray
exposure by about 0.03 mSv.

Smoking a pack of cigarettes a day exposes the smoker to an extra 53 mSv per
year!!!
In summary, because radiation exposure from all sources ADDS up over a lifetime,
and radiation can increase cancer risk, imaging tests that use radiation should only
be done for a good reason.

In many cases, non radiation procedures like ultrasound or MRI may be used esp in
the younger patients. But if theres a reason that an x-ray or CT scan is the best Ix,
the patient must be helped more than the small dose of radiation can hurt.

Dr Hu : http://www.ncbi.nlm.nih.gov/pubmed/23048079
Mark Otto Baerlocher, MD, Murray Asch, MD, and Andy Myers, MDCM

Metformin is excreted by the kidneys


Metformin is used in type 2 diabetes mellitus to decrease the amount of glucose
produced by the liver and to increase the bodys response to insulin. In patients with
renal failure (acute or chronic), the renal clearance of metformin is decreased, and
there is an associated risk of lactic acidosis, which has a mortality rate of up to
50%. Some patients who receive intravenous contrast may experience a
deterioration of renal function (contrast-induced nephropathy). Although the points
in this article discuss the use of intravenous contrast, the same principles apply to
intra-arterial contrast.
Use of metformin is not a contraindication to intravenous contrast administration
Metformin in isolation is not considered a risk factor for contrast-induced
nephropathy, but particular attention must be paid to patients taking metformin
who are scheduled to undergo contrast-enhanced examination (e.g., enhanced
computed tomography [CT], angiography, venography). Many physicians are
particularly cautious in the case of elderly patients aged greater than 80 years.

For most patients, metformin should be stopped at the time of contrast


administration
There is some controversy about when to stop and restart metformin for patients
scheduled to undergo intravenous contrast-enhanced examinations. The guidelines
from the Canadian Association of Radiologists state that patients taking metformin
who have an estimated glomerular filtration rate (eGFR) of less than 60 mL/min
should stop taking metformin at the time of contrast administration. The European
Society of Urogenital Radiology advocates stopping metformin 48 hours before CT
for patients with an eGFR of less than 45 mL/min.
Restarting metformin depends on renal function and the volume of contrast used
Guidelines from the Canadian Association of Radiologists state that patients taking
metformin who have an eGFR of less than 60 mL/min should restart the drug no
sooner than 48 hours after contrast administration and only if renal function
remains stable (< 25% increase in creatinine above baseline). Patients with an eGFR
above 60 mL/min who receive a larger amount of intravenous contrast (> 100 mL;
e.g., CT of the abdomen or pelvis, CT angiography of the aorta or lower extremities)
should restart metformin no earlier than 48 hours after the procedure.
For small volumes of contrast, patients with normal renal function taking
metformin may not require any changes in care
If patients with normal renal function who are taking metformin receive less than
100 mL of intravenous contrast (e.g., enhanced CT of the brain), stopping metformin
and/or rechecking creatinine levels 48 hours after the procedure may be
unnecessary, because the risk of contrast-induced nephropathy in patients with
normal renal function is very low.

Are you aware that from the age of 40years, you lose 1% of renal function a year
even if you have no other risk factors!
Prof : Why are these risks different for males and females?
Different body tissues have different sensitivity to radiation. Skin and bone are not
very sensitive, but breast tissue, bone marrow, and the lining of the stomach and

intestine are sensitive to the effects of ionising radiation. On average, females are
more sensitive to the effects of ionising radiation than males.

Why are these risks different for children?

The principal risk for children exposed to X-rays is that at the time of exposure, their
growth means more cells are dividing, providing a greater risk of radiation disrupting
cell development. Children also have a longer life expectancy, giving a longer time
for the effects of any radiation damage, if present, to have an effect on long-term
health.

28) On TB
Dear Yin ling,

M. tuberculosis divides every 1520 hours, which is extremely slow compared to
other bacteria, which tend to have division times measured in minutes (Escherichia
coli can divide roughly every 20 minutes). It is a small bacillus that can withstand
weak disinfectants and can survive in a dry state for weeks. Its unusual cell wall, rich
in lipids (e.g., mycolic acid), is likely responsible for this resistance and is a key
virulence factor. When in the lungs, M. tuberculosis is taken up by alveolar
macrophages, but they are unable to digest the bacterium. Its cell wall prevents the
fusion of the phagosome with a lysosome. Smart right! That's why giant cells and
granulomas form to contain it.

AFB direct smears and culturing method is the standard way of screening TB. Direct
smears often fail when the bacterial load is low while culture will pick it up as it
allows the bacteria to multiply for identification.
Most common media(egg-based) used in Malaysia is the Lowenstein-Jensen media
(used before) and now the OGAWA (replacing LJ media)

M. tuberculosis is grown on a selective medium known as Lowenstein-Jensen
medium, which has traditionally been used for this purpose. However, this method
is quite slow, as this organism may require up to 48 weeks to grow, which delays
reporting of results. Lwenstein-Jensen (LJ) medium is most widely used for
tuberculosis culture. LJ medium containing glycerol favours the growth of M.
tuberculosis. A faster result can now be obtained using Middlebrook medium or
BACTEC.

BUT What is BACTEC?



It is a broth-based culture system designed to improve the speed and sensitivity of
detection. The BACTEC system is based upon Middlebrook 7H12 medium containing
14C labelled palmitic acid with a mixture of antibiotics (PANTA) to suppress other
bacterial growth. The addition of NAP (p-nitro-alpha-acetylamino-beta-
hydroxypropiophenone) in the medium suppresses growth of other M. tuberculosis
complex organisms such as M. bovis but does not differentiate other
nontuberculous mycobacteria from M. tuberculosis.

Bacterial growth is indicated by the detection of 14C released by M. tuberculosis as
it metabolizes the palmitic acid. In AFB smear-positive specimens, the BACTEC
system can detect M. tuberculosis in approximately eight days compared to
approximately 14 days for smear-negative, culture-positive specimens. However,
the high cost of the equipment and the need for radioactive material that requires
disposal exclude its use in most TB endemic communities.
Growth detected by the indicator system in drug-containing broth is interpreted as
resistance to the drug. The turnaround time for resistance testing is 10 days.

This method is very successful and reliable and that confirmatory results for M.
tuberculosis can be obtained within two weeks. However, the BACTEC machine is
very expensive to purchase and to operate.


What is OGAWA MEDIUM?

It is an egg based medium to culture M. tuberculosis. OGAWA MEDIUM
is cheaper than Lwenstein-Jensen because it is made without asparagine. LJ need a
lot of ingredients compared to Ogawa. This is why LJ media costs more. This method
is much cheaper than Bactec.

The ideal medium for isolation of tubercle bacilli should (a) be economical and
simple to prepare from readily available ingredients, (b) inhibit the growth of
contaminants, (c) support luxuriant growth of small numbers of bacilli and (d)
permit preliminary differentiation of isolates on the basis of colony morphology. The
Ogawa media is better than LJ in terms of cost, growth rate and contamination
rate.
BUT Speed is King with A premium to pay

The mean detection time for M. tuberculosis complex was 15 days with the
BACTEC method, and 26 days with the Ogawa method.
Dr Rama: What is TB gold?
Prof: The QuantiFERON-TB Gold In-Tube test is a measure of cell-mediated immune
response to antigens simulating the mycobacterial proteins. Individuals infected
with Mycobacterium tuberculosis complex organisms including Mycobacterium
tuberculosis, Mycobacterium bovis, Mycobacterium africanum,Mycobacterium
microti, and Mycobacterium canetti usually have lymphocytes in their blood that
recognize these specific antigens. The recognition process involves the generation
and secretion of the cytokine, interferon-gamma. The detection and quantification
of IFN-gamma by enzyme-linked immunosorbent assay (ELISA) is used to identify in
vitro responses to TB antigens that are associated with Mycobacterium tuberculosis
complex infection. The ESAT-6, CFP-10, and TB7.7 antigens are absent from the
Mycobacterium bovis BCG strains and from most nontuberculous mycobacteria with
the exception of Mycobacterium kanasii, Mycobacterium szulgai, and
Mycobacterium marinum. Numerous studies have demonstrated that ESAT-6, CFP-
10, and TB7.7 stimulate IFN-gamma responses in T cells from individuals infected
with Mycobacterium tuberculosis but usually not from uninfected or BCG-vaccinated
persons without disease or risk for Latent TB.
Kong Wah Ng: QFT-G can be regarded as a functional test because it measures the
host's response to M. tuberculosis whereas PCR amplifies the presence of M
tuberculosis DNA but does not differentiate between life or dead bacteria - much
like the acid-fast stain, albeit much much more sensitive and specific

29) ON STROKE
Dear Yin ling,
Why is it that we refer to 7th and 12th CNs as either UMN or LMN but not when we
speak of the other CNs?

Almost all of the cranial nerves receive bilateral innervation from the fibers of the
pyramidal tract. This means that both the left and right members of a pair of cranial
nerves are innervated by the motor strip areas of both the left and right
hemispheres. This redundancy is a safety mechanism. If there is a unilateral lesion
on the pyramidal tract, both sides of body areas connected to cranial nerves will
continue to receive motor messages from the cortex. The message for movement
may not be quite as strong as it was previously but paralysis will not occur.


The two exceptions to this pattern are the portion of 12th CN that provides
innervation for tongue protrusion and the part of 7th CN that innervates the
muscles of the lower face. These only receive contralateral innervation from the
pyramidal tract. This means that they get information only from fibers on the
opposite side of the brain. Therefore, a unilateral upper motor neuron lesion could
cause a unilateral facial droop or problems with tongue protrusion on the opposite
side of the body. For example, a lesion on the left pyramidal tract fibers may cause
the right side of the lower face to droop (R UMN 7th CN palsy) and lead to difficulty
in protruding the right side of the tongue (R UMN 12th CN palsy).


But even the 12th often has a small supply from the other side, hence the quick
recovery of the deviation in UMN lesions. While most people have bilateral
innervation of the 12th CN, this is NOT seen in others. Whats the bottom line then...
contralateral innervation is dominant while there is bilateral innervation in some.
Hence in a UMN lesion, the tongue deviation is seen early and then may improve
with time, in those with very significant bilateral innervation, in a UMN lesion the
deviation is minimal or not seen at all.

The other cranial nerves involved in speech and swallowing would continue to
function almost normally as both members of each pair of nuclei still receives
messages from the motor strip.

Because most cranial nerves receive bilateral innervation, lesions of the upper
motor neurons of the pyramidal tract must be bilateral in order to cause a serious
speech problem. Hence Pseudo Bulbar and Bulbar palsies.

On the other hand, unilateral lesions of the lower motor neurons may cause
paralysis. This occurs because the lower motor neurons are the final common
pathway for neural messages traveling to the muscles of the body. At the level of
the lower motor neurons, there is no alternative route which will allow messages
from the brain to reach the periphery. Muscles on the same side of the body as the
lesion will be affected. When there is eg a 3rd CN palsy it is understood to be a LMN
lesion.

Why I am CRANKY when Teaching my medical students!


I believe that this is because my body and mind is irreversibly damaged/distorted by
years of medical school training. I believe that the Brain surgery that was done on
my brain made it non compliant to any students not conforming to the uni culture of
last century.
Let me recall some highlights...

In year1, we had a Professor of Anatomy who was so fierce that grown men will cry
when she walked in. The 'name who must not be mentioned' will walk around with a
forcep in the dissection hall, randomly pick up a structure and BOOM "Why is this?
Origin? Insertion? Function??"
Most of us were mentally castrated by the time we finished anatomy and any
survivors would have been poisoned by the all overwhelming Formalin anyway! By
the way, we dissected with our BARE HANDS! That's the 20th century for you. Thank
goodness Dr Tan Too Moh my year one buddy was a surgical genius who faithfully
dissected every superficial nerve and tendon while I navigated from the dissection
manual. When Prof "Who must not be named" walked near, I leap into life and
buried my hands in the leathery body.
We had Prof Raman in Physiology who recurrently threatened to hang us on the tree
whenever we flunked his MCQs. Many of us would have died many times! But this
was one great teacher who taught us much3 Physiology. The Prof Raman physiology
quiz now held annually is in his memory.
And Prof Loke of Biochemistry made us feel so so small when he wrote on the
Blackboard with BOTH HANDS simultaneously and effortlessly the complicated
formulas of Biochemistry like you and I writing a nursery rhyme. OK we felt like
Cretins by 1st year! And I hated the DNA structures that they taught.... why oh why
did they torture us with that!
And each subject had its own Essay question examination, MCQ and practical
exam!!! 9 exam sessions!!
By some unknown miracle we survived year1 only to plunge into the arms of Prof
Chai in year2, his Microbiology in no way made his hands micro.. for he will without
hesitation pinch anyone of us squarely in the belly when we do not know the Stains,
Cultures and immunology of all his unseen friends. OUCH3! BTW this was before the
days of measurement of abdominal skin fold thickness, had the good Professor
known then, he would have collected enough data for his PhD.
Prof Prathap of Pathology was the gentleman of gentlemen, ever so calm, cool and
collected. OK he was probably the only man in medical school who did not scold us,
BLESS the man.
In Parasitology, we stirred shit in saturated saline like how your mummy would beat
flour to make a cake, the difference being that the cake will smell delicious while we
smell in a manner that gave reason for the medical faculty to be placed in the far
flung corner of the campus. It also explains why the beautiful girls from the Arts
faculty stayed put in their diagonally placed faculty.

Some of those parasites may well have entered our brains too for we also slept,
studied and ate in the very same 'multi-discipline' laboratories'.
Pharmacology was about Pharmacokinetics and Phamacodynamics; it made our
minds as contorted as the double helix. The poor cats from the Clinical Hostel
involuntarily gave their lives for us to understand "Therapeutic Index" and "Lethal
doses"! Aiyo, I still shudder when I think of the poor feline on that heated table
hooked up on monitors. We watched in horror as adrenaline was slowly infused into
its veins and the cardiac monitor went dit, dit, dit then a hundred times faster when
a technician whipped out a scalpel and opened up its chest in like 3 secs exposing a
fibrillating cat heart. OK another Psychological scar lasered into my brain!
Whoopie! We finished the Basic sciences and walked proudly into University
Hospital with our Brand new stethoscopes and tendon hammers. 3 years of
Clerkship followed BUT wait!! Clerks sit down and work, we STOOD for 3 years! Now
I understand why my knees hurt whenever I stand for long now.
Students nowadays do NOT understand the meaning of the word "TERROR", nope
you do not until you are selected by the drawing of straws to present the Long Case
to Prof Danaraj. Now that's pure terror. The good Professor's expectation of History
is so detailed that War and Peace would look like a short story in comparison. His
expectation of our physical examination is so complete that THE ENTIRE Tally's that
you all use today will seem inadequate in his eyes. THE PATIENT IS ALWAYS
COMPLETELY EXAMINED, a concept today's students simply fail to grasp to my
eternal irritation. And the tendon hammer will whisk about freely like a conductor's
baton, occasionally landing on our thighs as a reward for a really ignorant answer!
My stomach probably developed gastritis from the ward rounds and clinics of Prof
Florence Wang, it was interminable as she patiently spoke to EVERY patient in the
ward and clinic, examined every patient personally NO MATTER who did so
beforehand and then grilled the Houseman and students in the entourage. She
taught me that while it is important to treat the patient's physical illness, it is even
more important to treat the patient's mind and show him/her that you care. Treat
the Human being, not just the disease.
In surgery, the teachers constantly reminded me that my knowledge of anatomy is
as low as "the skin of a pig's belly"... aka NEXT TO NUTHIN! Ok thats one reason I
chose Internal Medicine.
In Psychiatry we had a lecturer who wore a BLUE shirt for all the years that we were
at Uni! And Prof Deva will ask and ask and ask "WHY". Patient cannot sleep sir.......

WHY?, Because he feel stressed sir..... WHY?, because his business has problems
sir.... WHY?..... et infinitum.
And in Paediatrics Prof Lam will literally JUMP up and Down on the spot whenever
we do something or say something "STUPID"!
"This child was apparently well until.".
"Just because the ^%*_+ Hutchison says that you start the presentation that way,
you mindlessly can ONLY do it that way!!!!" JUMP3!
And Prof Sinnathuray of OG will literally STAND at the entrance to catch all the
students who came late! Half my students today will be queuing at the Dean's office
today to explain if we follow the same methods! Now we have a sweet counsellor to
slowly talk sense into them.
So much for why your brain damaged tutor today FREAKS out when the students
come late, not know their basic sciences, or do a physical examination so ANAEMIC
that Pernicious Anaemia seems more pink! It is really because of the software fed
into me that refuses to move into the 21st century.


30) On Rheumatoid Factor
Dear Yin Ling,
in 39 days you will sit for your exams. I will expedite all that I can do to help you
revise. I will even answer my own questions!

RF is a very common test.

Rheumatoid factor is an immunoglobulin which can bind to other antibodies.


Rheumatoid factor is found in about 1-2% of healthy people. The incidence of
rheumatoid factor increases with age and about 20% of people over 65 years old
have an elevated rheumatoid factor. That is why it is so commonly seen as positive
in screening blood tests in the elderly.

High titers of rheumatoid factor are associated with more severe rheumatoid
arthritis. The factor also has been associated with a higher tendency to develop the
non-joint complications of the disease such as rheumatoid nodules and rheumatoid
lung disease.

Rheumatoid factor is present in about 80% of adults (but a much lower proportion
of children) with rheumatoid arthritis. It is also present in patients with other
connective tissue diseases such as systemic lupus erythematosus and Sjgren's
syndrome, and some with infections diseases including infectious hepatitis.

Test: One method mixes the patient's blood with tiny latex beads covered with
human antibodies (IgG). The latex beads clump or agglutinate if rheumatoid factor
(IgM RF) is present.

A rheumatoid factor titer more than 1:80 is indicative of rheumatoid arthritis but
may also occur in other conditions.
False positive results can occur when the blood is high in fats.

A negative test result for rheumatoid factor does not exclude the diagnosis of
rheumatoid arthritis.

The presence of this marker is not, however, needed to make the diagnosis of
rheumatoid arthritis. In fact 15-25 percent of all patients with rheumatoid arthritis
do not have rheumatoid factor in their serum. There is no conclusive laboratory test
which confirms the diagnosis of rheumatoid arthritis.

Patients are often times fearful, when their doctor tells them that a rheumatoid
factor (RF) was found on routine laboratory testing. Immediately, they assume that
they have developed rheumatoid arthritis (RA). This is simply not the case.

Anti-citrullinated protein antibody (ACPA) is a better tool for identifying
rheumatoid arthritis than rheumatoid factor levels
Now remember the old British car MG and you will remember its IgM against IgG!!!


Dr Hu : What is seronegative stand for ? What is seronegative arthritis ?
(Please take note, rheumatoid arthritis (RA) is not included as seronegative arthritis
even it can be seronegative.).

Rheumatoid factor (RF) & Antibodies against Cyclic Citrullinated Peptides (ACCP)
have less sensitivity in the diagnosis of early disease phase of RA.

At present, RA is diagnosed based on fulfillment of the classification criteria set by
the American College of Rheumatology (ACR), which have recently been revised by a
joint ACR and EULAR committee. In the revised criteria, objective serological testing
for the presence of 2 RA disease markers, rheumatoid factor (RF) and antibodies
directed against cyclic citrullinated peptides (ACCP), is included as an important
criterium.

However, according to recent meta-analyses, approximately one-third of established
RA patients are seronegative for these 2 diagnostically applied disease markers.
Moreover, the sensitivities of both markers for RA are reported to be even lower in
the diagnostically important early disease phase.

Extra-articular/systemic manifestation can appear in early stage of RA

Extra-articular manifestation usually develop in patients with long term and severe
RA, however, in some cases systemic manifestation such as interstitial pulmonary
diseases, pleuritis or pericarditis can appear in early stages of the disease.

RA also can has positive ANA/anti-double-stranded (anti-ds) DNA.

Antinuclear antibodies (ANA) and anti-double-stranded (anti-ds) DNA antibodies


may be present in patient with RA, although, the physicians have to be aware that
treating RA with anti-TNF monoclonal antibodies infliximab can increase serum
levels of ANA and anti-ds DNA.

CRP more specific than ESR in measure disease activity

Typically, acute phase reactants such as sedimentation rate (ESR) and reactive C
protein (CRP) are used to measure the disease activity. These are markers of the
systemic acute inflammatory response, however, CRP levels are more specific for RA
compared with the erythrocyte sedimentation rate (ESR) levels, since CRP are more
correlated with higher levels of pro inflammatory cytokines, such as interleukin
(IL)6 and TNF- and related with radiological progression.



31) on WATER AND ELECTROLYTES
Dear Yin Ling,
Every year I ask the students as to how much urine they produce in a day.... and
every year I am AMAZED that not a few have NO CLUE!

You know of course that Water and Na balance are closely interdependent.
Total body water (TBW) is about 60% of body weight (ranging from about 50% in
obese people to 70% in lean people; I keep telling auntie that it is ok for me to run in
the rain to the car as I am 70% water!).

Almost 2/3 of TBW is in the intracellular compartment (intracellular fluid); the


other 1/3 is extracellular (extracellular fluid). Normally, about 25% of the ECF is in
the intravascular compartment; the other 75% is interstitial fluid.

Total body water = 70 kg 0.60 = 42 L.

The major intracellular cation is K, with an average concentration of 140 mEq/L. (To
remember think of Na concentration in blood). The extracellular K concentration is
ONLY 3.5 to 5 mEq/L. Do realise the HUGE Gradient!

The major extracellular cation is Na, with an average concentration of 140 mEq/L
and an intracellular Na concentration of 12 mEq/L. The Na-K exchange pump is
super duper efficient to maintain this gradient.

Osmotic forces: The concentration of combined solutes in water is osmolarity (its in
simple words the amount of solute per L of solution la), which, in body fluids, is
similar to osmolality (amount of solute per kg of solution). Words words words... a
litre of fluid is about 1 kg ma!

Hence, Plasma osmolality can be estimated according to the formula

Plasma osmolality (mOsm/kg) = Na + Glucose + Urea

where serum Na is expressed in mEq/L and glucose and BUN are expressed in
mg/dL. OLD UNITS. These are the BIG BOYs ma.

Whose the Number 1 badminton player in the world? Chong Wei, everyone knows
right! Whose no 2? Lin Dan right! Now whose no 3?? Errrr Errrr. Only the big boys
are BIG, the rest cannot remember! Same in Blood! Remember the BIG BOYs! Your
blood when you bite your tongue is SALTY! Thats the Chong Wei! If you are diabetic
it may be sweet lorr. Thats Lin Dan and of course as Renal failure comes, Urea is a
BIG player now.

Osmolality of body fluids is normally between 275 and 290 mOsm/kg.

Na is the major determinant of serum osmolality.

An osmolar gap is present when measured osmolality exceeds estimated

osmolality by 10 mOsm/kg. When this happens, there is a DARK HORSE!!


Someone has crept in and upset the RANKING ORDER! WHO IS THAT you must ask!

It is caused by unmeasured osmotically active substances present in the plasma. The
most common are alcohols (ethanol, methanol, isopropanol, ethylene glycol),
mannitol and glycine.

From the formula you can see how Urea affects the osmolality in the ESRF patient.

Water crosses cell membranes freely from areas of low solute concentration to
areas of high solute concentration. Thus, osmolality tends to equalize across the
various body fluid compartments, resulting primarily from movement of water, not
solutes. Note that Solutes such as urea that freely diffuse across cell membranes
have little or no effect on water shifts (little or no osmotic activity), whereas solutes
that are restricted primarily to one fluid compartment, such as Na and K, have the
greatest osmotic activity.

Water intake and excretion: The average daily fluid intake is about 2.5 L. ( Note the
IVD regimes we use when a patient is "Nil by mouth! How much fluid do we give??)

In the normal resting state, input of water through ingested fluids is approximately
1200 ml/day, from ingested foods 1000 ml/day and from metabolism 300 ml/day,
totaling 2500cc/day. Note the magic number!

The amount needed to replace losses from the urine and other sources is about 1 to
1.5 L/day in healthy adults. (Most of you are smart to carry a 1000cc bottle!)
However, on a short-term basis, an average young adult with normal kidney
function may survive on as little as 200 mL of water each day to excrete the
nitrogenous and other wastes generated by cellular metabolism. The urine will be
very concentrated! More is needed in people with any loss of renal concentrating
capacity.

If we are to use urine output as a means of judging adequate hydration status, this
firstly assumes that the renal function is NORMAL. If we see at least 1-1.5cc of urine
output a minute, this is reassuring of adequate provision of fluids. At 1cc a minute,
we expect 60cc an hour and about 1500cc per day. This is about the average urine
output for an adult.

When we do not drink or is not on IVD, the kidneys will concentrate our urine as you

will notice in the urine that you pass on waking up; its concentrated after not
drinking for 7-8 hours of sleep.

Renal concentrating capacity is lost in
-The elderly
-Patients with diabetes insipidus, certain renal disorders, hypercalcemia, severe salt
restriction, chronic overhydration, or hyperkalemia
-People who ingest ethanol, phenytoin, lithium, or amphotericin B
-People with osmotic diuresis (eg, due to high-protein diets or hyperglycemia)

Other obligatory water losses are mostly insensible losses from the lungs (about
500cc) and skin (depending on ambient temperature and humidity, it varies from
300 onwards), averaging about 650 to 850 mL/day in a 70-kg adult. Sweat losses can
be significant during environmental heat exposure (like in the last few days; notice
my water bottle empties fast to replace my sweat in the wards) or excessive
exercise. With fever, another 50 to 75 mL/day may be lost for each degree C of
temperature elevation above normal.

Assuming a loss of about 800cc from breathing and sweating PLUS whatever little
urine is passed by the ESRF patient on HD, that is his allowed water intake per day.

GI losses are usually negligible, except when vomiting, diarrhoea, or both occur.

Water intake is regulated by thirst. Thirst is triggered by receptors in the
anterolateral hypothalamus that respond to increased serum osmolality (as little as
2%) or decreased body fluid volume. Rarely hypothalamic dysfunction decreases the
capacity for thirst.
Water excretion by the kidneys is regulated primarily by ADH (vasopressin). ADH is
released by the posterior pituitary and results in increased water reabsorption in the
distal nephron. ADH release is stimulated by any of the following:
Increased serum osmolality
Decreased blood volume
Decreased BP
Stress!! Remember this! A patient post surgery, septic or in my Long Case class or
worse BEDSIDE Class will have increased ADH output. (I cannot understand why
some students keep going to the toilet, better check your renal function!)

ADH release may be impaired by certain substances (eg, ethanol, phenytoin) and
central diabetes insipidus

Water intake decreases serum osmolality. Low serum osmolality inhibits ADH
secretion, allowing the kidneys to produce dilute urine.



32) ON BILIRUBIN METABOLISM
Dear Yin Ling,

when we see a patient with jaundice, the physiology of Bilirubin metabolism must
be at the back of our minds in order to think of differential diagnosis. This separates
medicine from quackery. Not all jaundice is due to liver diseases and liver diseases
are not always accompanied with jaundice!!

Bilirubin, a physiological product of RBC, is metabolized in the liver and excreted into
bile ducts; an appearance of jaundice means that there is a breakdown of balance of
bilirubin metabolism and the patient may have a problem in the liver, or RBC
production and destruction, or excretion of bilirubin. eg hemolytic diseases: Always
keep it in mind when managing a patient with jaundice.

Bilirubin is an end product of heme metabolism, coming mainly, 70 ~ 80 %, from
hemoglobin of senescent red blood cells; it splits to heme and globin, then further
split to iron and biliverdin, and the biliverdin converts to bilirubin.

Bilirubin combines with albumin in the blood stream, only separated just before
being uptaken into liver cells. The bilirubin in the hepatocytes conjugates with

glucuronic acid to become conjugated bilirubin, which is excreted to the biliary tract
and intestines and finally excreted.
The bilirubin from hemoglobin is free unconjugated bilirubin in the blood stream
and is not soluble in water. After being taken into hepatocytes, it is converted to
soluble conjugated form and excreted into bile ducts.

The bilirubin is divided into two types, direct reacting bilirubin and indirect reacting
bilirubin, according to its mode of reaction during the test process. It can be
recognized that direct reacting bilirubin is the conjugated bilirubin and the indirect
reacting bilirubin as unconjugated bilirubin.

Conjugated bilirubin is absorbed in the distal portion of the ileum after its
hydrolyzed and converted to URObilinogen by the intestinal pathogens.
About 15 ~ 20 % of the urobilinogen is reabsorbed from the intestine into portal
veins and finally 90% of them return to the liver and is re-excreted in the bile, the
entero-hepatic circulation of bilirubin. The remaining 10 % gets into the systemic
circulation and finally excreted in the urine through kidney. Thus urine urobilinogen
increases in hemolytic disease.

Hyperbilirubinemia -- jaundice occurs when the bilirubin balance between
production and excretion breaks down.

the possible causes of hyperbilirubinemia:
1. over production of bilirubin from hemolysis
2. the impairment in bilirubin uptake and conjugation in the liver,
3. impaired excretion from the liver cells or the liver
4. the unconjugated and conjugated bilirubin that is leaked into the blood stream
from damaged liver cells.

High-unconjugated-bilirubinemia

(1) Overproduction:
Normal liver can handle the amount of seven times of normal daily bilirubin
production.
When the production of bilirubin is increased due to hemolysis and and Ineffective
erythropoiesis beyond the ability of normal liver to handle, the serum indirect
bilirubin will increase and this is prehepatic jaundice. AST, ALT and Alk-P, that reflect
the damage of hepatocytes will remain normal and predominantly indirect bilirubin

is increased. Note that The conjugated bilirubin may increase slightly because of the
high turnover.

(2) Abnormality in uptake and conjugation:

Serum indirect bilirubin may increase when there is problems of uptake and
conjugation in the liver cells of bilirubin. This is non-hemolytic unconjugated
hyperbilirubinemia.
Crigler-Najjar syndrome (congenital non-hemolytic jaundice) is caused by the
deficiency of glucuronyl transferase. The symptoms will appear in the infant stage,
and there are two types, Type I is more severe than Type II, and may induce
kernicterus.

Gilbert's syndrome or idiopathic unconjugated hyperbilirubinemia is caused by the
similar mechanism as Crigler-Najjar syndrome, and only different in degree.

High-conjugated-bilirubinemia:

unconjugated-bilirubin conjugates with glucuronic acid to become conjugated-
bilirubin.
When transportation of conjugated-bilirubin is impaired in the liver during the
excretion process from liver cells or during passage from bile ductules, the condition
is called cholestatic jaundice.

(1) Intrahepatic causes of cholestasis:

The jaundice in drug-induced hepatitis and in pregnancy is intra-hepatic cholestasis.

Dubin-Johnson Syndrome and Rotor Syndrome are congenital causes of intrahepatic
cholestasis. The increase of serum bilirubin is mainly conjugated-bilirubin, and Rotor
syndrome is considered as a variant of Dubin-Johnson syndrome. Morphologically,
melanin pigments deposit in the liver cells are noted in Dubin-Johnson syndrome
but not in Rotor Syndrome.

Primary biliary cirrhosis shows obstruction of biliary ductules and inter-lobular bile
ductules.

Primary/secondary sclerosing cholangitis will induce hyper-conjugated-


bilirubinemia.

In hepatocyte diseases, i.e. acute and chronic liver diseases including cirrhosis, the
uptake, conjugation and excretion of bilirubin in the hepatocytes are impaired and
induce an intra-hepatic cholestasis. Therefore, the serum bilirubin elevation is a
mixed type.

(2) Extrahepatic cholestasis:

Stones, parasites, tumours in the biliary tract, biliary obstruction due to external
compression from Ca Pancreas will induce elevation in serum conjugated-bilirubin.

No conjugated bilirubin is present in normal urine. Only conjugated-bilirubin will
pass through renal glomeruli. Serum level of bilirubin does not parallel to the
amount of urinary bilirubin.

Urobilinogen- only a small part of urobilinogen absorbed from the intestinal tract is
excreted out of the body through the kidney, and most of the urobilinogen return to
the liver and are re-excreted to the intestinal tract.
The amount of urinary urobilinogen is affected by the amount of conjugated-
bilirubin in the biliary duct and also intestinal pathogens that convert bilirubin to
urobilinogen.
Urobilinogen is a colourless product of bilirubin reduction. This constitutes the
"enterohepatic urobilinogen cycle".
Increased amounts of bilirubin are formed in haemolysis, which generates increased
urobilinogen in the gut.
In liver disease (such as hepatitis), the intrahepatic urobilinogen cycle is inhibited
also increasing urobilinogen levels.

Urobilinogen is converted to the yellow pigmented urobilin apparent in urine.
The urobilinogen is reduced to stercobilinogen in the intestine and is then oxidized
to brown stercobilin, which gives the feces their characteristic color.

In biliary obstruction, below-normal amounts of conjugated bilirubin reach the
intestine for conversion to urobilinogen. With limited urobilinogen available for
reabsorption and excretion, the amount of urobilin found in the urine is low. High
amounts of the soluble conjugated bilirubin enter the circulation where they are

excreted via the kidneys. These mechanisms are responsible for the dark urine and
pale stools observed in biliary obstruction.

Low urine urobilinogen may result from complete obstructive jaundice or treatment
with broad-spectrum antibiotics, which destroy the intestinal bacterial flora.
(Obstruction of bilirubin passage into the gut or failure of urobilinogen production in
the gut.)



33) on LIVER ENZYMES
My dear yin ling,

ALT, an enzyme in liver cells, with lesser amounts in the kidneys, heart, and skeletal
muscles, and is a relatively specific indicator of acute liver cell damage. When such
damage occurs, ALT is released from the liver cells into the bloodstream, often
before jaundice appears, resulting in abnormally high serum levels that may not
return to normal for days or weeks.

In combination, ALT and AST are two of the most reliable markers of hepatocellular
injury or necrosis. Of the two, ALT is more specific for hepatic injury because it is

present mainly in the cytosol of the liver and in low concentrations elsewhere. AST
has cytosolic and mitochondrial forms and is present in tissues of the liver, heart,
skeletal muscle, kidneys, brain, pancreas, and lungs, and in white and red blood
cells.

Markers for high alcohol consumption are carbohydrate deficient transferrin (CDT),
gamma glutamyl transferase (GGT) and aspartate aminotransferase (AST). BUT Most
have fairly low sensitivities and specificities

An elevated serum AST in relation to serum ALT (alanine aminotransferase) is likely
an indicator that alcohol has induced liver damage. Thus, when AST/ALT ratio is
>1.5, this is considered as highly suggestive that alcohol is the cause of the patient's
liver pathology.

However, many patients who doubtless consume high amounts of alcohol and
indeed are alcohol-dependent and display elevated serum aminotransferase levels
do not show a high AST/ALT ratio. This suggests that additional factors lead to the
high AST/ALT ratio seen in some patients. One such factor may be the severity of
the liver disease.

The well-recognised high AST/ALT ratio in alcoholic liver disease is, in fact,
predominantly found in patients whose disease is advanced. Different, to some
extent possibly interrelated, reasons have been reported for the high AST/ALT ratio
in alcoholic liver disease:
i) a decreased hepatic ALT activity as healthy hepatocytes decrease;
ii) pyridoxal 5-phosphate depletion in the livers of alcoholics;
and
iii) mitochondrial damage leading to an increase in serum activity of mitochondrial
aspartate in patients with high alcohol consumption leading to high AST.

BUT Most patients with high alcohol consumption do not have an AST/ALT ratio
above 1.

Hence, remember that a high AST/ALT ratio is suggestive of advanced alcoholic
liver disease, not just alcoholism.

Liver DAMAGE is seen with increased AST/ALT ratio in patients. This has also been
associated with the development of cirrhosis in Nonalcoholic Steatohepatitis.
Furthermore, a high AST/ALT ratio in patients with increased serum

aminotransferases has been reported in chronic viral hepatitis, possibly due to the
same reasons.

ALT is present mainly in the cytosol, while AST in the mitochondrias. During events
of INFLAMMATION, as hepatocyte cell wall integrity breaks down, ALT will
increase much much more than AST, eg in viral hepatitis.

ALT testing helps detect and evaluate progress and treatment of acute hepatic
disease, especially hepatitis, and cirrhosis.

Very high ALT levels (up to 50 times normal) suggest viral or severe drug-induced
hepatitis, or other hepatic disease with extensive damage of liver cells. (AST levels
are also elevated but usually to a lesser degree.)

Moderate-to-high levels may indicate infectious mononucleosis, chronic hepatitis,
intrahepatic cholestasis or cholecystitis, early or improving acute viral hepatitis, or
severe hepatic congestion due to heart failure.

Slight-to-moderate elevations of ALT (usually with higher increases in AST levels)
may appear in any condition that produces acute hepatocellular (liver cell) injury,
such as active cirrhosis, and drug-induced or alcoholic hepatitis.

Marginal elevations occasionally occur in acute myocardial infarction (heart attack),
reflecting secondary hepatic congestion or the release of small amounts of ALT from
heart tissue.

However in the events of CELL DEATH, AST will predominantly increase as cell
death release the AST enzymes from the mitochondria eg in liver cirrhosis, and
myocardial infarct.

AST levels fluctuate in response to the extent of cellular necrosis and therefore may
be temporarily and minimally elevated early in the disease process, and extremely
elevated during the most acute phase. Depending on when the initial sample was
drawn, AST levels can rise- indicating increasing disease severity and tissue
damage- or fall- indicating disease resolution and tissue repair. Thus, the relative
change in AST values serves as a reliable monitoring mechanism.

May this little yet important concept helps in your future LFT interpretation.

34) ON REFLEXES
Dear Yin Ling,

The NORMAL Superficial reflexes

Superficial reflexes are motor responses to scraping the skin.
They are graded simply as present or absent and markedly asymmetrical responses
would be considered abnormal.

These reflexes are VERY different from the muscle stretch reflexes in that the
sensory signal has to not only reach the spinal cord, but also must ascend the cord
to reach the brain. The motor limb then has to descend the spinal cord to reach
the motor neurons . As can be seen from the description, this is a polysynaptic
reflex.

This can be abolished by lower motor neuron damage or destruction of the sensory
pathways from the skin that is stimulated. However, the utility of superficial reflexes
is that they are decreased or abolished by conditions that interrupt the pathways
between the brain and spinal cord (such as with spinal cord damage).

Classic examples of superficial reflexes include the abdominal reflex, the cremaster
reflex and the normal plantar response.

The abdominal reflex includes contraction of abdominal muscles that is stimulated
by scraping the skin superficially and rapidly along a dermatome towards the
umbilicus . This contraction can be seen as a brisk contraction of the abdominal
muscles with the umbilicus moving towards the stimuli.

The cremaster reflex is produced by scratching the skin of the medial thigh, which
should produce a brisk and brief elevation of the testis on that side.


The normal plantar response occurs when scratching the sole of the foot from the
heel along the lateral aspect of the sole and then across the ball of the foot to the

base of the great toe. This normally results in flexion of the great toe (a "down-going
toe") and, indeed, all of the toes.

The "anal wink" is a contraction of external anal sphincter when the skin near the
anal opening is scratched. This is often abolished in spinal cord damage (along with
other superficial reflexes ).

And Now the "Pathological reflexes"

The best known (and most important) of the so-called "pathological reflexes" is the
Babinski response (upgoing toe; extensor response). The full expression of this reflex
included extension of the great toe and fanning of the other toes . It is actually a
superficial reflex that is elicited in the same manner as the plantar response (i.e.,
scratching along the lateral aspect of the sole of the foot and then across the ball of
the foot toward the great toe). This is a primitive withdrawal type response that is
normal for the first few months of life and is suppressed by supraspinal activity
sometime before 6 months of age.

Damage to the descending tracts from the brain (either above the foramen magnum
or in the spinal cord) promotes a return of this primitive protective reflex, while at
the same time abolishing the normal plantar response. The appearance of this reflex
suggests the presence of an upper motor neuron lesion.

35) ON SLE
Dear yin ling,
This I hope will help you remember

O- Oral ulcer
R- malar Rash
D- Discoid rash
E- Exaggerated photosensitivity
R- Renal disorders (proteinuria, cellular casts)

H- Haematology disorders (haemolytic anemia, leukopenia, lymphopenia,
thrombocytopenia)
I- Immunological disorders (anti-DNA antibody, anti-Sm, antiphospholipid antibody)
S- Serositis

A- ANA
N- Neurological problems
A- Arthritis

1. 95% of SLE are ANA +ve.
2. 50% dsDNA +ve, but is specific for SLE.
3. 25% RF +ve.
Prof Esha : Even if you forget the pnemonic, remember there are 4 mucocutaneous
features,(malar rash,photosensitive rash,discoid rash and oral ulcer),4 systemic
involvement( CNS,serositis,kidney and arthritis) and 3 lab finding(
Heamatology,Immunology and ANA stands on its own) Recent addition in lab
features are positive anti Sm antibody,antiphospholipis antibody,and low
complements,along with direct positive coomb's test.

36) ON MONOFILAMENT testing
Dear Yin Ling,

Are you familiar with Monofilament sensory testing devices which consist of a single
strand of nylon (typically attached to a plastic or paper handle) that can produce a
characteristic downward force when buckled onto a surface?
Are you aware that they come in different sizes?

The monofilaments commonly used to screen for sensory neuropathy are 4.17, 5.07,
and 6.10. The use of a single 5.07 monofilament is the accepted standard in medical
practice to screen for the minimum level of protective sensation in the foot. Ten
grams of reproducible buckling stress force are required to bend the 5.07
monofilament.

Why do we use monofilament testing? Why do we not just use our usual pin, cotton
and tuning fork??

When the monofilament bends, its tip is exerting a pressure of 10 grams (therefore
this monofilament is often referred to as the 10gram monofilament). If the patient
cannot feel the monofilament at certain specified sites on the foot, he/she has lost
enough sensation to be at risk of developing a neuropathic ulcer.
Testing of diabetic patients for protective sensation may be simplified to testing
under both first metatarsal heads. If a patient cannot sense the application under

either first metatarsal head, he or she probably has lost protective sensation and
should be considered to be at risk for undetected injury.

Generally, No person with a foot ulceration could feel the 5.07 (10g) filament,
concluding that monofilaments are an effective, inexpensive and simple screening
device in identifying the at risk foot.\

In contrast, a person who can feel the 10-gram filament in the selected sites is at
reduced risk for developing ulcers.
The patient must not watch while the examiner applies the filament.
Pre-Test the monofilament on the patient's hand or sternum so he/she knows what
to anticipate.

Typically we test five sites and document the findings. The number of sites may vary
from centre to centre.
Apply the monofilament perpendicular to the skin's surface
Apply sufficient force to cause the filament to bend or buckle

Pls Apply the filament along the perimeter and NOT ON an ulcer, callus, scar or
necrotic tissue. Do not allow the filament to slide across the skin or make repetitive
contact at the test site.
Have patients identify at which time they were touched. To avoid guessing,
randomize the sequence of applying the filament throughout the examination.
Patients should have their feet examined at least annually for impaired sense of
pressure, vibration, pain, or temperature, which is characteristic of peripheral
neuropathy.

Pressure sense is best tested with a monofilament esthesiometer as this picks up
patients with high risk of diabetic ulcers

Yin Ling,
The overall risk of developing a diabetic foot ulcer is determined by a combination of
factors.

In general, the risk is higher if:

Neuropathy is more severe (because more sensation is lost and multiple small
trauma breaks the skin)

Peripheral vascular disease is more severe (because there is less circulation to bring
enough oxygen to repair tissue damage. Just look at the dry black feet with curled
up nails and you see feet which are deserts)

There are coexisting abnormalities of the shape of the foot which make the local
effects of neuropathy or vascular disease more severe (because it increases local
pressure and callus; heavens let us ban all those fashionable but cruel footwear that
ladies wear as it deforms the feet into an abnormal shape)

The patient who is unable to practise reasonable self care of the feet and to prevent
trauma (because there are more chances of damaging the feet with fungal infections
in the webs, poor nail hygiene and cutting)

The diabetic control is very poor (because of susceptibility to infection and poor
wound healing)

There is a past history of foot ulceration due to diabetes (because all the above
factors persist)
Dear Yin Ling,

People often ask which is the earliest abnormality and expect a single answer that is
dogmatic. The truth is however much more complex and the answer is "It varies!"

Sensory or sensorimotor distal polyneuropathy is the most common of the diabetic
neuropathies. With this, numbness and paresthesias begin in the toes, and gradually
and insidiously ascend to involve the feet and lower legs. Very common, we see it all
the time.

With a sensory or sensorimotor distal polyneuropathy, both lightly myelinated and

unmyelinated small nerve fibers and the myelinated large nerve fibers are affected.
Small and large fiber dysfunction occurs in varying combinations;
however, in most cases, the earliest deficits involve the small nerve fibers.

Features characteristic of a small fiber peripheral neuropathy include burning or
lancinating pain, hyperalgesia, paresthesias and dysesthesias, **deficits in pain
and temperature perception** leading to foot ulceration.

Features characteristic of large fiber peripheral neuropathy include the loss of
position and vibration perception sense and loss of deep tendon reflexes, tested
with tuning fork and Ankle jerks.

So based on this what will you, yin ling, select as a mode for screening which must of
course pick up the pathology early!?

37) ON ISOLATED RAISED GAMMA GT
Dear YL,

Raised Gamma GT in isolation is a very common finding
It is From hepatocytes and biliary epithelial cells, pancreas, renal tubules and
intestine. Very sensitive but Non-specific
It is Raised in ANY liver disease hepatocellular or cholestatic But Usefulness is limited
It helps Confirm hepatic source for a raised ALP
Alcohol induces it
As it is an Easily Induced enzyme- many Drugs elevates it
*Pls note that an Isolated increase does not require any further evaluation, suggest
watch and rpt 3/12 and only if other LFTs become abnormal then investigate

38) ON PCM TOXICITY
Dear YL,
Are you aware that Paracetamol toxicity can occur in much lower doses in certain
circumstances eg

Alcohol use
Fasting state- Depletion of glutathione
Beware of overdosing in the fasting patient with pain eg. post surgery!!!
So if you jog, get dehydrated, low glutathione now and have muscle aches for which
you take PCM, you may have liver damage!!

39) ON ALCOHOL
Dear Yin Ling,
our Muslim friends are absolutely right with alcohol prohibition. And the Buddhist
Precepts also do not allow Alcohol consumption.

Alcohol, a very simple molecule is probably the most widely used drug in the world.
It is distributed to all the organs and fluids of the body, but it is in the brain that
alcohol exerts most of its effects. Like other general anesthetics, alcohol is a central
nervous system depressant. In general, its effects are proportional to its
concentration in the blood.

Yin Ling, How does the body handle alcohol?

Alcohol is rapidly absorbed from the gastrointestinal tract into the bloodstream and
from there it is distributed throughout the other bodily fluids and tissues. Alcohol is
principally metabolized by the liver into acetaldehyde, with the remainder being
excreted in the urine.
On average, it takes the liver about an hour to break down one unit of alcohol --
the amount typically found in 12 ounces of beer, 4 ounces of wine or one ounce of
50 proof hard liquor.

Blood alcohol levels decline at a fixed rate irrespective of the amount consumed.
The more consumed, the longer it takes to be metabolized. Additionally, blood
levels are greatly, and inversely, influenced by body weight. The thinner you are,
the greater the alcohol blood level for any given amount of alcohol consumed.
Because of these factors, blood levels may remain elevated for many hours after the
last drink.

Alcohol may impair temperature regulation both in the cold, and in the heat, in
countries with extreme climates, this can kill. It is also a potent diuretic and this may
lead to dehydration.


Yin Ling, What are the long term adverse effects of alcohol?

The chronic abuse of alcohol may cause numerous adverse health effects which
include:

Chronic alteration of brain and nerve function
Weakening of heart muscle
Testicular shrinkage and male breast enlargement
Impotency
Elevated triglycerides
Fat deposits in the liver
Cirrhosis and liver failure
Blood-clotting abnormalities
Pancreatitis
Vitamin deficiencies
Chronic skin alterations
Death

Chronic moderate ethanol ingestion by young female mice results in decreased
fertilization, embryo growth retardation, and abnormal embryo development in
vitro. The cardiovascular consequences of heavy ethanol consumption are several. If
ethanol exposure occurs prenatally at a period when the heart of the infant is
developing, then structural damage is observed and manifests as diminished
capacity of the heart to function properly. If the cardiovascular system is exposed to
excessive ethanol later in life, then a variety of problems can manifest, most
prominently are cardiomyopathy, hypertension, stroke and cardiac arrhythmias.

Cardiac arrhythmias have been observed after both acute intake of large amounts of
ethanol and after chronic alcohol consumption. For example, ethanol intake over a
long weekend may result in electrophysiological anomalies referred to as "holiday
heart syndrome", whereas sudden cardiac death has been associated with
alcoholism. A number of hypotheses have been advanced to explain the
disturbances in cardiac rhythms. These include scarring of the heart muscle,
alterations in the chemicals, which influence heart function such as electrolytes and
catecholamines, and alterations in the amount of oxygen coming to the heart.

There appear to be several mechanisms by which ethanol can lead to
cardiomyopathy. These include the following:


1. an alteration in the flow of calcium ions in the cardiac muscle, which in turn
reduces the efficiency by which calcium activates muscle contraction;
2. modification of the action of contractile proteins, actin and myosin;
3. reduction in the synthesis of proteins needed for contraction and energy;
4. an influence of deleterious ethanol metabolites (acetaldehyde) and free radicals;
and
5. activation of genes which may promote cell death.

Yin Ling, does alcoholism cause Cancer?

Consumption of alcoholic beverages is causally related to cancers of the mouth,
pharynx, larynx and esophagus and that studies indicate that the risk is most
pronounced among smokers and at the highest levels of consumption.

There is evidence that suggests a link between alcoholic beverage consumption and
cancer of the liver and breast.

There are some circumstances under which diabetics should not drink alcohol in any
amount. The key for those with diabetes is to understand what conditions can be
worsen if they consume alcohol.

According to the American Diabetes Association, drinking alcohol is a poor choice if
diabetics have the following conditions:

Nerve damage in the arms or legs.
Diabetic eye disease.
High blood pressure.
High levels of triglycerides.
Alcohol can damage nerve cells; even light drinking can cause nerve damage. For
diabetics with nerve damage drinking can increase the pain, numbness, tingling or
burning sensation associated with diabetic nerve damage.
For diabetics with eye disease symptoms, heavy drinking can make the condition
worse and heavy drinking is defined as three or more drinks during one day.
Diabetics who also have high blood pressure should also not drink alcohol.

Pls remember that Alcohol increases the amount of triglycerides in the blood. Even
very light drinking, defined as two drinks a week, can increase triglyceride levels.
Diabetics who have high triglycerides should not drink alcohol at all.

Yin Ling, what are the Sexual effects of alcohol?



Soon after consuming alcohol, is found the following common effects:
Alcohol has a disinhibiting effect, which can make people loosen up and feel
more comfortable initiating or engaging in sex. Alcohol may make you feel more
socially confident and in small quantities may facilitate more socializing and sexual
communication.
In small amounts alcohol has been reported to have a positive impact on sexual
desire and arousal. At the same time, research shows that even after a few drinks
sexual response is reduced.
In large amounts alcohol makes sex difficult to impossible. While in moderate
amounts alcohol can have an impact on engaging in risky sexual behavior, although
this impact is not fully understood.
As drinking increases both men and women will experience a reduction in sexual
arousal, men may have difficulty getting erections, and both men and women may
have difficulty experiencing orgasm.

yin ling, what are the Long term sexual effects of alcohol?

Chronic alcohol abuse, or alcoholism, inevitably has a devastating effect on
sexuality, including:
Erectile disorders and dysfunction in men
Loss of sexual desire, significant decrease in sexual arousal for men and women
Difficulty experiencing orgasm for men and women

Yin Ling, what causes Flushing in some people after drinking alcohol?

Studies of autopsy liver specimens from individuals of different racial groups
revealed a polymorphism in alcohol dehydrogenase (ADH) and aldehyde
dehydrogenase (ALDH). About 85% of the Japanese livers had an atypical ADH and
52% of the livers an unusual ALDH.

Data on the distribution of phenotypes in random European and Japanese
population as well as family studies suggest a direct relationship between the lack of
low Km isozyme of ALDH and alcohol-induced biological sensitivity. Alcohol
sensitivity is quite common in individuals of Mongoloid origin and might be due to
delayed oxidation of acetaldehyde by an unusual type of ALDH.

Many people of East Asian descent have a mutation in their alcohol dehydrogenase

gene that makes this enzyme unusually effective at converting ethanol to


acetaldehyde, and about half of such people also have a form of acetaldehyde
dehydrogenase which is less effective at converting acetaldehyde to acetic acid.
This combination causes them to suffer from the alcohol flush reaction, in which
acetaldehyde accumulates after drinking, leading to severe and immediate hangover
symptoms.


But it is Not just alcohol which causes flushing! Pls think of other causes as well!

40) ON BENCE JONES PROTEIN


Dear YL,


not many medical students know about this test now. When I was a student, Prof
Florence Wang would make us take urine and test it at the lab ourselves. Hence
these are sheared into my memory.

Urine is the best specimen in which to look for Bence Jones proteins which if present
suggests Multiple Myeloma. Proteins are usually too large to move through a
healthy kidney, from the blood into the urine. Bence Jones proteins are an
exception. They are small enough to move quickly and easily through the kidney
into the urine.

A routine dipstick urinalysis will not detect Bence Jones proteins. There are several
methods used by laboratories to detect and measure these proteins.

The classic Bence Jones reaction involves heating urine to 60C in a test tube placed
in a waterbath. At this temperature, the Bence Jones proteins will clump. The
clumping disappears when the urine is further heated to boiling and reappears
when the urine is cooled.

This simple test will reveal whether or not Bence Jones proteins are present, but not
how much is present. If you are in a rural hospital, this DIY test may be invaluable.
However, This classic heat test is plagued by a high false positive rate. The high tech
test for BJ in the urine is by electrophoresis

41) ON MYOCARDIAL O2 DEMAND


Dear YL,
What is Myocardial O2 demand?

Myocardial O2 demand is determined mainly by heart rate, systolic wall tension, and
contractility, hence narrowing of a coronary artery typically results in angina that
occurs during exertion and is relieved by rest. Simple but That's for the stable
plaques of course.

Q: How did William Heberden describe Classic angina?

Pls note a classic- Angina immobilises the patient from whatever he is doing -
he/she STOPS!

In addition to exertion, cardiac workload can be increased by disorders such as
intercurrent illnesses, fever, hypertension, aortic stenosis, aortic regurgitation, or
hypertrophic cardiomyopathy.

Q: What are the Classical Presentation symptoms of Aortic stenosis?

LVH from whatever cause can also decrease relative myocardial perfusion because
myocardial mass is increased (causing decreased diastolic flow).

A decreased O2 supply, as in severe anemia or hypoxia, can precipitate or aggravate
angina.

In stable angina, the relationship between workload or demand and ischemia is
usually relatively predictable. The great John Hunter predicted that his life is in the
hands of any rascals that infuriated him, I assumed he was refering to his med
students but nope he died after a hospital board meeting from a massive AMI. Told
you faculty meetings are dangerous affairs!

As the myocardium becomes ischemic, ventricular function deteriorates. Left
ventricular (LV) diastolic pressure usually increases during angina, sometimes
inducing pulmonary congestion and dyspnea. Remember 'angina equivalents' !
The exact mechanism by which ischemia produces discomfort is unclear but may
involve nerve stimulation by hypoxic metabolites.

Q: What are the limitations of the ECG?

The ECG reveals the heart rate and rhythm only during the time that the ECG is
taken. A spot picture only

The ECG can often be normal or nearly normal in patients with undiagnosed
coronary artery disease (false negative results.)
On the other hand, many "abnormalities" that appear on the ECG may turn out to
have no medical significance (false positive results).

Many people with ischaemic heart disease have a normal ECG at rest. During
exercise the heart beats faster and needs more oxygen. If one or more of the
coronary arteries are narrowed, part or parts of the heart muscle do not get enough
oxygen. This can cause the ECG tracing to become abnormal.

The degree of abnormality on the exercise ECG tracing can give a good idea of the
severity of the disease. Therefore, an exercise ECG test is often done to help to
decide if further investigations is needed. But I have colleagues who go straight to a
CT angio but that's another story.

Because angina resolves quickly with rest, ECG rarely can be done during an attack
except during stress testing. If done during angina, ECG is likely to show reversible
ischemic changes: T wave inversion, ST-segment depression (typically), ST-segment
elevation (ST-segment elevation rather than depression occurs during attack in
variant angina), decreased R-wave height, intraventricular or bundle branch
conduction disturbances, and arrhythmia (usually ventricular extrasystoles).

If the ECG is NORMAL despite pain, we can only be confident that the disease is
lightly NOT severe and outlook good, but we cannot exclude it if the history is
TYPICAL.
The resting ECG is often normal in stable angina pectoris in the absence of a
previous MI or a cause for LVH. Abnormal ECG changes are more common with
unstable angina pectoris.

The resting ECG (and usually LV function) at rest is normal in about 30% of patients
with a typical history of angina pectoris, even those with extensive 3-vessel disease.
In the remaining 70%, the ECG shows evidence of previous infarction, hypertrophy,
or nonspecific ST-segment and T-wave abnormalities.
In men with chest discomfort suggesting angina, stress ECG testing has a specificity
of 70%; sensitivity is 90%. Sensitivity is similar in women, but specificity is lower,
particularly in women < 55 (< 70%). However, women are more likely than men to

have an abnormal resting ECG when CAD is present (32% vs 23%). Although
sensitivity is reasonably high, exercise ECG can miss severe CAD (even left main or 3-
vessel disease). I had an eminent professor who had a 'normal' stress test a week
before his fatal infarct.

In patients with atypical symptoms, a negative stress ECG usually rules out angina
pectoris and CAD; but a positive result may or may not represent coronary ischemia
and indicates need for further testing.

The HISTORY is the MOST important diagnostic feature, the specificity of ST-T and T
wave abnormalities is provided more by the clinical circumstances in which the ECG
changes are found than by the particular changes themselves.

In patients who presents with a history suggestive of AMI within the last 2 - 3 hours,
a Normal/Nondiagnostic initial ECG predicts low risk. This is the reassurance of the
"Normal finding" but it cannot EXCLUDE the diagnosis, hence we need follow-up
ECGs, and blood tests.

Please realise that 3-10% of MI patients have initial normal ECG!
Of course, 25% of patients with missed MI had misread ECGs!!

It is important to compare with prior ECGs if available for this Increases Specificity!
Have a high level of suspicion when facing a typical history and seeing ECG Markers
of underlying CAD like
Eg
Left Ventricular Hypertrophy with or without 'Strain Pattern',
ST segment changes; how deep is the ST depression or elevation,
T Wave changes,
Q Waves in 2 contiguous leads to suggest previous events,
Loss of Progression of R waves to suggest previous events,
Left Bundle Branch Block or other conduction changes

On the other hand, some people have "abnormal" ECGs at baseline but this may be
normal for them. It is important that an electrocardiogram be compared to previous
tracings. The History cannot be over-emphasized for its importance in diagnosis
despite all our modern technologies.
I hope you learn this well.

Please remember that Diabetics have a high risk of Ischaemic heart disease and may
have atypical symptoms. A high degree of suspision is needed when managing these
patients and interpreting their ECGs.


Yin Ling: CHD RISK IN DIABETES
1. DM doubles the CHD risk for men and triples the CHD risk in women. Women do
worse
2. DM pt do poorly in regards to cvs outcomes
3. Indigenous ppl higher proportion of diabetes -? Thrifty genes
4. Current prevalence of diabetes in msia acc to NHMS is 15% , in which >50% is
undx, most of them are fr rural areas. They present to us with DM complications
5. 4/5 of diabetic patient DO NOT achieve glycemic target
Why the increased cvs risk in diabetics?
1. Genetics v important. If father and mother has diabetic 95% risk for offspring will
have DM!! Chromosome 11n15
2. Women with DM have a higher risk of CHD than men (>40% greater risk]
3. Cvs events starts 10 yr before the dx of DM
4. DM pt with acs have a lot more TCFA (thin cap fibroadenoma).
Increased lipid pool and Increased wall stress in hypertensives, plus inflammation....
All these are Key players in plaque rupture.

Message : global risk reduction is impt. Stenting cannot cure the problem bec we
have 60000 miles of arteries!!
5. Smoking In DM pt greatly elevate the risk.
7. Increase insulin resistance increases the risk of CHD. Insulin resistance is a Core
defect in type 2 DM.
A Fat belly with normal BMI increases cvs risk (3X) - esp in Asians
DM and HPT
In Diabetes Mellitus the superoxide pathway is activated leading to dilation of the
afferent vessels in the glomerulus resulting in intraglomerular hypertension.
ACEI and ARB both dilates the Efferent vessels leading to a reduction in the
intraglomerular pressures. This help to reduce proteinuria and slow down
progression to renal failure.
But Watch out for deterioration of renal function because of this effect. If > 20% the
drug may have to be stopped.
Hence they are the drug of choice for DM with HPT. But DO NOT COMBINE these 2
drugs.
Patients with proteinuria shd have BP of 135/85 and below. The target values have
undergone changes lately. It was 125/75 before but in Accord and Advance no
benefit was shown at this value in terms of mortality.
If > 1gram of proteinuria aim for <130/80
eGFR must be looked at esp in DM as se creatinine may not reflect their renal
function.
Salt and weight reduction is crucial for control of both DM and HPT.

Pls remember that Women are very different when compared to men as regards
CHD.
Stress testing is Cheap and easily available.
But women are from different planets and almost certainly a different species when
it comes to cvd!
Oestrogens? Is that the difference?

Stress test is still the diagnostic tool of choice.


But women have more microvascular disease which is not well picked up.
Remember Bayers theorem?
Pretest probability plus test accuracy
Pretest probably fr history helps us to determine the level of risk. Stress testing is
Most useful in INTERMEDIATE RISK GROUPS.
ECG changes in Low risk grps will likely be seen as false positive
while ECG changes in High risk grps will likely be interpreted as Positive, and as false
negative even if ecg normal
We must Look beyond the ST segments of mild ST depressions , slow upsloping and
classic ST downsloping
Women have similar Negative predictive values as men but much lower Positive
predictive values
METS: tells us about exercise capacity. If > 10mets likely low risk
If > 10mets with NO ST CHANGES VERY LOW RISK.
High risk if <5mets achieved
Chronotropic aka heart rates
220 - age for males
206 (0.88 times age in yrs) for women
Systolic bp is 160 to 190 as we exercise
> 10% drop in expected bp suggests possible lv dysfunction
Higher bp values > 190 suggests hpt.

HRT: is there any CV protection?
A third of a womens LIFE is POST MENOPAUSAL. Think abt that!!!
More women die from heart disease than men. Perhaps its because men broke their
hearts. More women die fr cvd than from Ca!
Women present with cvd a decade later than men because of possible pre
menopausal protection by oestrogen
Bld vessels have oestrogen receptors. It lowers ldl and increase hdl
But it increases TG and
IT DECREASES FASTING GLUCOSE AND INSULIN

HERS is a sec prevention study which changed thinking. No reduction in risk of MI


seen with hrt. There was increase in risk of stroke.
WHI is a primary prevention study which also did NOT SHOW ANY REDUCTION IN
RISK.
THERE IS HARM FROM VTE
Oestrogen plus progesterone increased risk of breast Ca, vte and stroke!!
In the oestrogen alone arm there was reduced risk of breast ca but still a small
increase of risk of stroke.
HRT is Not for primary or sec prevention of cvd
Q: were the women given hrt too late in the trials? Most were > 5yrs post
menopause
Younger patients given hrt appears to have better results... 30% RRR of chd
Not much data unfortunately
DOPS STUDY IS THE ONLY TRIAL In younger women and it shows RRR. We await
more data.

42) ON REFLEXES!
Dear Yin Ling,

Today I demonstrated Hoffman's and Chaddock's sign in a patient to my students at
my clinic. The patient's plantar response was equivocal but Chaddock's sign was
crystal clear. To my horror the 2 kiddies did NOT know anything about the 2 signs.

What are they??

Hoffmann's sign, named after the German neurologist, Johann Hoffmann (born
1857, Rheinhesse; died 1919, Heidelberg), is a finding elicited by a reflex test which
verifies the presence or absence of problems in the corticospinal tract. It is also
known as the finger flexor reflex.
The test involves tapping the nail or flicking the terminal phalanx of the third or
fourth finger. A positive response is seen with flexion of the terminal phalanx of the
thumb and index finger.


Relation to Babinski sign

Hoffmann's sign is often considered the upper limb equivalent of the Babinski's sign
because it, like the Babinski sign, indicates upper motor neuron dysfunction. BUT Its
mechanism differs considerably from the Babinski which is also known as the plantar
reflex; Hoffmann's sign involves a monosynaptic reflex pathway in Rexed lamina IX
of the spinal cord, normally fully inhibited by descending input. The pathways
involved in the plantar reflex are more complicated, and different sorts of lesions
may interrupt them. This fact has led some to reject any analogies between the
finger flexor reflex and the plantar response.

Superficial reflexes are motor responses to scraping the skin.
They are graded simply as present or absent and markedly asymmetrical responses
would be considered abnormal.
These reflexes are VERY different from the muscle stretch reflexes in that the
sensory signal has to not only reach the spinal cord, but also must ascend the cord
to reach the brain. The motor limb than has to descend the spinal cord to reach the
motor neurons. This is a polysynaptic reflex.

This can be abolished by lower motor neuron damage or destruction of the sensory
pathways from the skin that is stimulated. However, the utility of superficial reflexes
is that they are decreased or abolished by conditions that interrupt the pathways
between the brain and spinal cord (such as with spinal cord damage).

Classic examples of superficial reflexes include the abdominal reflex, the cremaster
reflex and the normal plantar response.

The abdominal reflex includes contraction of abdominal muscles that is stimulated
by scraping the skin superficially and rapidly along a dermatome towards the
umbilicus. This contraction can be seen as a brisk contraction of the abdominal
muscles with the umbilicus moving towards the stimuli.

The cremaster reflex is produced by scratching the skin of the medial thigh, which
should produce a brisk and brief elevation of the testis on that side.

The normal plantar response occurs when scratching the sole of the foot from the
heel along the lateral aspect of the sole and then across the ball of the foot to the
base of the great toe. This normally results in flexion of the great toe (a "down-going
toe") and, indeed, all of the toes.

The "anal wink" is a contraction of external anal sphincter when the skin near the
anal opening is scratched. This is often abolished in spinal cord damage (along with
other superficial reflexes ).

"Pathological reflexes"

The best known (and most important) of the so-called "pathological reflexes" is the
Babinski response (upgoing toe; extensor response). The full expression of this reflex
included extension of the great toe and fanning of the other toes. This is actually a
superficial reflex that is elicited in the same manner as the plantar response (i.e.,
scratching along the lateral aspect of the sole of the foot and then across the ball of
the foot toward the great toe). This is a primitive withdrawal type response that is
normal for the first few months of life and is suppressed by supraspinal activity
sometime before 6 months of age.


Damage to the descending tracts from the brain (either above the foramen magnum
or in the spinal cord) promotes a return of this primitive protective reflex, while at
the same time abolishing the normal plantar response. The appearance of this reflex
suggests the presence of an upper motor neuron lesion. Chaddock's, Oppenheim,
Gordon's are all tested for when the extensor response is equivocal.

GORDON'S SIGN :
Gordon's sign is a clinical sign in which squeezing the calf muscle elicits an extensor
plantar reflex. It is found in patients with pyramidal tract lesions, and is one of a
number of Babinski- like responses.

CHADDOCK'S SIGN:
Chaddock reflex is a diagnostic reflex similar to the Babinski reflex. It is designed to
identify lesions of the pyramidal tract, via stimulation of the skin over the lateral
malleolus leading to extension of the big toe.

It was identified by Charles Gilbert Chaddock



Oppenheim's sign is dorsiflexion of the big toe elicited by irritation downward of the
medial side of the tibia. It is named for Hermann Oppenheim.

Other "BABINSKI- LIKE RESPONSES" for the distinction student

Abnormal reflex seen as extension of the big toe
Bing's sign multiple pinpricks on the dorsum of the foot
Cornell's sign scratching the dorsum of the foot
Gonda's sign flexing and suddenly releasing the 4th toe
Moniz sign forceful passive plantar flexion of the ankle
Schaefer's sign squeezing the Achilles tendon
Stransky's sign vigorously abducting and suddenly releasing the little toe
Strmpell's sign patient attempts to flex the knee against resistance
Throckmorton's reflex percussion over the metatarsopahalangeal joint of the big
toe

Abnormal reflex seen as flexion of toes
Bekhterev-Mendel reflex flexion of the 2nd to 5th toes on percussion of the
dorsum of the foot

Rossolimo's sign exaggerated flexion of the toes induced by rapid percussion on


the tips of the toes

43) on PULSES
Dear Yin Ling,

Today's bedside class reveal a remarkable lack of knowledge about the physical signs
of valvular diseases. It is clear to me that the students know VERY little about the
examination of the pulse.

Pulse characteristics are often assessed at the radial and carotid arteries. However,
all of the peripheral pulses should be examined. Palpation of both radial pulses
simultaneously may pick up an Aortic dissection, and a radial and femoral pulse
together will help establish the presence of coarctation of the aorta.

The initial assessment of pulse rate, volume and character is usually obtained from
the radial pulse. The pulse is lovingly felt with the your index and middle fingers on
the palmar side of the wrist with your thumb on the dorsum of the wrist. Be sure to
explain to the patient what you are doing before attempting to palpate the femoral
pulse!

Radio-femoral delay is classically attributed to coarctation of the aorta, usually due
to narrowing of the aorta just beyond the origin of the left subclavian branch. It is
more often seen in young men. Sometimes a noticeably lower pulse volume in the
femoral artery compared to radial artery is all that is appreciated rather than the
classic delay in the femoral pulse. In older patients, radial femoral delay can
potentially occur due to atherosclerosis and stiffness between the lower limbs and
upper limbs arteries. Typically stiffer arteries lead to an increase in pulse wave
velocity with more rapid propagation of the pulse wave. These patients also tend to
have hypertension. Coarctation is sometimes diagnosed in older adults and should
not be dismissed because of old age.

Rate
Ideally, Count the number of beats per minute. It is often convenient to count the
number of beats over 15 seconds and then multiply by four. If the pulse is slow or
irregular however, counting over a full 60 seconds will be more accurate. A normal
resting heart rate ranges between 60 and 100 beats per minute.

It is a normal for the pulse rate to vary slightly with respiration, particularly in the
young, called sinus arrhythmia. This arrhythmia is related to differential filling of the
left and right side of the heart and vagal tone with inspiration and expiration.
Typically the heart rate increases slightly during inspiration, and decreases with
expiration. As it is partly mediated by vagal tone, this effect, tends to decline with
age.

Rhythm
The pulse is regular with sinus rhythm (apart from the caveat of sinus arrhythmia
described above). A very rapid regular rhythm may indicate sinus, supraventricular
or ventricular tachycardia. An irregular pulse can be regularly irregular (a recurring
pattern such as bigeminy or type II heart block) or irregularly irregular (no clear
pattern, such as atrial fibrillation).

Volume
The volume of the pulse is best assessed by palpating one of the larger arteries such
as the carotid, brachial or femoral pulses. It is a subtle sign that requires experience
over years for the examiner to recognize low and high volume pulses. A semi-
quantitative scale is used to describe pulse volume (increased, normal, reduced,
thready)

Character
This refers to an impression of the pulse waveform derived during palpation.

Some abnormalities of pulse are described below.

Anacrotic pulse
This is seen in aortic stenosis, and refers to a pulse wave that is slow rising and
generally flat volume associated with a low cardiac output and prolonged left
ventricular ejection time. It suggests more severe aortic stenosis.

Collapsing pulse
This is a sign of aortic regurgitation, although it is also seen in patients with a
hyperdynamic circulation and with a rigid arterial system. A stiff arterial system
leads to an accentuated systolic peak in the peripheral pulses. The pulse has an early
peak and then quickly falls away, giving it a tapping quality. The preferred method is
to palpate the radial pulse with the palm and fingers wrapped around the flexor
aspect of the wrist, and with the arm elevated upwards. This accentuates the
tapping quality of the pulse. The collapsing pulse is also referred to as Corrigans or a

water-hammer pulse, after a 19th century toy that was a vacuum tube containing
water or mercury that was flipped creating a tapping or hammer sensation at the
fingertips.

When a collapsing pulse is detected look for the following signs;

Duroziez sign: Seen in severe aortic regurgitation. Place the diaphragm of the
stethoscope over the femoral artery and press downwards. Initially a systolic
murmur will be heard. Gradually increase pressure over the artery- a diastolic
murmur will become evident also related to the flow reversal with profound aortic
regurgitation. Now tilt the proximal edge of the stethoscope further downwards if
aortic regurgitation is present the systolic murmur is accentuated and the diastolic
component is diminished. Now tilt the distal edge of the stethoscope downwards,
the diastolic component will now be accentuated and the systolic reduced. This sign
has a positive predictive value of close to 100% for aortic regurgitation, and can
detect this lesion in some patients in whom it is not possible to hear the
characteristic diastolic murmur on auscultation of the heart.

Traubes sign: A pistol shot sound heard over the femoral artery with the aid of a
stethoscope. It is necessary to compress the femoral artery distal to the stethoscope
head to produce the characteristic double tone sound.

Hills sign: This is a nonspecific sign of aortic regurgitation- it is also seen in other
causes of a hyperdynamic circulation, such as thyrotoxicosis, beri-beri, or pregnancy.
Check the blood pressures in the upper and lower limbs. If the pressure in the lower
limbs exceeds that in the upper limbs by more than 20 mmHg then the sign is
positive.

Quinkes sign: Pulsatile blanching of the nail bed

De Mussets sign: Named after the famous French poet whose head nodded in time
with his arterial pulsations due to his syphilis related aortic regurgitation.



Pulsus paradoxus
This is a misnomer. This is an exaggerated physiological phenomenon, rather than a
paradox as the name implies.

The volume of the pulse rises with expiration with the increase in stroke volume.
and falls during inspiration. When it is present, it suggests either restricted left
ventricular filling during inspiration (associated with a mild increases in pericardial
pressure and increased right heart filling that shifts the interventricular septum
towards the left ventricle to impair left sided filling) such as in pericardial
tamponade, or exaggerated changes in intrathoracic pressure as in severe asthma..
Other causes of pulsus paradoxus include right ventricular infarction, large
pulmonary embolus, and tense ascites or obesity.

Pulsus alternans
This abnormality describes a pulse that alternates between a larger and smaller
volume on a beat to beat basis. This is a regular pulse and is seen in severe cardiac
failure.

Jerky pulse
This is often seen in hypertrophic cardiomyopathy as the hypertrophied ventricle
rapidly empties and then quickly drops its output as the outflow pathway is
obstructed.-

44) on FUNCTIONAL MURMURS
Dear Yin Ling,

I tried very hard today to teach the kiddies the approach to cardiac examination.
Most importantly I told them that the best thing they can do to improve their
cardiac examination technique is to throw away the stethoscope. The majority of
Cardiac problems can be diagnosed from peripheral signs without the stethoscope.
The stethoscope confirms what we already know.

Hence I get infuriated when the kids rush in to auscultate. NO NO NO! That's the
final curtain call!
Innocent systolic ejection murmurs are produced by turbulent flow through the
proximal arteries at the time of ventricular ejection. The intensity of the murmur, as
influenced by stroke volume or velocity of ejection, and proximity of the arteries to
the chest wall determine whether the murmur is audible with the stethoscope.

This ESM type murmur tends to peak in early to midsystole, usually ends before the
second heart sound, and is best heard on the left side over the mitral or aortic valve
or at the thoracic inlet. The murmur may be the result of turbulent flow into the

pulmonary artery or aorta, or both.



Functional systolic murmurs are produced by increased velocity of blood within the
cardiovascular system and by extracardiac factors. Anemia, fever, hyperthyroidism,
and tachycardia of any cause may produce functional murmurs.

Decreased blood viscosity and increased velocity of blood flow produce turbulence.
The functional murmur of anemia is usually of low intensity and high frequency and
occurs during early systole to midsystole. Anemic murmurs are best heard over the
mitral valve or aortic valve area.

Functional murmurs may be audible when states of high cardiac output exist. The
increased cardiac output is produced by both tachycardia and increased stroke
volume.

Some characteristics of Innocent/Functional Murmurs:
1. Systolic in nature
2. Usually short in duration
3. Usually soft
4. Usually heard along left sternal edge
5. Intensity varies with phases of respiration and posture - usually louder when
supine
6. Intensity louder with exercise, anxiety, fever

Structural Lesions which may be missed:
There are some structural lesions which give rise to murmurs closely resembling an
innocent murmur. These differential diagnoses must be borne in mind and efforts
made to exclude them clinically or with the help of investigations such as an
electrocardiogram (ECG) or chest X-ray (CXR).

Structural lesions which may be missed.

Atrial Septal Defect:
Ejection systolic murmur
Fixed splitting of 2nd heart sound
Mid-diastolic murmur at tricuspid area
Right bundle branch block

Pulmonary Valve Stenosis:


Ejection systolic murmur
ejection click, at 2nd intercostal space
Murmur best heard during inspiration
Post-stenotic dilalation of pulmonary artery

MILD Aortic Valve Stenosis:
Ejection systolic murmur at aortic area with ejection click
LVH

Mitral Valve Prolapse:
Midsystolic murmur at mitral area with ejection click

Hypertrophic Cardiomyopathy:
Family history
Systolic murmur at LSE radiating to aortic area
Abnormal deep Q waves seen
LVH

45) on VON RECKLINGHAUSEN
Dear Yin Ling,

I always have difficulty remembering the chromosomes involved

"Neurofibromatosis Type I" = "von Recklinghausen has exactly 17 alphabets!" =
Chromosome 17
"Neurofibromatosis Type II" = "Type 2 = 22" = Chromosome 22

46) on WILSONS DISEASE
Dear Yin Ling,
To help you remember an uncommon condition commonly seen in exams.....

Wilson's disease : ABCD

A - Asterixis
B - Basal ganglia degeneration

C - Copper accumulation with reduced Ceruloplasmin level, causing Cornea deposits


(Kayser-Fleischer rings), Choreiform movements, psyChiatric abnormalities, liver
Cirrhosis and treatment is with Chelation.
D - Dementia

47) on OCULOMOTOR NERVE
My dear yin ling,

Cranial nerves III, IV and VI are usually tested together. You instruct the patient to
hold his head still and follow only with the eyes your finger or penlight that
circumscribes a large "H" in front of the patient. Or you place your hand on his head
to immobilise it for other than the abnormal medical person, any normal person will
turn his head when asked to follow the finger!

Since the oculomotor nerve controls most of the eye muscles, damage to this nerve
is also known by the down n' out signs, because of the position of the affected eye.

Pupillary reflex
The oculomotor nerve also controls the constriction of the pupils. This can be tested
in two main ways. By moving a finger rapidly towards a person's face to induce
accommodation, their pupils should constrict.

Shining a light into their eyes from the side will make their pupils constrict. Do not
shine from directly in front for any normal human will look at the light hence have
accommodation reflex! Both pupils should constrict at the same time, independent
of what eye the light is actually shone on. I tell my students a hundred times to
Remember to approach from the side as the patient WILL focus on the penlight if
you approach from his front and hence initiate accommodation and associated
pupillary constriction, yet when I examine them, they will nonchalantly shine from
the front. Peptic ulcer disease!

Argyll-Robertson pupil (Aka "Prostitute's pupil" - A prostitute Accommodates, but
does not react )
Accommodation reflex present, Pupillary reflex absent

The oculomotor nerve arises from the anterior aspect of midbrain. There are two
nuclei for the oculomotor nerve:

The oculomotor nucleus originates at the level of the superior colliculus. The
muscles it controls are the ciliary muscle (affecting accommodation ), and all
extraocular muscles except for the superior oblique muscle and the lateral rectus
muscle.
The Edinger-Westphal nucleus supplies parasympathetic fibres to the eye via the
ciliary ganglion, and thus controls pupil constriction. When these fibres are
damaged, the delicate sympathetic- parasympathetic balance which maintains our
pupillary size is upset and unopposed sympathetic drive causes a dilated pupil.

47) on STUMP APPENDICITES
Dear Yin Ling,

A male patient comes with fever, loss of appetite and RIF pain. He has past history of
appendicectomy 3 years ago. On examination there is tenderness and mild guarding
in the RIF.

What are your thoughts?
Do you know appendicitis can re occur even after appendectomy ?

Stump appendicitis, although rare, is a real entity that is often not considered during
the evaluation of patients with right lower quadrant pain and a surgical history of
appendectomy. The history of appendectomy may delay the diagnosis and
management of this entity by misleading the physician into thinking that this patient
could never have appendicitis again. However, the diagnosis of appendicitis should
be considered in any patient with right lower quadrant pain, even if there is a
history of appendectomy.

Stump appendicitis is the re-inflammation of any residual appendiceal tissue after an
appendectomy. Since the initial reports of stump appendicitis, it remains a rare
condition; it occurs when there has been an incomplete appendectomy . Complete
removal of the appendix is essential in preventing the occurrence of this condition.
Many reports have suggested that stump appendicitis results from a stump that is
left too long. It has been suggested that with the widespread use of laparoscopic
technique, there might be an increase in the incidence of stump appendicitis, but
stump appendicitis has been reported to occur after either laparoscopic or open
appendectomy. Therefore, the relationship between laparoscopic appendectomy
and stump appendicitis is not proven.

One disadvantage of laparoscopic appendectomy is the inability to feel structures


and judge pathology by palpation. Difficulty in retracting the long appendix may be
responsible for the long appendiceal stump that may occur with laparoscopic
appendectomy. The incidence of stump appendicitis is also probably
underestimated and under-reported.

48) on Graves disease
Dear Yin Ling,
Grave's disease is defined by the three features of Eye involvement, Acropachy and
occasionally Pre tibial myxodema on top of the Hyperthyroidism. Sometimes the eye
involvement precedes the hyperthyroidism. They present as double vision,
chemosis, protrusion, etc. To help you remember, pls recall
NO SPECS is the mnemonic used for the clinical progression of Graves'
opthalmopathy.

N: No signs or symptoms
O: Only signs i.e. lid lag and lid retraction
S: Soft tissue involvement (conjunctivitis, chemosis, and periorbital edema)
P: Proptosis
E: Extraocular muscles involved, commonly inferior rectus resulting in diplopia when
looking up
C: Corneal involvement (drying and ulceration due to the inability to close the eye
lids)
S: Sight loss.

49) on Renal Failure


Dear Yin Ling,

are you aware that the humble Starfruit is a killer!?

Star fruit, belonging to the Oxalidaceae family, species Averrhoa carambola, is a
popular fruit among Orientals. There have been reports of hiccup, confusion, and
occasional fatal outcomes in uraemic patients after ingestion of star fruit.

An excitatory neurotoxin from star fruit has been implicated although the exact
nature of this toxic substance has not been identified. Renal failure patients can
have symptoms including hiccup, confusion, vomiting, impaired consciousness,
muscle twitching and hyperkalaemia shortly after ingestion of star fruit.

Symptoms of most patients can be resolved with dialysis or spontaneously. The
close temporal relationship of ingestion of star fruit and onset of symptoms strongly
suggests the existence of a causal relationship between the two.

It is recommended that uraemic patients should totally abstain from star fruit due to
these potentially fatal complications.

50) on Paralysis
Dear Yin Ling,
a patient presenting well and the after a while suddenly plops down in severe
weakness and lethargy is dramatic and frightening. A condition that we suspect is
Hypokalaemia.
The hypokalemic paralysis include a group of disorders with differing etiologies but a
common presentation.
Familial hypokalemic paralysis (FHP) is a genetic channelopathy, with an autosomal
dominant pattern of inheritance. Thankfully not common.
Thyrotoxic hypokalemic periodic paralysis (THPP) is an acquired disorder that can
present in a very similar manner to FHP when clinical features of hyperthyroidism
are sub-clinical, as they commonly are. Treatment of hyperthyroidism prevents
THPP, though the degree of hyperthyroidism does not predict the degree of
paralysis.
Patients often have no other symptoms of hyperthyroidism during attacks.

Thankfully the patients I had seen all had obvious Grave's disease so it was not too
difficult to diagnose.
Thyrotoxic hypokalemic periodic paralysis is likely due to excessive thyroid-
hormone-induced adrenergic stimulation of the sodium-potassium (Na+-K+) pump,
which drives potassium into cells.
Potassium levels are normal between attacks, and there is no decrease in total body
potassium. Attacks of paralysis are classically precipitated by an insulin surge after a
high carbohydrate meal, or increased adrenergic activity with physical exertion.
However, other physiologic stressors such as sepsis, trauma, hypothermia, menses,
and emotional stress are known triggers of THPP.
Episodes are acute in onset and often occur in the nighttime or morning following a
day of strenuous exercise. Patients may also describe a prodrome of muscle aches or
cramps in the days preceding an acute episode.
The patient may not have symptoms of overt hyperthyroidism, so thyroid function
tests should be checked in all patients with hypokalemic paralysis. The EKG will
demonstrate findings typical of hypokalemia.
The hypokalemia of THPP affects the striated/skeletal muscles, not the nerves or the
smooth muscle. Proximal muscles and those of the lower extremities are more
severely affected, usually, but not always, in a symmetrical pattern; patients may
have either hemiplegia or paraplegia. Muscles of respiration are typically spared, but
cases of respiratory failure have been reported. Mental status and sensation are
unaffected.
In patients with THPP, hypokalemia is often exacerbated by associated
hypophosphatemia and mild hypomagnesemia. Transport of phosphorus across cell
membranes occurs with that of potassium.
Thyrotoxic hypokalemic periodic paralysis is 20 to 40 times more common in
hyperthyroid males than in hyperthyroid females, and occurs in up to 15% of
hyperthyroid Asian males. All the patients I had seen were Males.
Administering potassium during an attack can rapidly abort the episode. However,
this should be done cautiously given the transient nature of the transcellular shift, as
there is a risk of post-treatment hyperkalemia in up to 40% of patients. Cardiac
arrest due to rebound hyperkalemia has been reported. Furthermore, spontaneous
recovery of flaccid paralysis suggests that potassium can shift back out of the cell
without supplementation.
Nonselective beta-blockers such as propanolol is used to inhibit adrenergic
stimulation of the Na+-K+ pump. (Remember that Hyperkaleamia is a
contraindication for the use of propanolol!) With adequate suppression of
adrenergic activity, as indicated by reduced heart rate, increases in serum potassium

and rapid improvement of paralysis can be seen. This may eliminate the need for
supplemental potassium. Patients with more severe or rapidly progressive paralysis,
or with cardiac conduction abnormalities consistent with hypokalemia, should be
treated with potassium.
Prevention of acute episodes depends primarily on maintenance of a euthyroid
state. Nonselective beta-blockers are also effective. Potassium supplements can be
prescribed to be taken in case patients have an acute recurrent episode despite
these measures.
However, because potassium levels are normal between attacks, there is no role for
routine potassium supplementation or potassium-sparing diuretics.
Management of THPP should include supportive care, with particular concern for
respiratory status, and monitoring of serum electrolytes. Serial measurement of the
FVC can give early warning that the patient may need respiratory support.
Characteristics which would favor FHP over THPP include a family history of
hypokalemic paralysis, Caucasian ethnicity, and female sex.
Differentiation between FHP and THPP is critical because treatment differs. Familial
hypokalemic paralysis requires a more vigorous potassium repletion, while THPP
may be treated more conservatively, sometimes solely with observation or
propranolol.

51) on Associations
Dear Yin Ling,

Brainstem answers now!

When seeing nausea + tiredness+ hyperpigmentation =?

addisonian crisis

Weight loss + Amenorrhea + falling hair=?

thyrotoxicosis

Syncope + angina+dyspnoea = ?

SAD is the symptoms of what?

this is a classic triad!!!



aortic stenosis

weight loss + malaise + fever + pruritus=??

lymphoma

and the name of the fever????

pel ebstein fever

Cant see, cant pee, cant climb a tree=??

reiter's disease

abd pain + jaundice + fever=???

ascending cholangitis

5 causes of Fever, Chills and Rigours???

lobar pneumonia, abscess, pyelonephritis, malaria, ascending cholangitis
others include Septicaemia, Influenza, Dengue

Give me 5 causes of ESR >100

TB, Multiple myeloma, PMR, CTD,
ADvance Malignancies esp with Paraneoplastic syndromes
Septicaemia

52) on Peripheral Arterial Disease
Dear Yin Ling,

Significant proportions of patients do not have classical features of intermittent
claudication and therefore PAD is an under-diagnosed condition. My students love
to ask about calf pain and pain before how many "blocks", this as you know is utterly

unreliable!

Clinical examination is important to decide which patients should undergo further
assessment, and palpation of the peripheral pulses is important. Any palpable pulse
abnormality (absent or reduced femoral, popliteal, dorsalis pedis, or posterior tibial
arteries) increases the likelihood of PAD and the absence of any palpable pulse
abnormality decreases the likelihood of PAD.
However, assessing patency of the popliteal artery is not always possible by
palpation. This is due to a combination of deep placement of the artery in the
popliteal fossa and the presence of considerable quantity of fat both behind the
artery and in the superficial tissues.

One technique tested in OSCEs is using the flexion of the knees to facilitate
relaxation of the gastrocnemius making it more accessible to palpation. In a patient
with patent popliteal artery, the to and fro pulsatile movement of the foot (in
conjunction with heart beat) will be observed when he sits in a high chair with legs
crossed such that the popliteal fossa of the leg being examined lies over the knee of
the opposite leg. The patient must sit on a high chair so that the popliteal fossa lies
against the knee cap of the opposite leg. The top leg should be completely relaxed
so the weight of the leg compresses the artery between the knee cap of the lower
leg and the proximal part of the tibia

This is called the "Cross leg test" or "Fuschig's test"


53) On Hypothyroidism and vague symptoms
Dear yin ling,
at every bedside class, I beg my students to always think of Hypothyroidism across a
wide range of complaints from ACS to deafness to loss of memory to pruritus.

Pls always think of it too in patients with muscle weakness or ENLARGED muscles!!

Hypothyroid myopathy typically manifests as polymyositis-like myopathy with
proximal muscle weakness and an increased creatine kinase level. However, it
sometimes manifests as muscle enlargement (pseudohypertrophy); in adults, this
condition is called Hoffman syndrome. In children with hypothyroid disease
(cretinism), a pattern of proximal weakness and diffuse muscle enlargement is also
seen. Pls also note that hypothyroidism is thought to predispose individuals to
rhabdomyolysis too.

There is an inverse correlation between thyroid function and CK levels: in
hypothyroid patients, CK levels were elevated, while in hyperthyroid patients, CK
levels were lower than normal. After treatment to restore normal thyroid function,
CK levels returned to normal.

Three out of four patients with hyperthyroidism experienced muscle cramps during
treatment to rapidly reduce thyroid function to normal levels.

Serum creatinine kinase is decreased in thyrotoxicosis but increased in
hypothyroidism. Prof Khalid has a publication on the subject in 1998 when he
published about these subtle changes seen in subclinical hyper and hypothyroidism
and hence questioned whether we should treat low TSH or high TSH when T4 or T3
are in the normal range.
The Sign of Hertoghe or Queen Anne's sign is a thinning or loss of the outer third of
the eyebrows, and is a sign of hypothyroidism.

Professor Khalid Kadir

53) on MEN
Dear Yin Ling,

Please do NOT proceed if you are under 21.

I always had difficulty remembering MEN 1 and 2. Luckily I am not the one sitting for
exams! This was until I was taught this!!

There are 2 kinds of MEN in this world

MEN Type 1, are straight guys, and they only think about Pussy, Pussy, Pussy!
So they have malignancies of the

Pancreatic
Parathyroid
Pituitary.

MEN Type 2, are gay guys, and all they think about is Anus, Penis, and Testicles!
So they have malignancies of the

Adrenals
Parathyroid
Thyroid.
And there you have it, both types of Multiple Endocrine Neoplasia... I mean, both
types of MEN!

54) on Coarctation
Dear Yin Ling,

What is Dock's sign?

Bilateral rib notching seen in CXR in Coarctation of the Aorta is known as Dock's sign,
corresponding to the collateral circulation of the internal mammary arteries typically
at ribs 3-8.
Inferior Rib Notching is usually due to enlargement of the neurovascular bundle.
Consider the following main causes:

Coarctation of the Aorta
Fallot's tetralogy - unilateral left following Blalock-Taussig shunt (left subclavian
artery anastamosed to left pulmonary artery - collateral circulation via ribs to supply
arm)
Neurofibromatosis (more classically wavy 'ribbon ribs')
Longstanding SVC obstruction (venous collaterals)

Coarctation of the Aorta is classified into 2 types
Localised (Adult) Most common type. Short narrowing close to ligamentum
arteriosum. Cardiac anomalies uncommon.
Tubular hypoplasia (Infantile). Long segment stenosis. Cardiac anomalies common.

55) on Pulmonary Embolism


Dear Yin Ling,

undiagnosed PE has a hospital mortality rate as high as 30%, which falls to about 8%
if PE is diagnosed and treated appropriately

The diagnosis of PE remains one of the most difficult problems confronting
clinicians. PE is considered in the differential diagnosis of many clinical
presentations, including chest pain, hemoptysis, and dyspnea. Yet less than 35% of
patients suspected of having PE actually have PE

In patients with Pulmonary embolism, pleuritic chest pain and haemoptysis indicates
that Pulmonary Infarction had occurred.

Only about 1 in 10 patients with Pulmonary embolism develop into pulmonary
infarction!

Sudden onset of dyspnoea, vague chest pain, light headedness are the most
common symptoms of PE. The majority of patients have no signs other than
tachypnoea and tachycardia, while extensive embolism will result in cyanosis,
increased JVP, hypotension.

Pulmonary infarction will result in haemoptysis, Pleural rub, FEVER, dullness and
crepts over the infarcted area and sometimes pleural effusion.

Ple remember that recurrent small PE is a cause of PUO.

Hemodynamic decompensation occurs not only because of physical obstruction of
blood flow but also because of the release of humoral factors, such as serotonin
from platelets, thrombin from plasma, and histamine from tissue.

Acute PE increases pulmonary vascular resistance, partly attributable to hypoxic
vasoconstriction. In patients without prior cardiopulmonary disease, the mean
pulmonary artery pressure can double to approximately 40 mm Hg.

Pls note that No ABG data had sufficient negative predictive value. A normal ABG
does NOT exclude PE. Hypoxaemia seems to be a reliable sign of pulmonary
embolism, a pO2 of under 50 mm Hg was always associated with a severe embolism
with amputation of over 40% of the pulmonary vascular bed. The failure of

supplemental oxygen to correct arterial hypoxemia accompanying acute PE often


reflects the existence of right to left shunting of venous blood through the heart, the
lungs, or both.
Respiratory alkalosis is commonly seen

There is Right-to-left shunting: no ventilation and venous blood enters systemic
circulation. Increased anatomic dead space: breathed gas does not enter gas
exchange units of the lung

A neg D Dimer serves only to exclude and a positive result cannot confirm.

Wells' Two-level PE Score helps in decision making



Hamptons hump long arrow: the wedge-shaped opacity at the peripheral left lung
field
Pallas sign short arrow: enlargement of the Right descending pulmonary artery


Westermarks sign: the area of hypoperfusion distal to the right pulmonary
vasculature. This represents a probable pulmonary embolism.

56) on Red Meat
Dear Yin Ling,

"The gouty arthritis patient was advised NOT to eat red meat"
'Why?' I ask? 'WHY???'

Why some meats not just pork or chicken are lighter or darker is a fairly
complicated subject. Several factors contribute to the colour of meat. Why is your
stirred fried horr funn's beef dark in colour ie red meat?
Haemoglobin, is absorbed in muscle, contributes a bit of red colour. Muscles that
are heavily used may not be able to get enough oxygen from the blood, and must
resort to oxygen stored in myoglobin molecules.
Both haemoglobin and myoglobin are red when carrying oxygen. Depending on how

much use the muscle sees, there is more need of the oxygen stored in myoglobin,
and will be darker as a result.

Turkeys for example, which stand around a lot and hardly ever fly, have dark leg
meat but breasts that are white.
Game animals, which tend to use all their muscles, are essentially all dark meat,
while domesticated animals generally have a mix of both light and dark.

In terms of cooking, dark meat generally has more flavour, but, because those
muscles were more actively exercised, tends to be tougher. Lighter meats tend to be
more tender but have less flavour. Now you know why some meats are tender and
some tough.

Diets which are high in purines and high in protein have long been incriminated of
causing an increased risk of gout. High uric acid levels associated with gout derive
largely from foods rich in protein and purine, which produce uric acid as a waste
product

People who consumed the highest amount of meat were 40 percent more likely to
have gout than those who ate the least amount of meat.
People who ate the most seafood were 50 percent more likely to have gout.

Obesity can be linked to high uric acid levels in the blood. People who are
overweight should go on a reasonable weight-loss program. Fasting or severe
dieting can actually raise uric acid levels and cause gout to worsen.
Usually people can eat what they like within limits.

People who have kidney stones due to uric acid, gouty tophi, chronic tophaceous
gout may need to actually eliminate purine-rich foods from their diet because those
foods can raise their uric acid level.
Consuming coffee and tea is not a problem but alcohol can raise uric acid levels and
provoke an episode of gout. Drinking at least 10-12 eight-ounce glasses of non-
alcoholic fluids every day is recommended, especially for people with kidney stones.

57) On Prednisolone
Dear Yin Ling,
Prednisolone can have unique side effects well known to us all.
Among the most memorable ones are

Pancreatitis
Avascular necrosis of the head of the femur
Posterior sub capsular cataract
Extensive acne
Intracranial HPT
Papillodema
Psychosis
Glaucoma

I remember these well because Prof Florence Wang taught me this. She was running
the Lupus clinic in UH and used a lot of prednisolone.
Any patient who is receiving 40 mg or more of prednisiolone for more than 7 days or
who is taking 20 mg or more of prednisolone for more than 14 days is classed as
immunocompromised. Patients receiving lower doses of prednisolone (or equivalent
doses of other corticosteroids) are not considered to be immunocompromised
unless they are taking additional immunosuppressive medication.
Remember "yi fatt" 28o mgs of Prednisolone continuously and you will FATT.
The is key to understanding why we keep our courses of Prednisolone only 5 days in
asthma and Acute exacerbation of COPD for eg.

58) On FT3
Dear Yin Ling,

What is this T3 that we see in our thyroid function tests?
How do we interprete it?
When is it useful?


TRI-IODOTHYRONINE (T3) There are two assays available, one measures total T3 and
the other free T3. The total T3 comprises of both protein bound and free T3. The
free component is the active form and comprises a mere 0.3% of total circulating T3.

T3 is the biologically active thyroid hormone, possessing 5 times the metabolic
power of T4. In man some 80% of T3 is produced from T4 by conversion in liver and
kidney. Therefore little is produced in the thyroid itself.

The conversion of T4 to T3 can depend on a number of situations such as chronic


illness or surgical stress which cause a fall in T4 to T3 conversion (called low T3
syndrome). Hence FT4 may be normal and the patient is hypothyroid!

Starvation also alters T4 to T3 conversion with a fall in T3 as the body tries to reduce
its metabolism to conserve energy. This is survival adaptation.
Measurement of fT3 in patients with suspected hyperthyroidism is rarely indicated.
This is reserved for situations where hyperthyroidism is suspected clinically and TSH
is suppressed, but the fT4 is not elevated. fT3 if elevated tells us that T3
thyrotoxicosis is present.

Measurement of fT3 is not indicated in hypothyroidism.
In hyperthyroidism, both thyroxine T4 and T3 levels (total and free) are usually
elevated. Treatment and monitoring can be done with FT4 but if FT4 is normalised
with treatment and patients still appear hyperthyroid then FT3 should be measured.
Pls note that the suppressed TSH takes time to recover and hence is not helpful
initially.

FT3 levels may be required to evaluate clinically euthyroid patients who have an
altered distribution of binding proteins (eg, pregnancy, dysalbuminemia).
Btw pls do not diagnose subclinical thyroid dysfunction unless you are sure that the
following is fulfilled.

The following five criteria define endogenous subclinical thyroid dysfunction:

a. TSH increased above, or decreased below designated limits
b. Normal free T4 concentration and free T3
c. The abnormality is not due to medication
d. There is no concurrent critical illness or pituitary dysfunction.
e. A sustained abnormality is demonstrated over 3-6 months.
Therefore in summary

Measurement of serum T3 is indicated, as follows:

a. In suspected hyperthyroidism with suppressed TSH and normal serum T4, to
identify T3-thyrotoxicosis and distinguish this entity from subclinical thyrotoxicosis.
b. During antithyroid drug therapy to identify persistent T3 excess, despite normal

or low serum T4 values.


c. For diagnosis of amiodarone-induced hyperthyroidism, which should not be based
on T4 excess alone because of the frequency of euthyroid hyperthyroxinemia during
amiodarone treatment.
d. To assess the extent of T3 excess in individuals treated with thyroid extracts of
animal origin, our sinsehs uae monkey thyroid dried and grounded! And to assess a
potentially damaging hormone excess that is not reflected by the level of T4.
e. To identify T3-predominant thyrotoxicosis, an entity that is less likely to achieve
remission.
f. To detect early recurrence of hyperthyroidism after cessation of antithyroid drug
therapy.
h. To establish the extent of T3 excess during high-dose replacement or suppressive
therapy with T4, or after an accidental or intentional T4 overdose.

Low serum T3 concentrations have little specificity or sensitivity for the diagnosis of
hypothyroidism. Many patients with nonthyroidal illness have low values, and the
serum T3 concentration can remain in the reference range until hypothyroidism is
severe.

59) on Calcifications in CXR


Dear Yin Ling,

This CXR shows innumerable small, punctate calcifications throughout both lungs.
No difficulty for you. What are the differential diagnosis?



Calcification in a pulmonary nodule (PN) on imaging indicates a high probability that
the lesion is benign. But not all calcified PN are benign and the differential
considerations include a primary central lung carcinoid, metastasis and a primary
bronchogenic carcinoma.

Radiological demonstration of calcification in lung cancers is uncommon but when
encountered may lead to misdiagnosis. Amorphous, punctate, and reticular patterns
of calcification have been described in lung cancer. Malignant tumors may engulf a
pre-existing granuloma.

The prevalence of calcified lung cancers identified on conventional chest


radiographs is said to be 1%. Malignant nodules may mimic the appearances of
benign calcified granulomas; typical examples are metastases from osteogenic
sarcoma or chondrosarcoma.

Differential diagnosis of diffusely distributed small calcified nodules includes
infections, lung metastases, chronic pulmonary hemorrhage, pneumoconiosis,
deposition diseases and idiopathic disorders such as pulmonary alveolar
microlithiasis.

Post Chicken pox pneumonia
Chicken pox pneumonia may cause tiny widespread micronodular calcification with
nodules 1-3 mm in diameter as a late sequela. There is no associated calcification of
mediastinal lymph nodes.

TB
Histoplasmosis
Dystrophic calcification follows caseation, necrosis or fibrosis. Calcified nodules
following infections are well defined and often measure 2-5 mm in diameter. Such
nodules often follow healed disseminated histoplasmosis and rarely may follow
healed miliary tuberculosis. When secondary to tuberculosis or histoplasmosis,
there is generally associated calcified hilar or mediastinal lymph nodes.

Occupational Lung disease
Patients with silicosis and coal miner's pneumoconiosis often develop small (<5 mm)
diffuse lung parenchymal calcified nodules, often associated with egg-shell
calcification of hilar or mediastinal lymph nodes.

Microlithiasis alveolarum aka Pulmonary alveolar microlithiasis is a rare idiopathic
lung disorder characterized by the intra-alveolar accumulation of microliths of
calcium phosphate. Most are an incidental finding on conventional chest
radiographs and seen as innumerable tiny calcific densities.

60) on Sputum
My dear Yin Ling,
Now tell me what has wasabe got to do with Respiratory medicine!?
Why does sputum turn Green and what does this indicate!?
What is the cause of the green colour of pus or nasal mucus?

Green pus, or green nasal mucus, is caused by iron-containing myelo-peroxidases


and other oxidases and peroxidases used by polymorphonuclear (PMN) granulocytes
(neutrophils).

These short-lived phagocytising leucocytes avidly ingest all sorts of bacteria and
inactivate them by oxidative processes, involving the iron containing enzymes. The
resulting breakdown product, comprising dead PMNs, digested bacteria and used
enzymes, pus, contains significant amounts of iron, which gives it its greenish colour.

At the start of a URTI
Nasal mucus produced at the beginning is clear and is produced in response to
tissue damage caused by the invading rhinovirus. It only turns green a few days into
the infection as neutrophils respond to clear away the cellular debris and secondary
bacterial infection sets in.

Polymorphonuclear leucocytes are equipped with a number of enzymes, the most
potent of which is peroxidase.

This same peroxidase is also found in horseradish, giving it a distinctive green color
and a sharp bite, as anyone who has tried Japanese wasabi paste can confirm.

When sputum turns green, sputum has stagnated inside the respiratory system long
enough for the myeloperoxidase to act. It also implies bacterial superinfection.

Wasabe anyone!?

61) on Iron studies
Dear Yin Ling,
My students are frequently confused by iron studies when I ask them to interprete it
as part of the work up for anaemia.
Serum Iron
Iron contained in blood serum is normally bound to the protein transferrin. It cannot
be floating around by itself! Iron is toxic!
Each molecule of transferrin can transport two molecules of iron to areas of the
body that need this element. Think of it as a small lorry, the pasar malam type.

Most of the bodys iron (about 60%) is contained in hemoglobin. Another 30% is
stored in ferritin, and a few percent in myoglobin. When body iron stores increase
above these relatively normal ratios, proportionally greater amounts of iron are
stored in ferritin or a complex called hemosiderin.
Generally men have higher levels of serum iron than women. When laboratories test
for Se Iron, they are testing iron contained in plasma that is generally bound to
transferrin.
In most people, ONLY about 25 35% of the transferrin contained in the serum is
used to bind iron for transport. SO THERE ARE SPARE LORRIES. When laboratories
measure serum iron they also measure transferrin and calculate the percentage of
transferrin molecules that are used to bind iron.
Total Iron Binding Capacity (TIBC) and Transferrin Saturation % (TS%)
Total iron binding capacity: This measurement indicates the TOTAL potential
capacity of transferrin molecules to bind with serum iron, its telling you how much
load your entire fleet of lorries can carry.
When TIBC is at or below the low end of a laboratory range, it is an indication that
there is limited capacity for transferrin molecules to accept additional iron. If that
occurs in combination with a relatively high measure of serum iron, it is likely that
the ability of transferrin to safely bind serum iron is impaired. Your toxic products
are going to spill on the road!
Iron in the plasma that is not bound to transferrin is called non-transferrin bound
iron (NTBI). This is a potentially toxic form of iron that can damage body systems.
Generally, when 40% or less of transferrin molecules are used, iron is considered
safely bound. Much above that, transferrin becomes saturated and it binding
capacity drops to a point where it will no longer can efficiently harbor NTBI. Some of
the iron will then bind to other molecules that does not have transferrins ability to
protect you. This causes oxidative stress, a process that if not countered by the
bodys antioxidant defenses, will over time result in cell, tissue and DNA damage.
Transferrin saturation percentage (TS %) is calculated by dividing serum iron by TIBC,
then multiplying by 100. The resulting number is referred to as transferrin saturation
percentage (TS %). In people with undiagnosed hemochromatosis, this number is
often above 50%, and sometimes even as high as 100%. The normal range of TS % is
generally between 2535%. When the percentage is calculated to be less than 17%
or higher than 45%, a condition of either iron deficiency or iron overload is possible.
Either too little goods or too much goods for your lorries! In either case, further
investigation is warranted including ferritin testing. Very low or very high ferritin in
combination with low or high TS % can help a physician confirm a diagnosis of either
iron deficiency or iron overload.

Serum Ferritin (SF)


Ferritin is a protein that is mainly utilized to store iron for future use. The body
requires iron to make hemoglobin for blood and myoglobin for muscles. Iron in
excess of daily needs is stored in ferritin molecules, huge storehouses which hold up
to 4,500 iron atoms each!
Normally, dietary intake offsets daily loss iron loss (only about 1 to 1.5 milligrams
per day). Therefore, ONLY A MINISCURE one gram of storage iron (1,000 milligrams)
is adequate to meet all foreseeable needs. The body routinely loose iron as a result
of trauma, blood loss or through menstruation.


However, more than one gram of storage iron can stress the bodys ability to
provide a safe harbor for this potentially toxic metal. With a few exceptions,
including events of inflammation (ferritin RISES IN INFLAMMATION eg think of
Dengue crisis) or anemia of chronic disease, a blood test measuring SF can provide
an accurate surrogate measure of iron stored in the body.
Only a very small fraction of the bodys stored iron is actually stored in transferrin or
ferritin molecules circulating in the bloodstream. However, in healthy individuals,
the relative amount of ferritin found in serum is an accurate surrogate measure for
iron stored in body organs.
Ferritin can be elevated even when both serum iron and transferrin saturation
percentages are at low-normal levels or below.

High ferritin under these circumstances might not signal iron overload, but can
result from a defense mechanism, an acute phase reaction. This is seen in anemia of
chronic disease, or inflammatory anemia.

62) on Renal function
Dear yin ling,
elevations in levels of blood urea and/or serum creatinine do not necessarily
indicate structural renal disease.

Conversely, blood urea or serum creatinine values, which appear to be within
normal, do not by themselves rule out significant reduction in glomerular filtration
rate.

Any interpretation of the blood levels of these two substances must be done with
the awareness that a variety of extrarenal factors can affect them.

What is it which affects these 2 common biochem parameters ?
Urea is the final product of protein catabolism. The ammonia formed in this process
is synthesized to urea in the liver. This is the most important catabolic pathway for
eliminating excess nitrogen in the human body. Increased blood urea may be due to
prerenal causes eg cardiac failure, dehydration, increased protein catabolism, and
high protein diet, renal causes and postrenal causes eg obstruction of the urinary
tract from stones, enlarged prostate gland, tumours.
The rate of urea production is not constant. This is its major disadvantage when
used to monitor renal function. It is elevated in those who consume a diet fairly high
in protein and in conditions characterized by enhanced tissue breakdown (eg,
hemorrhage, trauma, glucocorticoid therapy, chemotherapy). Certain antibiotics,
such as tetracyclines, may interfere with protein synthesis and tend to be catabolic,
thereby also increasing BU levels.

On the other hand, a low-protein diet or liver disease can decrease the BU level
without affecting GFR or renal function. My consultant used to ask me about liver
disease and BU levels. Liver disease may be associated with near-normal values of
both BU (due to decreased urea production) and serum creatinine (due to muscle
wasting), despite a significant decline in renal function manifested by decreased
GFR.

Approximately 40%-50% of the filtered urea undergoes passive reabsorption in the


proximal tubule. In states of dehydration and intravascular volume depletion,
proximal sodium and water reabsorption increases, coupled with a parallel increase
in the reabsorption of urea. This results in a disproportionate rise in BUN levels
relative to any change in serum creatinine levels. This is the beauty of understanding
physiology and applying it to clinical medicine.
The blood urea test is a measure of the amount of nitrogen in the blood that comes
from urea. That's why its also called bld urea nitrogen.

The most common cause of an elevated BU, azotemia, is poor kidney function,
although a serum creatinine level is a more specific measure of renal function.
Elevated BU in the setting of a relatively normal creatinine may reflect a
physiological response to a relative decrease of blood flow to the kidney (as seen in
heart failure or dehydration) without indicating any injury to the kidney.

However, an isolated elevation of BU may also reflect excessive formation of urea
without any compromise to the kidneys. Increased production of urea is seen in
cases of moderate or heavy bleeding in the upper gastrointestinal tract. This is
something i repeatedly tell my students. The nitrogenous compounds from the
blood are reabsorbed as they pass through the GI tract and then broken down to
urea by the liver. Enhanced metabolism of proteins will also increase urea
production, as may be seen with high protein diets after auntie and i force feed you,
steroid use, burns, or fever.

My consultant Dr Mrs Kula used to ask us abt causes of low BU!
This include liver problems, malnutrition (insufficient dietary protein), or excessive
alcohol consumption. Overhydration from intravenous fluids can result in a low BU
too. Physiological changes in renal blood flow in pregnancy will also lower BU.

You may be surprised to know that Urea itself is not toxic. BU is a marker for other
nitrogenous waste. Thus, when renal failure leads to a buildup of urea and other
nitrogenous wastes, what we label as uremia is a surrogate marker!

Prolonged periods of severe uremia may result in the skin taking on a grey
discolouration or even "uremic frost" on the skin.Because multiple variables can
interfere with the interpretation of a BU value, GFR and creatinine clearance are
more accurate markers of kidney function. Age, sex, and weight will alter the
"normal" range for each individual, including race. In renal failure or chronic kidney
disease, BU will only be elevated outside "normal" when more than 60% of kidney

cells are no longer functioning. This is a wee bit TOO LATE! Hence, more accurate
measures of renal function are generally preferred to assess the clearance for
purposes of medication dosing. The eGFR available to us nowadays makes life easier
but pls remember that this is an estimate.


Dr Mrs Kula who more than anyone else taught me what it means to be a
Compassionate Physician.

63) on IV fluids
Dear Yin Ling,
These coming questions are Crucial for all Houseofficers.

why is it that we infuse Dextrose 5% and not give Glucose 5%?
What is the difference if any?
"Glucose represents the two forms sugars, isomers, that mirror each other, d-
glucose and l-glucose. Only d-glucose is biologically active and is chemically known
as dextrose monohydrate."
Dear Yin Ling,
What is Hartman's and when do you use it?

Hartmanns solution is a super yummy potpourri needed for electrolyte losses
* It contains potassium and calcium in concentrations that approximate the free
(ionized) concentrations in plasma.

* The addition of these cations requires a reduction in sodium concentration for


electrical neutrality, so Hartmanns has a lower sodium concentration than either
isotonic saline or plasma.
* The addition of lactate (28 mEq/l) similarly requires a reduction in chloride
concentration.

Lactated Ringer solution has an electrolyte composition which mimics the ionic
concentrations of calcium, potassium, and chloride in plasma more closely than
isotonic saline. This is the Advantage.

Why Lactate?

The added lactate is converted to bicarbonate by the liver. The intention was for the
bicarbonate to act as a buffer in patients with metabolic acidosis; however, this
effect is not a clinically significant one. Pls remember that the added calcium may
act as a binder for many drugs and impair both bioavailability and efficacy.
In contrast, Normal (isotonic) saline 0.9% N/S
* It is a solution of 0.9% sodium chloride in water, it contains EQUAL quantities of
sodium and chloride (154 mEq/l). The plasma is NOT like that, our Cl is much2 lower!
* It has a much higher concentration of chloride compared to plasma and infusion of
large volumes of isotonic saline can produce a metabolic acidosis.

So be careful when you tell me that N/S is given a lot. Now do you see clearer why
you need to give some "free water" too!

Disorders of sodium concentration are nearly always caused by excess free water
(hyponatraemia) or free water loss (hypernatraemia).
crystalloids will distribute between the Intravascular space and the interstitial space.
Hence, patients undergoing large volume fluid resuscitation are at high risk for
developing pulmonary and generalized interstitial edema, particularly with a
preexisting state of fluid over- load (e.g., congestive heart failure and renal failure).
Now you see why we cannot just give more!
We discussed "free water", while IV dextrose solutions are beneficial in the
management of glucose-deficient states, eg fasting pre and post operations, it is the
least potent volume expander of the crystalloid solutions for once the dextrose is
rapidly taken up, Free Water is now available which will diffuse into cells. For
rehydration it is useful.

Dextrose 5% is hence a Maintenance Fluid. Its not good for fluid expansion but is
good to supply water. If your patient is losing water because of fever, tachypnoea,
etc, water needs to be supplied!

Dextrose 5% contains 5g of dextrose (D-glucose) per 100ml of water. This glucose is
rapidly metabolized and the remaining free water distributes rapidly and evenly
throughout the bodys fluid compartments. So shortly after iv administration of
1000ml 5% dextrose, 670ml water will be added to ICF and 330ml water to ECF.
Dextrose 5% is an isotonic solution.

The net effect is of administering pure water, so it is distributed throughout the total
body water.
Be cautious though that we just cannot keep on giving crystalloids. One of the
strongest arguments against the use of crystalloid fluids has been that they cause a
dilutional hypoalbuminemia that puts patients at increased risk for developing
pulmonary edema. Watch out for those basal crepts as I repeatedly told you all
kiddies!!!
Now you all know why I get very upset when my students do not auscultate the
lungs because its "an abdominal problem ma!"

'Mah your head' I would scream!!
Because the osmolarity of normal saline matches that of the serum, it is an excellent
fluid for volume replacement. Hence in hypoVOLAEMIC states, this is the fluid of
choice.

Hypotonic fluids such as D5W should never be used to replace volume. They are
the water trucks in this dry and hot season.
Lactated Ringers solution is commonly used for surgical or trauma patients
because of electrolyte losses; however, only NS can be given in the same line with
blood components. Calcium in IV fluids means an exclusive line not to mix with other
things!
Now Dextrose 5% in N/S aka Dextrose saline is not as popular now as before. Yin
Ling is unfamiliar with it. Times have changed. It was commonly used when I was a
young doctor. It still has a role despite being a hypertonic fluid and can be used as
maintenance fluid for suspected meningitis, acute neurological conditions where
you do NOT want the hypotonic free water of D5% once the dextrose is taken up to
precipitate cerebral odema,

and in gastroenteritis with sodium losses or when the serum sodium is low.
The Daily maintenance fluid requirements vary between individuals based on weight
and sex.
Eg in a 70 Kg male = 2.5 - 3.0L water is needed, and only 120 140 mmol sodium to
replace loss in sweat and urine. Our bodies are super efficient salt conserving
machines. One litre or 2 "pints in local colloquial" of N/S will provide 154mmols of
sodium, more than enough for basal needs of replacement!

Hence, a regime for eg of 2L 5% Dextrose + 1L Normal saline will provide 3L water
and 154 mmol sodium.
This is what Yin Ling tells me she prescribes often for maintenance IV regimes in
fasting patients.

But how much calories did she give?

Know that 5% dextrose means the solution contains 5g/100ml of solution. Hence 1
litre has 50 grams which translates to 50 x 4 calories which is 200 calories. 2 l of D5%
a day will ONLY provide 400 calories. What does this mean? I hate all these numbers
as it is like saying "put 30 grams of salt in the soup" in cookbooks! @@

A bowl of rice is 200 calories! This is equal to 2 bowls of PLAIN rice per day. No
wonder your patient is losing weight! A plate of nasi lemak with chicken thrown in is
500 calories, hence we are not doing a good job with our patients nutrition.

Now finally Potassium. This is essential too. We need 0.5 to 1 mmol per kg BW ie abt
35 to 70 mmol per day.

1 Gm of KCl is equal to 13.5 mmol of Potassium.
so if we give 1 gm in each infusion in alternate "pints" we will give 3 gms ie 13.5 x 3 =
40mmols per day. Just enough!

Now all the above is for a "NORMAL" person with no additional losses.

Daily fluid requirements increase in illness:
Fever (500 ml/day for every degree above 37oC)
Breathlessness and tachypnoea
Diarrhoea and vomiting
Haemorrhage
Surgical drains, stoma and fistulae

Polyuria
Third space losses (pancreatitis, bowel obstruction, and after laparotomy)
SIRS capillary leak

Electrolyte losses must be factored in as well.
A BUSE is indispensable as an aid.


As a starry eyed Houseofficer.

64) on Bony Secondaries
Dear Yin Ling,
we often see secondaries in bones and commonly they are osteolytic lesions.
Occasionally we see Hyperdense secondaries! What are your thoughts when you see
one? Where do they arise from?

Sclerotic (or blastic) bony metastases can arise from a number of different primary
malignancies, including:
prostate carcinoma : most common
breast carcinoma (may be mixed)
transitional cell carcinoma (TCC)
carcinoid
medulloblastoma
neuroblastoma
mucinous adenocarcinoma of the gastrointestinal tract: , e.g. colon carcinoma
lymphoma

In general, the spine is the most common location of metastatic disease. Metastases
distal to the knee and elbow are extremely uncommon, but approximately 50% of
these acral metastases are secondary to primary lung tumors. Carcinomas, such as
those of the breast and prostate, rarely exhibit such a distinct pattern.

Computed tomography scanning: Most sensitive imaging modality to detect bone
destruction, providing the best assessment of the extent of cortical destruction
Bone scanning: Very sensitive study for the detection of occult lesions and the
assessment of the biologic activity of lesions


Prof Esha Dasgupta: I can't resist to pen down my mnemonic for tumours which
cause bone metastasis. It is PUBLIK T(oilet) ( PUBLIC TOILET) P for prostate,U for
uterus, B for breast and bladder,L for lungs,I for intestine aka colon,K for kidneys
and T for thyroid.

65) on Septicaemia
Dear Yin Ling,
What is Osler's triad?

It is also known as the Austrian triad
It describes the Triad of pneumonia, endocarditis, and meningitis occurring
simultaneously or subsequently in a patient.
The triad was initially reported by the Austrian pathologist Richard Ladislaus Heschl
(1824-1881) in 1862, and later by Osler in 1881.

This is actually a rare combination of Streptococcal septicaemia, with Streptococcus


pneumoniae endocarditis and with Strep pneumonia, and Strep meningitis. Often
associated with alcohol abuse, mortality is extremely high.

Streptococcus pneumoniae is responsible for a remarkable array of disease
processes. Examples of these infections includes pancreatic and liver abscesses,
aortitis, gingival lesions, testicular and tubo-ovarian abscesses, and necrotizing
fasciitis. Currently, multidrug-resistant S. pneumoniae remains susceptible to
vancomycin and several new third-generation fluoroquinolones.

I recall in fear when I remember how we nonchalantly handled agar plates teeming
with Strep pneumonia as second year students doing Microbiology practicals!


66) on Iron supplement
Dear Yin Ling,

Should a person with thalassemia trait avoid iron, such as iron-fortified vitamins?

Iron tablets or iron-supplemented vitamins should be taken only as directed by a
physician to treat PROVEN ACTUAL iron deficiency or to prevent iron deficiency in
high risk circumstances (e.g pregnancy).

People with thalassemia trait (thalassemia minor) are not per se at greater risk of
complications from iron in the diet than anyone else in the general population. So
eat your green veggies, liver and steak. No issue there.

My concern as greatly emphazised in class is misdiagnosing thalassaemia minor as
iron deficiency, or in patients with coincident beta thalassaemia and iron deficiency,
not realising that Hb electrophoresis done then will NOT demo an elevated HbA2
pattern as the iron deficiency will not allow more HbA2 to be formed. Hence in
patients with iron deficiency, the iron deficit must be corrected before the Hb
electrophoresis is ordered.

Can the anemia produced by thalassemia be corrected or improved by taking more
iron?

In the absence of concomitant iron deficiency, iron supplementation will neither


correct nor improve anemia due to thalassemia.

For people with both iron deficiency and thalassemia, eg girls with heavy menses,
iron replacement will lessen the severity of the anemia, until the iron deficiency is
corrected. The blood count will level off and no further improvement will occur. We
need to confirm iron deficiency and a simple cheap screening test is Se ferritin.

67) on Jaundice
Dear Yin Ling,
are you aware that mild Unconjugated hyperbilirubinaemia may actually be
beneficial??
For eg, Oxidative stress is considered a key element in the progression of non-
alcoholic fatty liver to non-alcoholic steatohepatitis (NASH). Unconjugated bilirubin
is the main endogenous lipid antioxidant and is cytoprotective in different tissues
and organs. Bilirubin can act as an important cytoprotector of tissues that are poorly
equipped with antioxidant defense systems, including myocardium and nervous
tissue.
So perhaps conditions that cause mild chronic Unconjugated Hyperbilirubinaemia
like Gilbert's Syndrome may have survival benefits and perhaps is a desirable
mutation and may be among genes considered longevity genes. There are studies
that show a correlation between Gilbert's Syndrome and lower incidence of certain
diseases such as arteriosclerosis and cancer.
Lipid oxidation and formation of oxygen radicals are important elements of arterial
plaque formation and atherosclerosis, and are involved in the pathophysiology of
coronary artery disease. Because unconjugated bilirubin has antioxidant properties,
it has been suggested that it may have a protective role in the atherosclerotic
process.
Both conjugated bilirubin, and unconjugated bilirubin were all noted to be effective
scavengers of peroxyl radicals and to be able to protect human LDL against
peroxidation. Under physiological conditions, the predominant circulatory form of
bilirubin is the unconjugated, albumin-bound form.
Lipoproteins, and particularly LDL, are highly susceptible to oxidation, and it is
known that the atherogenic process involves uptake of oxidized LDL by intimal
macrophages, leading to the accumulation of lipid-rich foam cells. Given the
antioxidant capacity of bilirubin, it is plausible that bilirubin protects lipids and
lipoproteins against oxidation and thereby offers protection against atherogenesis.

In 1994, Schwertner et al were the first to observe a significant inverse correlation


between total bilirubin plasma concentrations and the prevalence of CAD. This
important finding indicated that a lower than normal serum bilirubin concentration
is associated with the presence of ischemic heart disease!
Breimer et al observed a U-shaped relationship between circulating bilirubin
concentrations and cardiovascular risk, leading to the conclusion that low
concentrations of serum bilirubin are associated with increased risk of ischemic
heart disease. These and other investigators found that plasma bilirubin correlated
inversely with several known risk factors for CAD, such as smoking, LDL-cholesterol,
diabetes, and obesity, and correlated directly with the protective factor HDL-
cholesterol.
Low bilirubin was suggested as an independent risk factor for CAD, and an inverse
correlation was demonstrated between bilirubin concentration and CAD morbidity.
Further support for the existence of this inverse correlation came from the work of
Hunt et al, who described a genetic variation in bilirubin concentration, with
individuals with early CAD displaying lower bilirubin than unaffected persons.

68) a typical Bedside class
notes taken by Dr Chin Ming Lee when she was a year3 student
Bedside Teaching (Dr. Wong)
03/03/2011

1. Walks in

Right hemiplegic Stroke (left sided, internal capsule, posterior limb).
gait
Arm flexed, pronated, adducted.
Leg abducted, twisted in (have to swing when he walks).
Unilateral weakness and spasticity with the upper
extremity held in flexion and the lower extremity in
extension. The foot is in extension so the leg is "too long"
therefore, the patient will have to circumduct or swing
the leg around to step forward. Seen with a UMN lesion.
Cerebellar ataxia Looks like someone drunk walking.
Drunken gait, stagger side to side, widen legs for balance.
Gait is wide-based with truncal instability and irregular
lurching steps which results in lateral veering and if
severe, falling. This type of gait is seen in midline
cerebellar disease. It can also be seen with severe loss of
proprioception (sensory ataxia).
Shuffling gait
Parkinsons disease.

Hand pronated.
Seen with rigidity and hypokinesia from basal ganglia
disease. The patient's posture is stooped forward. Gait
initiation is slow and steps are small and shuffling; turning
is en bloc like a statue.
High stepping gait Bilateral Syphilis. Lose proprioception, so stomp feet on
floor.
Unilateral Damage to the sciatic nerve.
Most often seen in peripheral nerve disease where the
distal lower extremity is most affected. Because the foot
dorsiflexors are weak, the patient has a high stepping gait
in an attempt to avoid dragging the toe on the ground.

http://library.med.utah.edu/neurologicexam/html/gait_abnormal.html#12

2. Sits down

Gowers sign Weakness of the proximal muscles, namely those of the lower
limb. The sign describes a patient that has to use his hands and
arms to "walk" up his own body from a squatting position due
to lack of hip and thigh muscle strength.
Classically seen in Duchenne muscular dystrophy, but also
presents itself in centronuclear myopathy, myotonic dystrophy
and various other conditions associated with proximal muscle
weakness.


Continuous stream of slow, sinuous, writhing movements,
typically of the hands and feet. Said to be caused by damage to
the corpus striatum of the brain Specifically to the putamen. It
can also be caused by a lesion of the motor thalamus.
Corrigans
Rapid rise and fall of pulse of the carotid artery on inspection
sign
which is consistent with the wide pulse pressure of aortic
regurgitation.
Collapsing pulse.
Early diastolic murmur.
Wide pulse pressure (need reading with heart beat).
Dancing uvula.
Dancing capillaries with beat.
Fasciculation Normal when stick out tongue.
Athetosis

To find pathology, leave tongue in mouth.


LMN Motor neuron disease Fasciculation everywhere.

3. Face and others

Endocrine
Graves, acromegaly, Cushings.
CVS
Sternotomy (CABG), hyperlipidemia (arcus lipidus), central
cyanosis, clubbing, congenital cyanotic heart diseases,
Eisenmenger syndrome.
Neurology
Bells palsy : Absence of forehead wrinkles, nasolabial fold
(7th).
Ptosis, eyeball down and out (3rd).
Nephrology
CRF Sallow face (lighter than tea colour), uremic breath.
Nephrotic syndrome Peri-orbital edema.
Hepatology
Alcoholic cirrhosis Ascites, gynecomastia, female public hair
distribution.
Rheumatology SLE Butterfly rash.
RA + Swan neck deformity + Dry eyes and mouth = Sjogrens
syndrome.
Dermatomyositis Heliotrope rash.

4. Hands and others

Hands
Acromegaly (spade like).

Darkening of palmar creases (+ pigmentation in mouth) ACTH,
Addisons disease. Unable to produce cortisol, so feedback is to
increase the production of ACTH (same structure as MSH),
leading to hyperpigmentation.

Raynauds phenomenon : White Blue Pink
- The symptoms include several cyclic color changes
1. When exposed to cold temperatures, the blood supply to the
fingers or toes, and in some cases the nose or earlobes, is
markedly reduced; the skin turns pale or white (called pallor),
and becomes cold and numb.
2. When the oxygen supply is depleted, the skin colour turns
blue (called cyanosis).
3. These events are episodic, and when the episode subsides or
the area is warmed, the blood flow returns and the skin
colour first turns red (rubor), and then back to normal, often
accompanied by swelling, tingling, and a painful "pins and
needles" sensation.

- In men, this phenomenon sometimes occurs in individuals


who work with vibrating instruments.
- In patients with systemic scleroderma, attacks are, in
general, more frequent and more severe.

Cervical spondylitis.
Oslers nodes (painful), Janeway lesions (painless).
Ulnar claw 4th and 5th fingers drawn towards the back of the
hand at the first knuckle and curled towards the palm at the
second and third knuckles.



Dupuytrens contracture in CRF Fixed flexion contracture of
the hand where the fingers bend towards the palm and cannot
be fully extended (straightened). Due to contractures of the
palmar aponeurosis (or palmar fascia).



Mixed CT disease ("Sharp's syndrome")
- Autoimmune disease.
- Combines features of scleroderma, myositis, SLE, and RA
(with some sources adding polymyositis, dermatomyositis,
and inclusion body myositis), and is thus considered an
overlap syndrome.
- Commonly causes joint pain/swelling, malaise, Raynauds
phenomenon, Sjgren's syndrome, muscle inflammation,
sclerodactyly (thickening of the skin of the pads of the
fingers).

- Lichenification Thick, leathery skin, usually the result of


constant scratching and rubbing.


Livedo reticularis
- Mottled reticulated vascular pattern that appears like a lace-
like purplish discoloration of the lower extremities. The
discoloration is caused by swelling of the medium veins (not
small) in the skin, which makes them more visible. So it can
be caused by any condition that makes venules swell.
- Broad differential diagnosis, broadly divided into possible
blood diseases, autoimmune (rheumatologic) diseases,
cardiovascular diseases, cancers, and endocrine disorders.
SLE, anti-phospholipid syndrome, Sneddon's syndrome.


Boutonniere deformity (Button through button hole)
- Deformed position of the fingers or toes, in which the joint
nearest the knuckle (PIP) is permanently bent toward the
palm while the furthest joint (DIP) is bent back away (PIP
hyperflexion with DIP hyperextension). It is commonly
caused by injury or by an inflammatory condition like RA.
- This flexion deformity of the proximal interphalangeal joint is
due to interruption of the central slip of the extensor tendon
such that the lateral slips separate and the head of the
proximal phalanx pops through the gap like a finger through
a button hole (thus the name, from French boutonnire
"button hole"). The distal joint is subsequently drawn into
hyperextension because the two peripheral slips of the
extensor tendon are stretched by the head of the proximal
phalanx (note that the two peripheral slips are inserted into
the distal phalanx, while the proximal slip is inserted into the
middle phalanx). This deformity makes it difficult or
impossible to extend the proximal interphalangeal joint.


Scleroderma, psoriasis, vasculitis.

Tophaceous gout A chronic form of gout. Nodular masses of
uric acid crystals (tophi) are deposited in different soft tissue
areas of the body. Even though tophi are most commonly found
as hard nodules around the fingers, at the tips of the elbows,
and around the big toe, tophi nodules can appear anywhere in
the body (e.g. ears, vocal chords, spinal cord).

Digital gangrene
- Atherosclerosis.
- Vasculitis.
- IgA nephropathy (Berger's) Primary IgA nephropathy is
characterized by deposition of the IgA antibody in the
glomerulus. There are other diseases associated with
glomerular IgA deposits, the most common being Henoch-

Schnlein purpura (HSP), which is considered by many to be


a systemic form of IgA nephropathy. HSP presents with a
characteristic purpuric skin rash, arthritis, and abdominal
pain and occurs more commonly in young adults.

Marfans
- Arachnodactyly (fingers are abnormally long and slender in
comparison to the palm of the hand).
- Hyperextensibility.
Toxic shock syndrome (TSS)
- Potentially fatal illness caused by a bacterial toxin.
- Staphylococcus aureus and Streptococcus pyogenes.
- Diffuse rash, intense erythroderma, blanching with
subsequent desquamation, especially of the palms and soles.
- Tampons and septicemia.


Tylosis Rare inherited disease characterized by excess skin on
the palms and soles. Affected patients have a much higher
probability of developing esophageal cancer than the general
population.

Nail-pitting Psoriasis.
Beaus lines Unwell.
Koilonychia Iron deficiency.
Pigmentation of nails Zidovudine. Bluish or brownish-black
discoloration of nails may develop during the first month or two
of zidovudine therapy and usually disappears within 2 months if
the drug is discontinued. Discoloration may occur as longitudinal
streaks or transverse bands.


Charcot Marie Tooth
- An inherited disorder of nerves (neuropathy) that takes
different forms.
- Characterized by loss of muscle tissue and touch sensation,
predominantly in the feet and legs but also in the hands and
arms in the advanced stages of disease.

Clubbing

Thenar
muscles

Dystrophia
myotonica

- Usually, the initial symptom is foot drop early in the course


of the disease. This can also cause claw toe, where the toes
are always curled. Wasting of muscle tissue of the lower
parts of the legs may give rise to "stork leg" or "inverted
bottle" appearance. Weakness in the hands and forearms
occurs in many people later in life as the disease progresses.
- Breathing can be affected in some; so can hearing, vision,
and the neck and shoulder muscles. Scoliosis is common. Hip
sockets can be malformed. Gastrointestinal problems can be
part of CMT, as can chewing, swallowing, and speaking (vocal
cords atrophy). A tremor can develop as muscles waste.
Endocarditis.
Congenital cyanotic heart disease.
PDA complicated by Eisenmenger (only toes, distal to SCA).
Bronchiectasis.
Lung abscess.
CA lung.
Idiopathic lung disease.
IBD.
Liver disease.
Thyrotoxicosis.
Atrial myxoma
Fibrous alveolitis.
* NOT in COPD.
Flattened Median nerve palsy.
Bilateral carpal tunnel
Acromealgy
Inreased weight.
Menopausal
Chronic, slowly progressing, highly variable inherited multi-systemic
disease.
Stiff handshake, cannot let go.
Sad, shabby, droopy face.
Wasting of the face (triangular face, lower half of face sunken).
Marked wasting of the neck muscles May have floppy neck.
Bilateral ptosis.

Rheumatoid
arthritis


Swan neck deformity DIP hyperflexion with PIP
hyperextension.
Boutonniere's deformity
Z deformity of thumb Hyperextension of the interphalangeal
joint, and fixed flexion and subluxation of the
metacarpophalangeal joint.
Finger ulnar deviation (MCP).
Hepatology Palmar erythema Liver failure, PRV, pregnancy, thyrotoxicosis.
and nails
Terrys nail (leukonychia, white nails) Hypoalbuminaemia
caused by nephrotic syndrome.
Half and half nails ("Lindsay's nails")
- Proximal portion of the nail white and the distal half red,
pink, or brown, with a sharp line of demarcation between the
two halves.
- Chronic renal failure.
EhlersDanlos syndrome (EDS)
- A group of more than 10 different inherited disorders; all
involve a genetic defect in collagen and connective-tissue
synthesis and structure.
- Can affect the skin, joints, and blood vessels.

Scarlet fever
Caused by exotoxin released by Streptococcus pyogenes.
Characterized by
(1) Sore throat.

(2) Fever.
(3) Bright red tongue with a strawberry appearance Inflamed red papillae.
Seen in Kawasaki disease, toxic shock syndrome, and scarlet fever. May mimic
glossitis or B12 vitamin deficiency.
* Kawasaki disease An autoimmune disease that manifests as a systemic
necrotizing medium-sized vessel vasculitis and is largely seen in children less than
5 years of age. It affects many organ systems, mainly those including the blood
vessels, skin, mucous membranes and lymph nodes; however, its most serious
effect is on the heart where it can cause severe coronary artery aneurysms in
untreated children.













(4) Characteristic rash.
- Fine, red, and rough-textured. Blanches upon pressure.
- Generally starts on the chest, armpits, and behind the ears.
- Spares the face (although some circumoral pallor is characteristic).
- Worse in the skin folds. These Pastia lines (where the rash runs together in
the armpits and groin) appear and can persist after the rash is gone.
- May spread to cover the uvula.

Reddened sore throat, a fever at or above 101 F (38.3 C), and swollen glands in the
neck. The tonsils and back of the throat may be covered with a whitish coating, or
appear red, swollen, and dotted with whitish or yellowish specks of pus. Early in the
infection, the tongue may have a whitish or yellowish coating. Also, an infected
person may have chills, body aches, nausea, vomiting, and loss of appetite.




Hansens disease/Leprosy
Chronic disease caused by the bacteria Mycobacterium leprae and
Mycobacterium lepromatosis.
Primarily a granulomatous disease of the peripheral nerves and mucosa of the
upper respiratory tract; skin lesions are the primary external sign.

Left untreated, leprosy can be progressive, causing permanent damage to the


skin, nerves, limbs and eyes. Contrary to folklore, leprosy does not cause body
parts to fall off, although they can become numb or diseased as a result of
infection; infection results in tissue loss, so fingers and toes become shortened
and deformed as the cartilage is absorbed into the body.


69) On DOPES!
Dear Yin Ling,
We must not allow Dopeys in medicine! That is only allowed in the 7 dwarfs!
No apex beat felt - WHY?
Remember DOPES

Dextrocardia (pls don't say this first!),
Obesity,
Pleural/Pericardial effusion,
Emphysema, Empyema
Shock, Silly medical student


70) on Eating
Yin ling says I feed her too much at dinner and hence she is sleepy post dinner! True
or false?
Is there a physiological basis?
Did your teachers in year 1 teach you well?

High-carb, high-fat foods (like most Msian meals esp my beloved Hokkien mee)
trigger a neural response when it reaches the small intestine. Thats why your
sleepiness is Not instant but after an hour or so.
That response, via the parasympathetic nervous system, focus on moving the
intestines and digesting the food rather than study. Think of the python after
swallowing a chicken... it lies flat and unmoving like my med students!

A group of brain cells called orexin neurons are found in the hypothalmus and are
very sensitive to glucose levels, which spike after a CHO meal. These neurons
produce orexin, which moderates wakefulness in the brain.
But orexin isn't the only sleep-related neurohormone affected by food. As the

quantity of food increases, so too is the amount of insulin released. The insulin
increases the amount of seratonin and melatonin that flood the brain, two
chemicals associated with drowsiness (and, for that matter, happiness). Remember
the Melatonin tablet sold at airports to help travellers sleep? Now you know also
why when some people are stressed they EAT! Natural SSRI !

Hence we feel Happy after our CNY reunion dinner. Drinking tea helps to keep us
awake. And we just want to slouch on the sofa and sing or chit chat or 'chill'

So what can the med student or post grad do to solve this dilemma? Easy. Give me
the food. I can afford to sleep!


71) on Beta blockers and Hypoglycaemia

Dear Yin Ling,
What hypoglycemic symptom is not masked by beta blockers?




Symptoms and Signs of Hypoglycemia
Autonomic:
Sweating
Feeling hot
Pins & needles
Shakiness
Anxiety
Palpitations
Cognitive:
Difficulty speaking
Loss of concentration
Drowsiness
Dizziness
Hemiplegia
Arrhythmias, Seizures, Coma, Death
Nonspecific:
Nausea
Hunger
Weakness

Chor Kuan:
1)Because of its ability to mask autonomic symptoms and suppress glycogenolysis,
beta blockers must be used with caution by diabetic patients

2)First, by blocking beta-1 receptors, autonomic symptoms are inhibited. Among
them is tachycardia, which normally serves as an early warning signal that blood
glucose levels are falling too low. (When glucose drops, the sympathetic nervous
system is activated, causing autonomic symptoms)

3) Second, by blocking beta-2 receptors in muscle and liver, beta-blockers suppress
glycogenolysis, thereby eliminating an important mechanism for correcting
hypoglycemia (which can occur when insulin dosage is excessive).

4) By masking these autonomic symptoms, beta-blockers can delay awareness of
hypoglycemia, thereby compromising the patient's ability to correct the problem in
a timely fashion.

5) Therefore, patients should be taught to recognise alternative signs which are NOT
masked namely the cognitive and non-specific signs (eg: hunger, fatigue, poor
concentration, confusion) that blood glucose is falling dangerously low.

6) SWEATING is an exception to this. Since sweating during activation of the ANS is
mediated by release of Acetylcholine via activation of a nicotinic receptor and NOT
BY NORADRENALINE via an adrenergic receptor, sweating may be one of the only
signs recognised by diabetic patients on a beta-blocker.


Hypoglycemia related symptoms is seen in patients who are either aware,
or have attenuated sympathetic neural response (hypoglycemia unawareness) that
can develop in older patients, patients suffering from recurrent hypoglycemia, given
beta blockers or those with diabetic autonomic neuropathy.
In patients who are hypoglycemic aware, a fall in blood glucose below ~3.8 mM
results in an acute release of counter-regulatory hormones including glucagon &
norepinephrine. The release of norepinephrine results from a CNS-mediated
sympathetic discharge triggered by hypoglycemia. The sympathetic discharge
produces adrenergic symptoms produced by the release of norepinephrine (and

possibly by epinephrine release from the adrenals) such as palpitations, tremor &
anxiety.
Cholinergic symptoms such as sweating & hunger occur from the release of
acetylcholine from sympathetic postganglionic (cholinergic) neurons.
Cognitive dysfunction begins to deteriorate when blood glucose falls to ~ 3 mM. The
onset of autonomic symptoms is important because it makes the patient aware of
their condition & enables them to take appropriate corrective action before
cognitive impairment occurs.
However in patients who are hypoglycemic unaware this early phase of
sympathetic discharge (with associated warning signs) does not occur until after
cognitive impairment begins, which increases the likelihood that they can become
severely hypoglycemic.
The threshold for cognitive impairment does not change when patients develop
hypoglycemia unawareness.

Beta-blockers should be used with caution (have a relative contraindication) in
diabetics because of their ability to block these sympathetic increases and thus
make proper awareness more difficult. However, if a diabetic patient had suffered a
previous MI, has CHF, or has a combination of hypertension and coronary artery
disease, the concern about the potential benefits of beta blocker therapy would
generally outweigh the concern about its affects to blunt reactions to hyperglycemia
in most diabetic patients.
Note that some responses to hypoglycemia (such as sweating) would not be blocked
by beta blockers. Because Cholinergic symptoms such as sweating & hunger occur
from the release of acetylcholine from sympathetic postganglionic (cholinergic)
neurons, beta blockers do not mask these symptoms!
Kudos to basic physiology!! ALL HAIL the late Prof Raman!!


Dr Hu Mung Chee: There is evidence that hypoglycemia may increase the risk of
dementia many years later.

There are several possible mechanisms by which hypoglycemia could increase risk of
subsequent dementia in older patients. Severe hypoglycemia can result in
permanent neurological sequelae including neuronal cell death, which may
accelerate the process of dementia. Hypoglycemia also increases platelet
aggregation and fibrinogen formation, and this may accelerate vascular compromise
in the brain. Animal studies have illustrated that hypoglycemic coma causes damage
to neuronal receptors in the ca-1, subiculum dentate, and granule cell areas of the
hippocampus, regions critical for learning and memory. Repeated episodes of
hypoglycemia could affect cognition through damage to these regions, particularly
in brains that may be vulnerable due to old age.

~ JAMA. 2009;301(15):1565-1572


On Beta Blockers and Heart Failure


In CCF, Beta blockers block beta receptors in the kidneys which leads to reduction of
Renin Angiotensin Aldosterone output.
Choose beta blockers with no ISA activity because the partial agonist properties
negates its beneficial effects.
Bisoprolol being a highly selective beta1 blocker has minimal effects on lung
function in asthmatics.
It appears also to have no effect on Hba1c in diabetics.
In patients with LVef of less than 40% beta1 selective beta blockers should be used.
Bisoprolol or Cardivelol are the drugs of choice after optimization of volume status
and successful discontinuation of iv diuretics.
Start LOW and GO SLOW in uptitration.
Patients with CCF in early stages die from arrthymias at home, beta1 selective
blockers reduce that risk.
In advanced CCF PATIENTS die from pump failure

Yin Ling, what are the contraindications to Beta Blockers?

A Asthma, ACUTE pulm odema, not stable CCF, Athletes
B - bradycardia, heart block
C - COPD
D - DM, dyslipidemia, Dreams
E Extremities, ED
F - Fatigue, foetus
G - Geriatrics
H Hypothyroid, hypotension, Hypoglycaemia, Hyperkalaemia
I had to commit the list to memory as a HO 30 years ago. My Consultant Dr Mrs Kula
would NOT allow us to use Beta Blockers if we cannot remember this list.


72) on Triglycerides and DM
Dear Yin Ling,
I grilled the students of causes of hypertriglyceridaemia and many have no idea!
Always think of Alcoholism and Diabetes when seeing patients with High TG which is
very2 common. Metabolic syndrome seen in 32.2% of adult Msians!
High TG contribute to residual risk even after statin Rx

Insulin resistance causes release of FFA fr the liver. This is association with high TG,
HIGH SMALL LDL and low HDL levels. TG causes small dense LDL particles to increase
and these are highly atherogenic
LDL particle numbers increase with increasing TG

LOW LDL LEVELS AND LOW LDL PARTICLE NUMBERS MEANS LOW RISK
HIGH LDL LEVELS BUT LOW LDL PARTICLE NUMBERS IS ALSO LOW RISK!!!

Low carbo, fructose and sugar intake is key to TG reduction.
Fibrates makes small dense LDL BECOME BIG, LESS DENSE LDL.

We are not able to routinely measure LDL3 the small dense LDL.
SURROGATE FINDINGS OF HIGH TG, LOW HDL AND HIGH LDL IMPLIES HIGH LDL3
With Fibrates Rx we see small rise in HDL and mark drop in TG. This suggests that
despite no change in total LDL LEVELS, THE LDL3 IS LESS AND LDL1 MORE. LDL1 is
bigger and less DENSE

We have to Move beyond Statins in DM dyslipidaemia
DM macro and micro vascular disease has multifactorial contributors.
Pls note that ldl is not homogenous in its size as noted above. But LDL remains the
target to address on Rx. Guidelines all agree that it is our primary target.
Ldl reduction is effective in reducing CV risk. This is clear. And a statin is clearly the
drug of choice.
BUT Pls note that there is High residual risk even with statin Rx.
The lower the hdl the higher the risk. This holds true even for those on statins.
When the tg is high this further compounds the problem
High tg and low hdl together occurs in 20% of diabetics.
High tg alone occurs in 33%
Small dense ldl3 is typically seen when tg is high and hdl low as noted above and this
is highly atherogenic.
Niacin in the Aim High study had no difference in event rate.
Fibrates in the Helsinki heart study using gemfibrosil showed good results in the high
tg, low hdl group.
Bezafibrate similarly showed benefit only in the high tg group

Fenofibrate 200mg micronised in Field study again showed 23% reduction in risk in
high tg group.
But 27% in high tg and low hdl group and 14% in the low hdl group.
The highest cv risk was in the high tg low hdl group.
There is a role for patients on statins who still have low hdl and high tg to take
fenofibrate as well.
Fenofibrate is very useful for Microvascular complications:
Fibrates stimulate ppar alpha, and have anti inflammatory properties.
These benefits DM retinopathy;
40% reduction, the same as with intensive bld sugar control.
Fenofibrate is approved as a specific medication to treat DM retinopathy. Even
patients with NO dyslipidaemia benefitted.
Renal effects: there is a rise in measured se creatinine with fenofibrate but this is
not due to a decrease in GFR!
Real Creatinine clearance does not change. This is different from ACEI
eGFR will drop because it depends on se creatinine levels for calculation.
The increase in creatinine reverses when the drug is stopped.
Fenofibrate reduces the progression of microalbuminuria. There appears to be a
protective effect on the kidneys.
There is also a reduction in non traumatic amputations.
There is reduction of monofilament positive neuropathy.
Use in renal impairment: Even in patients with baseline GFR of 30 in the FIELD study,
patients using the usual dose had no issues.
In patients with GFR even lower than that, fenofibrate usage is questionable.
......................
Gemfibrosil significantly interacts with statins and elevates statin levels. There is
increase risk of complications. But There is no evidence of benefit with regards to
retinopathy, renal or neuro complications with gemfibrosil.
.....................
A rise in Post Prandial TG is the earliest indication of impending DM!

In Malaysia, in 2011 for those more than 18yrs, the prevalence was 15%
For those more than 30yrs old, it was a shocking 20%.
No difference between rural and urban popn!
Hence a terrifying situation exists whereby we drs must start acting fast.

The epic centre of DM is now in Asia.
Asians dev DM younger and at lower bmi!
Post prandial hyperglycaemia...... isolated IGT reflects a high risk of DM!

We eat lots of Rice... even the normal non diabetic Chinese have a much higher area
under the glucose curve post prandially cf to caucasians.
Asians who are diabetics are MORE LIKELY to dev esrf.
Indian diabetics have the highest risk of IHD

We need to screen YOUNGER PATIENTS AND AT LOWER BMIs
Start at 30yrs. And watch for renal impairment early.
We need a cultural change to eat less carbos,, less rice.
And we should cease to greet each other with.... "have you eaten?" But "have you
exercised?"


73) on Dippers

Dear yin ling,
Are you aware that Lung Function also has a diurnal variation; it is WORSE at 4am
and Best at 4pm!

Now do you know why asthmatics and COPD patients go WHHEEEEE when the HO
just about enters REM sleep?
Early morning "Tightness" is an important question to ask as some patients do not
wheeze.
Do you wake up at night is another important question.... Its not just heart failure
that causes PND!!

How good is your patient's asthmatic control??
If he uses MORE than 1 reliever inhaler A YEAR, his control is NO GOOD!

Good control is when a patient uses NOT MORE than or Equal to merely 2 puffs a
week, hence 52 x 2 = 104 puffs an ENTIRE YEAR! A single MDI is more than sufficient
for this.
In general, 80% of patients with asthma are NOT well controlled.

The use of a steroid inhaler as preventer alone has the Disadvantage that the patient
Does NOT feel better, hence compliance is poor. Patients will only take medicines
which makes them Feel Better no matter how much we talk!
Hence a Combination of a LABA+Steroid has an advantage here. Not only is the
inhaled steroid which is all important delivered, the patient feels a difference and is
psychologically convinced that he is using a "good" medicine.
Steroid inhalation is the bedrock of asthma treatment for any acute episode of
asthma BEGETS more asthma!

Exacerbation is furthermore associated with LOSS OF LUNG FUNCTION!
Asthma is like Diabetes and Hypertension; a CHRONIC disease which causes more
and more damage. While we had over decades of hard work convinced patients on
the need to treat DM and HPT to prevent its terrible complications, Many Doctors,
medical students and Patients are NOT even aware that Asthma if uncontrolled will
accelerate the loss of lung function.
Many are aware that COPD is associated with progressive deterioration of Lung
function but now pls know that SO IS ASTHMA when uncontrolled!
Only Inhaled steroids can Blunt this effect.

As we age we loose Lung Function progressively; typically about 20% is lost by the
age of 80 years in a Non Smoker staying in a non polluted place. Here with all our
second hand smoke, smog and CO and CO2, it is worse!
Many asthmatics BECOME COPD like patients as their lung function deteriorate.
Have I not screamed myself hoarse at bedside about the 3 circles of Asthma, the In
Betweens and COPD!?

Many COPD patients with NO history of smoking or cooking with firewood may
have had poorly controlled asthma in their younger years.
Even with relatively well asthmatic patients with minimal symptoms NOT on
treatment with inhaled steroids, airway biopsies have shown persistent eosinophillic
infiltration. This leads to progressive inflammation and destruction. Inhaled steroids
has been shown to prevent this.
Evidence does NOT support the use of an inhaled steroid ALONE or the use of
Inhaled reliever alone even with so called "well controlled" asthmatics. A
combination of LABA plus a steroid is still better. Combination is better than either

component medication used singly. The FDA however does not agree but that is
another story!


74) on Glucosamine and Gingko

Dear Yin Ling,

Many patients take Glucoasamine for joint disorders. Are you aware of any
dangers associated with it?

From Medscape:
Glucosamine supplementation was linked to significant, reversible increases in
intraocular pressure (IOP) in a small, retrospective study published online May 23 in
JAMA Ophthalmology.
Frequently, patients are being told that while studies give conflicting data as to
whether glucosamine and chondroitin sulfate are effective in reducing arthritic pain,
there does not appear to be any risk in trying these supplements.
US prevalence of osteoarthritis is 27 million, and for open-angle glaucoma, it
exceeds 2 million, according to the Centers for Disease Control and Prevention.
Although it is unclear what the implications of having increased IOP would be for
individual patients, the risk is that ocular damage could possibly occur from what
were previously thought to be benign supplements.

Yin Ling,
you know of folks who take Gingko in hopes of boosting their memory or even
preventing Alzheimers (and the latter is not true as studies show Gingko does not
prevent Alzheimers disease) . Now there is more data to suggest caution before you
decide to take Gingko.
The first US government toxicology study of ginkgo biloba found that the extract
caused cancer in lab animals, including an excessive number of liver and thyroid
cancers, as well as nasal tumors.
The findings were somewhat surprising because ginkgo biloba has had a long and
apparently benign history of human use. Although it has been associated with
bleeding and cerebral hemorrhages in the elderly, there have generally been few
reports of serious side effects.

The results of the study do not confirm that ginkgo biloba is dangerous to humans,
but it is disturbing that the laboratory animals all tended to suffer the same sorts of
injuries. The study concluded that there is clear evidence that ginkgo causes
carcinogenic activity in the livers of mice and some evidence linking it to
carcinogenic activity in rats thyroids.
Findings in animal studies may not necessarily translate to humans but nevertheless
it is disturbing to know that Gingko has carcinogenic activity.
Before you ingest any herb, always ask yourself
- what are the proven benefits?
- what is the potential harm?
If in doubt, DONT


75) on Pre Hypertension

Dear Yin Ling,

If I am to ask you to comment on this, what will you say?

This is like getting engaged but not married, question is, ARE YOU IN BIG TROUBLE
OR ARE YOU NOT?
Can you get out of the mess or are you stuck!?
The diagnosis was invented in May 2003
Systolic 120 to 139
Diastolic 80 to 89
Based on MRFIT there is a continuous increase in risk as bp rises
If there is Pre DM, why not Pre HPT?
Optimal bp is less than 120 80
Annual mortality rate for those with Pre HPT is more than 3 times that of a normal
person. NHAMES
Over 4 yrs, conversion of Pre Hpt to Hpt in the elderly is more than 50%
Progression is fast, almost 60% by 4 yrs

Pre Hpt is less in those more than 60yrs as most have PROGRESSED TO HPT!
Prevalence is 37% in msia! MORE COMMON IN MALES
Worldwide PREVALENCE is 38%
Rising obesity is associated with Pre Hpt
Pre Hpt is associated with insulin resistance
LV mass is higher in Pre Hpt
Pre Hpt 27% increase in all cause mortality
And
66% increase in cvs mortality
Management. Wt reduction, salt reduction, stress management, but reduction is not
sustainable with time.

Wt loss is best
Salt reduction is second best
Drugs?? Do we treat one third of the entire adult population??
No outcome data if Pre Hpt ALONE!
BUT trials have shown that we shd treat when there are other risk factors or when
risk scores are high.
Look for target organ damage, if so TREAT!
The younger the patient, the more vigilant the search for secondary causes.
1. Confirm the High BP
2. Search for underlying causes
3. Search for Target organ damage
Remember "High BP causes damage!"

76) on the Solitary Pulmonary nodule

Dear Yin Ling,

This finding is worrying for both doctor and patient. How do you approach this
diagnostic problem?



More than half of all solitary pulmonary nodules are benign. Benign nodules have
many causes, including old scars and infections. It is surrounded by normal lung
tissue and is not associated with any other abnormality in the lung or nearby lymph
nodes
Infectious granulomas (reactions to a past infection) cause most benign lesions.
Common infections that increase the risk for developing a solitary pulmonary nodule
include:
Tuberculosis
Lung diseases caused by a fungus
Lung cancer primary or secondary is the most common cause of cancerous
(malignant) pulmonary nodules. Approximately 20-30% of all cases of lung cancer
appear as Solitary Nodules on chest X-ray films. Therefore, the goal of
investigating is to differentiate a benign growth from a malignant growth as soon
and as accurately as possible.
It should be considered potentially cancerous until proven otherwise.
A solitary pulmonary nodule is most often found on a chest x-ray or a chest CT scan,
which are often done for other symptoms or reasons.
The clinical decision is whether the nodule in lung is probably benign. This is more
likely if:

The nodule is small, has a smooth border, and has a solid and even appearance on
an x-ray or CT scan
Patient is young and do not smoke
We may then choose to just watch the nodule on x-rays in 2 mths.
Age: Risk of malignancy increases with age.
Risk of 3% at age 35-39 years
Risk of 15% at age 40-49 years
Risk of 43% at age 50-59 years
Risk of greater than 50% in persons older than 60 years.
Patients who have an older chest X-ray film has a goldmine for comparison. This is
important because the growth rate of a nodule can be ascertained. The doubling
time of most malignant Nodules is 1-6 months.
Repeat chest x-rays or chest CT scans are the most common way to follow the
nodule. If the CT scan demonstrates fat within the nodule, the lesion is likely benign,
hamartoma. Nowadays lung PET scans may be done. Malignant cells have a higher
metabolic rate than normal cells and benign abnormalities; therefore, the glucose
uptake of malignant cells is higher. Positron emission tomography (PET) involves
using a radiolabeled substance to measure the metabolic activity of the abnormal
cells. Malignant nodules absorb more of the substance than benign nodules and
normal tissue and can be readily identified on the 3-dimensional, colored image.
PET scan is an accurate, noninvasive exam, but the procedure is expensive.
If repeated x-rays show that the nodule size has not changed over 2 years, it is most
likely benign and a biopsy is not needed.
Persons who have been clinically diagnosed with a benign Nodule should schedule
the recommended follow-up, as follows:
Chest X-ray films should be taken every 3 months for the first 12 months and then
every 6 months for the following 12 months.
After this 2-year period, the Nodule may be observed yearly for up to 5 years.
We may choose to biopsy the nodule to rule out cancer if:
You are a smoker
You have other symptoms of lung cancer
The nodule has grown in size or has changed compared to earlier x-rays.
A transbronchial needle aspiration (TBNA) biopsy may be done if the nodule is close
to the airway.

Transthoracic needle aspiration (TTNA) biopsy: This type of biopsy is used if the
lesion is not easily accessible on the airway wall or is smaller than 2 cm in diameter.
If the SPN is on the periphery of the lung, a biopsy sample has to be taken with the
help of a needle inserted through the chest wall and into the SPN. It is usually
performed with CT guidance.
With SPNs larger than 2 cm in diameter, the diagnostic accuracy is higher (90-95%).
However, the accuracy decreases (60-80%) in nodules that are smaller than 2 cm in
diameter.




77) on Blessings

Dear Yin Ling,

Are you aware of the Different kind of Blessings?

The Ulnar Blessing:


An ulnar claw is an abnormal hand position that develops due to a lesion with the
ulnar nerve. A hand in ulnar claw position will have the ring and little fingers drawn
towards the back of the hand at the MPJoint and curled towards the palm at the PIP
and DIP when the fingers are extended.
Remember the action of the Lumbricals!? What is the LAST muscle used in the act of
MALE Micturition?!! The medial two lumbricals are not working due to ulnar
damage, the 3rd & 4th lumbricals are unable to extend the PIP & DIP joints at the
4th & 5th digits, so the medial two digits are clawed; clawing is caused by extension
of MP joint by Extensor digitorum and flexion caused by Flexor digitorum..
The ulnar nerve controls the 3rd & 4th lumbricals, the three hypothenar muscles,
the dorsal & palmar interossei, the palmaris brevis and the adductor pollicis. Ulnar
nerve damage may also cause hypothenar atrophy.


The ulnar claw can present as a "hand of benediction" or "pope's blessing".
However, the term "hand of benediction" or "pope's blessing" also commonly refers
to a similar hand position which is caused by damage to the median nerve and is
only present when the patient is asked to ........ make a fist!!
Hence students are often confused because of these different Blessings!!
A different hand of benediction results from injury of the Median nerve:
The pope's hand is seen with median nerve dysfunction when asking the patient to
make a fist due to inability to flex 1st & 2nd fingers at PIP. The median nerve
controls the 1st & 2nd lumbricals, three thenar muscles (abductor pollicis brevis,
flexor pollicis brevis, and via a distal branch the opponens pollicis). Additionally
there may be thenar atrophy.
The extensor digitorum is left unopposed and the metacarpophalangeal joints of
index and middle fingers remain extended while attempting to ..... make a fist.

Therefore in median nerve injury, there will be BENEDICTION sign when making a
FIST....but NO benediction sign when fingers are EXTENDED!!
Medical students must first understand the ALL IMPORTANT Question the late
Professors of Anatomy used to ask us: What is the last muscle used in the act of
MALE micturition? and all is crystal clear!

So first understand two important concepts: 1) The lumbricals and what they do. 2)
Is the patient being asked to extend their fingers or make a fist?
The Lumbricals are responsible for flexing the MCP and extending the PIP and DIP
joints. The Median nerve controls the lumbricals for the Index and Middle fingers.
The ulnar nerve controls the lumbricals for the ring and little fingers.
Therefore if the lumbricals dont work then we have the opposite: extended MCP
and flexed PIP and DIP.




78) on Heart Blocks

Dear yin ling,
What is it that makes Mobitz type 2 so important?
Mobitz type 2 is associated with bigger infarcts, typically Anterior with larger area of
ischaemia. The AV node is affected by ischaemia from the front of the heart to the
back! And the AV node when it is blocked results in a Complete heart block paced by
a focus much LOWER in the conduction system resulting in LOW heart rates
inadequate for perfusion.
Wenkebach on the other hand is typically associated with an Inferior infarct and is
affected from the back. A relatively smaller area of at risk ischemic myocardium is
involved and when the AV node is blocked temporarily the resultant pacer is usually
high in the ventricular conduction system giving a reasonably sufficient ventricular
rate. And it tends to reverse back to Type1 and then sinus.
Remember that Wang Kee Bak is a good guy. Its area of infarct is smaller, a more
distal coronary lesion and if it progress to CHB, then a HIGH ventricular pacer with
sufficient heart rate takes over.

Mr Mobitz type2 is a bad guy with larger area of infarct, more ischemic myocardium
and more proximal coronary obstruction. When it progress to CHB the ventricular
pacer is LOW down with Slow heart rates. Pacing is need



79) on Hepatic Encephalopathy
Dear yin ling,
What are the factors that tip a Liver failure patient to Encephalopathy?
In a small proportion of cases, the encephalopathy is caused directly by liver failure;
this is more likely in acute liver failure. But more commonly something tipped the
patient from stable to decompensated and then Encephalopathy, and this is

especially in chronic liver disease. We need to identify these causes as reversing and
avoiding them is important to treatment and prevention.
Think of causes of Excessive nitrogen load.
Eating an entire bucket of KFC is unlikely to happen but it is in theory a possible
cause as large amounts of protein is eaten and will break down to nitrogen.
Ammonia remains as the most important factor in the pathogenesis of Hepatic
Encephalopathy. Ammonia is generated in the intestines from different sources:
nitrogenous components of the diet, deamination of glutamine, and breakdown of
urea by urease present in colonic flora.

I wondered aloud as to how Dracula does it but a significant amount of blood in the
GIT will do the same thing. Cirrhotic patients with portal HPT and gastrointestinal
bleeding from esophageal varices is the KFC equivalent. Patients with hepatic
dysfunction and renal failure is at high risk as they are unable to excrete nitrogen-
containing waste products and urea is our surrogate marker of this. A very common
cause is constipation with bacteria having a nice party in the gut, hence we must
ensure that there is regular bowel movements. Lactulose is often used for this
purpose.
In normal circumstances, most ammonia is metabolized to urea in the liver. Portal-
systemic shunts and liver failure cause a rise in blood ammonia that may affect brain
function. Skeletal muscle is capable of decreasing blood ammonia by metabolizing
ammonia to glutamine. The kidney has also an important role in determining blood
ammonia by excreting urea in the urine and generating ammonia. An increase in the
generation of ammonia in the kidney has been shown after gastrointestinal
bleeding and may follow dehydration and the administration of diuretics.
Hence the Houseofficer may contribute to Electrolyte disturbance and tipping the
patient by his treatment with diuretics for the edema and ascites! Dehydration,
Hyponatraemia and hypokalaemia is common with diuretics.
The old chappie is restless and disturbing the nurses and the other patients. The
nurse calls the HO, what does he do? SEDATE?? Sedatives such as benzodiazepines,
narcotics for the patient complaining of discomfort may well tip him to gagaland.
(My apologies to Lady Gaga).
Anyone lying in bed, anyone in HOSPITAL is at risk of Infections! Yup hospitals are
dangerous places!! He may have a Pneumonia from lying unmoving or from
aspiration or from his neighbour coughing his lungs away (and not picked up
because the HO DID NOT EXAMINE the lungs!), or a urinary tract infection because

of the Folley's catheter or prostatism, and yup that's the final straw. Avoid
infections! The temptation to tap that huge ascitic abdomen is hard to resist but any
procedure may introduce infections too. And of course there is Spontaneous
Bacterial Peritonitis!
And there are many instances whereby the precipitant is Unknown! Perhaps as
many as 2030% of the patients.



Hu Mung Chee: Creatinine is an inaccurate marker of renal function in most cirrhotic
patients. In liver cirrhosis patients, there are many biases and pitfalls in the
interpretation of creatinine as well as creatinine-based estimates of renal function.

Assessment of glomerular filtration rate (GFR) by common creatinine-based
methods potentially is very inaccurate in patients with cirrhosis. Cirrhotic patients
have several underlying conditions that contribute to "falsely low" serum creatinine
concentrations, even in the presence of moderate to severe renal impairment, and
often cause creatinine-based methods to overestimate true GFR. Such underlying
conditions include decreased creatinine production secondary to decreased hepatic
creatine synthesis, increased tubular creatinine secretion, and decreased skeletal
muscle mass. These factors all contribute to serum creatinine concentrations that

often do not accurately reflect renal function. Another bias comes from fluctuations
of serum creatinine, especially in those with refractory ascites and/or those
receiving diuretics.

Significant inter-laboratory variations may also be observed in the measurement of
creatinine, mainly due to an interaction with bilirubin. Routine creatinine
measurement is based on spectrophotometry. In patients with jaundice, bilirubin as
a chromogen interferes with creatinine measurement, resulting in falsely low
creatinine values. The higher is serum bilirubin, either conjugated or unconjugated,
the higher is the interference.

Yin Ling:
Chronic liver diseases are accompanied by changes in splanchnic and systemic
circulation. These changes are characterised by a reduction in peripheral vascular
resistance and an increased cardiac output at rest. An increased release of nitric
oxide (NO) has been proposed to play a role in the pathogenesis of vasodilatation
and vascular hypocontractility.
In cirrhotic patients, the increase in portal pressure results from a combination of
increased portal blood flow secondary to splanchnic arteriolar vasodilation and
elevated resistance to outflow through distorted hepatic sinusoids.
The potent vasodilator nitric oxide (NO) plays an important role in portal
hypertension. In patients with cirrhosis, NO bioavailability is decreased in the
intrahepatic circulation due to defects in the posttranslational regulation of
endothelial NO synthase. This deficiency of NO, along with mechanical factors in the
sinusoids, contributes to the increase in intrahepatic resistance. In the systemic and
splanchnic circulation, NO bioavailability is increased due to upregulation and
posttranslational regulation of endothelial NO synthase, thereby increasing
splanchnic vasodilatation and leading to increased portal venous inflow. This results
in a marked increase in cardiac output and so-called hyperdynamic circulation.
The presence of ascites as well as the the progression of oesophageal varices were
associated with higher circulating nitrate levels. The connection between increased
nitric oxide production and the haemodynamic sequelae of portal hypertension is
also apparent in the significant correlation between plasma renin and serum nitrate
levels.
http://www.ncbi.nlm.nih.gov/pubmed/12436367

Nitrates are no longer used as monotherapy for preventing variceal hemorrhage,


and their use in combination with beta-blockers is controversial. The systemic
vasodilatory effects of nitrates exacerbate the hyperdynamic state that is
characteristic of cirrhosis, thereby limiting their use and tolerability in many
patients.
Nonselective beta-blockers, the most commonly used drugs for preventing first
esophageal variceal bleeding, decrease portal pressure by blocking both beta-1 and
beta-2 adrenergic receptors. Beta-1 blockade decreases portal flow by decreasing
the heart rate and cardiac output, while blockade of beta-2 receptors results in
unopposed alpha-adrenergic-mediated vasoconstriction.

80) On Diastolic heart failure
Dear Yin Ling,
What do you know about HfpEF?
This stands for Heart failure with Preserved Ejection Fraction.
Left atrial size vol is increased and measuring the LA volume is impt. Almost all
patients have dilated LA.
Pum hpt is associated.
If pulm hpt is due to HFpEF the LA will be dilated. IF THE LA IS NOT DILATED PLS
THINK OF OTHER CAUSES OF PULM HPT.
More HFpEF IS SEEN in females
LV dilation works like a pump sucking blood from the LA in diastole. The LA
contraction is the final kick to push bld into the LV. HENCE AF IS A DISASTER IN
HFpEF patients.
Suspect in patients with symptoms of heart failure but NO LV DILATION and with
PRESERVED EF OF AT LEAST 50%
Because dilation is limited, EF is limited by lowered diastolic filling. In patients with
severe disease the HR is impt for cardiac output and attempting to slow the heart
rate in an attempt to have more time for diastolic filling makes no difference. A beta
blocker slowing the HR will worsen symptoms in this situation. While Rate control is
impt to maximise diastolic filling, a fine balance is needed as in these patients with
advance non reversible diastolic heart failure, slowing the HR may make symptoms
worse. Clinical evaluation is crucial to determine response.

Treatment: no treatment is shown to reduce M + M


ACEI and ARBs : All neutral outcome
Spironolactone also no sig difference.
Ivabradine: ongoing studies. We await hopefully a positive result.
We need to keep the diuretics. They need it for symptom relieve.
Remember that the diagnosis is a CLINICAL DIAGNOSIS!!!
Symptoms of heart failure but with none of the classic signs of systolic failure must
make us think of it.
A simple ECHO will show preserved EF which indicates the diagnosis.
Pulm HPT is a COMMON RESULT OF THE DISEASE.

Treat ALL identifiable risk factors and control it well. HPT , DM , ISCHAEMIC HEART
DISEASE MUST BE ADDRESSED AGGRESSIVELY.
Remember that A NORMAL ANGIOGRAM MERELY SHOWS EPICARDIAL VESSELS...
THERE MAY BE MICROVASCULAR DISEASE.

THE MAIN PROBLEM AS IT STANDS IS THAT THE PATIENT AFTER A NORMAL ECHO IS
TOLD THAT THERE IS NO HEART FAILURE... MISDIAGNOSIS AND UNDERDIAGNOSIS IS
VERY COMMON.

Emphasis: this condition requires all the medical skills of a 3rd year med student to
take history and do a competent phy exam!!

81) On Calcium supplements
Calcium and VitD. Is it Good or bad?
To use or not to use is the Q. Are we helping or harming?
In nursing homes and Ca deficient popns YES it is helpful.
But in community populations, the data is unclear
AMI and CVA is our concern.
More cvs events and earlier ami is possible at 1000mg Ca supplement
Dly req is only 300 to 400mg
More than 500mg dly is asso with increased risk
The elderly needs even lower doses for maintainance.

Statins and low fat low cholesterol diets reduce VitD absorption.
We need 600mg VitD dly in temperate countries.
In tropical countries we need less.
1200mg dly is no longer recommended.
VitD may either in low or high values increase vascular calcification... a U shaped
response curve is seen
VitD is needed in preg , osteoporosis , elderly.
30mins of sunlight provides this dose.
In pre menopausal women Ca goes to bones while in post menopausal women some
of it goes to the vessels.
Bones and arteries: Patients with osteoporosis ofren suffer from vascular
calcification!! There appears to be some common signalling pathways which gives
rise to this.
In those More than 70yrs old vascular calcification actually decreases paradoxically
Hpt uraemia, dm, lipids, high Ca and VitD increases vascular calcification

82) On Smokers and beta carotene
Now every now and then so rarely, I ask yin ling a Q and she goes "Huh?"
So occasionally I still have something to teach her though this is getting lesser by the
hour.
So I asked her:
"Yin Ling, can a smoker take beta carotene supplements?"
"huh?"
Years after they stopped taking high-dose beta-carotene supplements, a group of
smokers still suffer extra-high rates of lung cancer and death.
The smokers took part in two ill-starred clinical trials testing whether beta-carotene
and vitamin A could prevent lung cancer. Nearly everybody thought it would work.
And they were wrong. Lung cancer, heart disease, and death from all causes shot up
in those who took high-dose beta-carotene. (Abandon all hope ye who thinks that
taking huge amounts of carrots like rabbits will make you perform like a rabbit!)

In the Finnish ATBC study of 29,000 male smokers, 20 mg beta-carotene


supplements taken over six years were linked to lung cancer. In the U.S. CARET study
of more than 18,000 male and female smokers and male asbestos workers, 30 mg
beta-carotene supplements over four years were linked to a 28% higher risk of lung
cancer and a 17% higher risk of deaths from all causes compared with smokers
taking a placebo.
Both studies were halted: ATBC in 1994 and CARET in 1996.
In smokers, the higher risk of lung cancer and death continues even after a person
stops taking beta-carotene. In CARET participants, the relative risk of lung cancer
remained elevated (although not statistically significant) four years after the end of
the intervention.
Most of this risk was seen in female smokers. It's not clear why women have a
higher risk. For that matter, it's not clear why beta-carotene supplements hurt
smokers. The most likely reason is that damaging free radicals in cigarette smoke
may get a boost from megadoses of beta-carotene.
Don't stop getting beta-carotene in your diet, even if you are a smoker. Other
studies show that people who get lots of beta-carotene-rich foods have a lower risk
of cancer and heart disease.
A lifetime of healthy eating is far better than a few years of high-dose vitamins
during middle age.

83) Gynaecomastia
Dear Yin Ling,
how do you differentiate gynaecomastia from just, well, FAT?
Pseudo Gynaecomastia vs Gynaecomastia
Not all breast enlargement in men are caused as a result of excessive breast tissue
growth or gynecomastia. Gynecomastia has its own underlying etiology and definite
characteristics. Pls name 5 causes stat!
Pseudogynecomastia looks much the same as gynecomastia with the exception of
the underlying breast tissue. In the case of pseudogynecomastia, the reason for
breast enlargement is actually fat rather than breast tissue. OK go examine any
overweight male.

In the case of gynecomastia, what leads to the feminization of male breast is the
growth of excessive breast tissue. However, if it is pseudogynecomastia, the
underlying mass is not breast tissue but fat.
Examination:
The patient lies flat on his back with his hands clasped beneath his head. Using the
separated thumb and forefinger, the examiner slowly brings the fingers together
from either side of the breast. In patients with true gynecomastia, a rubbery or firm
mound of tissue that is concentric with the nippleareolar complex is felt, whereas
in patients with pseudogynecomastia, no such disk of tissue is found.
Essentially,fat will be like fat, the soft blubbery thing that we have around our belly.
But in Gynecomastia male breasts have Both fat and gland, a combination of both.
So do not miss it.
Pseudo Gynecomastia is supposed to be fat only. So if you feel any glandular tissue,
its gynaecomastia. Fat tends to be soft, gland tends to be firm. Gland tends to be
located under the nipple and pinching pressure can cause the nipple to be pulled in.

Causes: Decreased metabolism of estrogen - liver cirrhosis, renal failure

Increased production of estrogen relative to testosterone - adrenal tumour,
testicular cancer, pituitary adenoma

Endocrine - hypogonadism hypothyrodism, Graves' disease, acromegaly

Some congenital causes like Kallman syndrome, Klinefelter syndrome

Drug induced which is remembered by SACKED - spironolactone, alcohol, cimetidine,
ketoconazole, estrogen ( for example patient taking anabolic steroids for muscle
building ) and digitalis

Others - puberty and paraneoplastic manifestation of lung cancer ( esp large cell )

The tragedy og modern medicine is clear in the review below. Pls see carefully

A 60-year-old man was referred to the endocrinology clinic for evaluation of
bilateral gynecomastia of 6 months' duration. He reported erectile dysfunction and
decreased libido. On further review of systems, he reported no changes in testicular

size, no history of testicular trauma, no sexually transmitted diseases, no headaches,


no visual changes, and no change in muscular mass or strength. Initial laboratory
assessment showed estrone and estradiol concentrations to be 4-fold increased
above the upper limit of the reference range. Subsequent findings from testicular
ultrasonography; computed tomography of the chest, abdomen, and pelvis; and
positron emission tomography were normal.

"Because of the normal findings from the imaging evaluation, the patient was
interviewed again",

and he described a daily intake of 3 quarts of soy milk. After he discontinued
drinking soy milk, his breast tenderness resolved and his estradiol concentration
slowly returned to normal.

Conclusions: This is a very unusual case of gynecomastia related to ingestion of soy
products. Health care providers should thoroughly review patients' dietary habits to
possibly reveal the etiology of medical conditions.

Can you all believe this!!!!:
"Because of the normal findings from the imaging evaluation, the patient was
interviewed again". Now its LETS SCAN EVERY DAMN ORGAN... Oh it is all Normal...
Come let us talk to the patient!!!

84) On TTP
Dear Yin Ling,
How does simple blood tests caution us of DIC or TTP?
Can you describe these tests and what you expect to see?

Yin Ling: TTP is a microangiopathic Hemolytic anemia.. in other words, its the
clumping of PLTs in the veins which will narrows them causing high shearing forces
when RBC passes through and lead to RBC lyses. Hence the hemolytic anemia.

So with this in mind, we will see low PLT atogether with anemia on CBC. That would
be the first warning. Then looking into the LFT, there will be signs of hemolysis eg
high indirect bilirubin, high LDH (not routinely done tgt with LFT). Rericulocytes will
be raised if u order for one.

The key test that differentiates TTP from DIC will be the coagulation profile, which
will be normal in TTP as consumption of coag factors does not happen like in DIVC.

More test can be send to further confirm, but these are basic tests that ring an
alarm bell.

* sometimes i also see a pictue of TTP in really septic patient, low PLT, anemia,
deranged organ fx with everything raised, but a PBF will lend a hand- TTP will have
schistocytes helmet cells etc, but not in sepsis.

Prof: Talking to Yin Ling over tea today,

Thrombotic thrombocytopenic purpura (TTP) hemolytic uremic syndrome (HUS) is
a thrombotic microangiopathy superficially like DIC, but distinctly different in
pathophysiology; in contrast to DIC, the mechanism of thrombosis is not via the
tissue factor (TF)/factor VIIa pathway.

Results of blood coagulation assays in TTP-HUS are usually normal or near normal. In
TTP-HUS, thrombosis arises from direct platelet activation, usually as a result of
widespread endothelial damage or an inherited or acquired impairment of
ADAMTS13, a protease that normally cleaves von Willebrand factor (vWF), which
results in an ultralarge vWF (ULVWF) that agglutinates/activates platelets, leading to
thrombosis and shearing of red blood cells on the ULVWF.

Thrombocytopenia is present in both DIC and idiopathic thrombocytopenic purpura


(ITP). However, ITP is distinct from DIC in terms of its pathophysiologic mechanism
and does not involve coagulation activation or microangiopathic hemolytic anemia.

A subpopulation of patients who have received heparin develop antibodies against
platelet antigens (PF4), and a diminution of platelet number can result. If the
antibody is particularly reactive with platelets, its direct activation of platelets leads
to thrombosis. In HITTS, although the platelet count is decreased, the plasma
prothrombin time (PT), the activated partial thromboplastin time (aPTT), and the
fibrinogen levels are normal.

Thrombotic Thrombocytopenic Purpura (TTP) is a rare blood disorder with clotting in
small blood vessels of the body, resulting in a low platelet count. In its full-blown
form, the disease consists of the pentad of microangiopathic hemolytic anemia,
thrombocytopenic purpura, neurologic abnormalities, fever, and renal disease.

Neurologic manifestations include alteration in mental status, seizures, hemiplegia,
paresthesias, visual disturbance, and aphasia.
Fatigue may accompany the anemia
Severe bleeding from thrombocytopenia is unusual, although petechiae are
common

We see the presence of microangiopathic hemolytic anemia (schistocytes, elevated
LDH, and indirect hyperbilirubinemia) and thrombocytopenia.
CBC usually reveals a normal or slightly elevated total white blood cell count.
Hemoglobin is moderately depressed at 8-9 g/dL.
Platelet count generally ranges from 20,000-50,000 per microliter.
Some consider schistocytosis the sine qua non for diagnosis.
Lactic dehydrogenase (LDH) and bilirubin are indirect measures of the degree of
hemolysis.
LDH level being in the 1000 IU/L range (normal < 200 IU/L) is not unusual.
Generally, a moderate degree of hyperbilirubinemia (2.5-4 mg/dL) is present, with
the indirect form predominating.

The coagulation studies typically are normal in patients with HUS and TTP, although
some series report patients with slight elevations of both.
D-dimers are indicative of fibrinolysis and thus, thrombin activation, which usually is
normal or mildly elevated in patients with TTP.
Fibrinogen typically is in the high to high-normal range.

These tests are useful in differentiating TTP/HUS from disseminated intravascular


coagulation (DIC), in which most of these coagulation parameters are abnormal.

A platelet count of less than 20 x 10(3)/microL (20 x 10(9)/L) and a PT within 5
seconds of the upper limit of the reference interval had a specificity of 92% for TTP-
HUS. Common readily available laboratory assays in the proper clinical scenario can
help differentiate TTP-HUS from DIC.
Although we tend to distinguish between TTP and HUS, the presenting features are
essentially the same in most adult patients:

In a few patients, neurologic abnormalities are dominant and acute renal failure is
minimal or not present; these patients are considered by some to represent
idiopathic or "classical" TTP.
When acute renal failure is dominant, the disorder is considered by some to
represent HUS.

85) On Dengue (Yin Ling is the one asking the questions here, the student is now
as good if not better than her teacher)
1) Whats Dengue?
its a mosquito borne viral infection - dengue virus- with 4 closely related serotypes .
its vector is our beloved Aedes Aegypti mosquitoes with stripes on its hind legs. But
they are only the vectors! WE, the human beings are the source of the virus for
VIRGIN mosquitoes, we feed them- hence why if resources are adequate, we would
placed mosquitoes netting for our dengue infected patients in the hospital, to
prevent human-mossies-human infection. By the way, the mosquitoes are infected
for life. Poor things.

There are 3 phrases in DF, named febrile, critical and recovery.


Febrile phase is when patient comes with fever.

Critical phase is when fever subsides and patients think that they are healthy. This
phase is called "critical" as vascular permeability increases and platelets start
extravasate to third space together with plasma. Patient is dehydrated with low
platelet, decompensated shock ( tachycardia yet normal BP). Hct starts to rise
( haemoconcentration is when Hct in male>46%, Hct female>40%).
Be careful of third space loss such as pleural effusion, ascites, ankle oedema.


In 2009, WHO released new guidelines on the management of dengue fever which
no longer use the previous classification of dengue hemorrhagic fever and dengue
shock syndrome. Now it is classified into:
1. Dengue without warning signs
2. Dengue with warning signs
3. Severe dengue

Emphasis of the old guidelines was put on bleeding tendency, but in fact signs of
capillary leakage is more worrisome in the management of dengue fever. We might
miss a patient with severe dengue and discharge the patient too early if we are only
looking for hemorrhage or thrombocytopenia.

2) We all know about the 4 serotypes of Dengue, Den 1-4. Why is it so important to
know the existence of these serotype?
CROSS-REACTIVE BUT NOT CROSS PROTECTIVE. This statement underlines what
we understand about the dengue virus. Being infected by one strain of dengue virus
confers a lifetime immunity to THAT strain only. More importantly, CROSS
REACTION will give ANTIBODY DEPENDENT ENHANCEMENT (ADE)- hence the fearful

SECONDARY INFECTION- much much much worse dengue infection the second time
around!!
3) What is the pathophysiology that outlines dengue infection?
The dengue antigen (NS1) forms ag-ab complexes and cause inflammatory reactions,
causing increase of vessels permeability hence third space loss. There is also platelet
activation and platlet aggregation, which cause platelets to decrease. Protein and
albumin decreases as they are lost from vessels. Generalized vasoconstriction will
give a higher diastolic BP in dengue patients.
The cytokines are associated with increased inflammation. Another consequence of
the cytokine storm in dengue virus infection is an increased capillary permeability
syndrome.
Severe dengue occurs most commonly in individuals experiencing a second infection
with a serotype distinct from that of a past exposure. In these so-called heterotypic
infections, the host anamnestic immune response is postulated to mechanistically
contribute to the syndrome of increased capillary permeability that characterizes
severe dengue.

Increased capillary permeability in dengue usually occurs between days 4 and 6 of
illness, is transient, and only occasionally results in hypovolemic shock. At this stage
of the illness, viremia is in steep decline and serum cytokine concentrations of IFN-
and IL-10 are at or near their peak levels .
'Antibody-dependent enhancement'
Basically it is due to host's immune response towards different dengue serotypes. If
you were previously infected with a dengue serotype (say DEN-1), there is a life-long
immunity towards DEN-1 and cross-reactive immunity to the other serotypes for
about 6 - 12 months. Now if you are unfortunately infected with a different serotype
(say DEN-2) after a year, the antibody from the first infection will not be able to
neutralise the second infection due to different serotypes.
Instead, it will enhance the host's immune response towards the virus, with
resultant excessive release of pro-inflammatory cytokines and complement
activation, causing endothelial dysfunction, tissue damage, bone marrow changes
and plasma leakage. Plasma leakage results in increased hematocrit,
hypoproteinemia, pleural effusion (fluid leaking into pleural space), ascites and
reduced plasma volume, leading to hypovolaemic shock (and multi-organ failure).
Bone marrow changes results in defective maturation of megakaryocytes, leading to
thrombocytopenia. The tissue damage results in concomitant activation of
coagulation and fibrinolytic systems, depleting the clotting factors and may also

cause DIC (which may contribute to multi-organ failure). Together, endothelial


dysfunction (vasculopathy), thrombopathy (impaired platelet function and
thrombocytopenia) and coagulopathy results in the abnormalities in haemostasis,
with resultant bleeding tendencies.

4) WHY is it dengue so bad this time around?!
primary infection : NS1Ag positive, igG negative.
secondary infection: NS1Ag positive, IgG positive.
The dengue outbreak is so horrible this time around mostly because we think our
patients are having secondary infections. They come to us with SEVERE DENGUE,
with multiorgan involvements, or they deteriorate really really fast.

Yin Ling: Few tips for dengue management
1) Firstly history on admission -WARNING SYMPTOMS is of utmost importance
(abdominal pain, persistent vomiting which is defined as vomiting more than 3 times
per day and patient is unable to tolerate orally, mucosal bleeding including menses).
Admission is warranted once patient has warning symptoms.
Don't rule out dengue even if the platelet is normal e.g. 180 if history suggestive.
The patient's baseline could be 400.
2) Although not in the warning signs criteria, we also take diarrhoea as a warning
because patient is losing fluid.
3) Remember comorbids eg DM HPT IHD age>65, obesity, pregnancy. They tend to
perform worse!
4) TOUCH the patient! The act of holding the patient hand can save one life. look for
CCTVRU colour, cap refill, temp, pulse volume and rate. Sound the red alert is
patient is tachycardic, small pulse volume, and feels COLD (dont just blame it on the
ED aircon). What is the Urine output like?
Evaluation of hemodynamic state & perfusion is never complete without taking into
consideration the urine output.

Why is monitoring of urine output crucial?
-It reflects renal blood flow, kidneys regulate intravascular volume
-In early shock states, kidneys conserve fluids by reducing urine volume
-In severe shock, NO urine is produced

What is considered ADEQUATE urine output?


-In OPD setting, the patient should be able to drink enough to void at least 4-6 times
per day
-A patient with dengue shock should produce at least 0.5ml/kg/hour of urine (if
urine exceeds this amount, reduce IV fluids). Indwelling catheter will give you the
most accurate measurement

Pitfall?
-Uncontrolled diabetes and hyperglycaemia will produce inappropriately large
amount of urine. This will make the shock worse..

5) How do you know when is patient with chronic hypertension and currently on
beta blockers is in SHOCK? Cold periphery, capillary refill time prolonged with poor
pulse volume. No tachycardia as patient is on beta blocker. Reduced urine output at
later stage when body shutting down kidney.
6) For younger patients- children , the 5th centile of their SBP is 70+(2xage)
7) Maintainence fluid is calculated according to the IDEAL BODY WEIGHT. The IBW is
only used for overweight /obese patients. Thin patients- HIS WEIGHT.
8) Daily review including asking the patients few golden questions- fluid intake,
urination frequency, latest voiding, appetite, and how does he feels ?

9) Review the LFT, LDH also. Hepatitis is a sign of patient not doing too good. CK, and
LDH if deranged and history suggestive think of Lepto
How frequent does Full blood count need to be taken and reviewed?
* FBC looking for lymphopenia and thrombocytopenia, suggesting dengue
infection. We are doing thrice daily FBC reading, and adjust our frequency or FBC
monitoring as pt gets better.

*Liver Function test- dengue infection often involves the liver. ALT and AST tells us
how bad the liver is being affected hence how our pt will perform. AST are often
raised much more than ALT. Remember, ALT which is showing a rising trend and
suddenly a drop- this can mean pt is getting better OR they are going into liver
failure. Send for both the ALT and AST.

*Urea and electrolytes : we would want to monitor the renal function and
electrolytes closely with so much drip use and vomiting and diarrhoea.

*NS1Ag : will be positive in early dengue infection up to the 5th-6th day.



*Dengue IgM and IgG: will be positive after day 5, and igG positive is suggestive of a
secondary infection- bad news.

*Blood Film for Malaria Parasite (BFMP) : presentation of malaria and dengue can be
hard to differentiate.

Other blood investigations if you are suspecting something other than dengue!

10) Patient who works as in army(many of them) ask about jungle training. Be
aware of lepto and scrub typhus and malaria.
11) School holidays. Ask about river swimming - lepto.
12) Critical phase/leaking phase occurs 24-48 hours after deferverscence, when PLT
is decreasing and pt dont feel too well.
Pls dont give IV Fluids too aggressively if patient is able to take his own fluids. 2-3L
per day with good urine output is adequate.
13) Be keen to STOP the IV fluids once patient is able to tolerate orally well without
vomiting on diarrhoea.
14) Encourage coconut water. Fruit juice is generally not encouraged. Also 100 plus
is too sweet. Coconut water is the best, so is barley water or porridge water.
15) Pt is consider to be in the recovery phase once his PLT starts to stabilise (some
might drop a little but not drastic) and pt generally feels BETTER. Careful with the
drips, dont overload.
16) During the recovery phase, HCT may goes up a little and this is usually due to
patient pouring out urine, pitfall here is when we give them more drips because we
still think they are leaking. DONT aggressively correct with IVD if pt is feeling ok!
17) NS1 is v sensitive. We are quite confident that we are seeing a dengue when its
positive. When dengue serology is persistently negative, think of malaria. Think of
leptospirosis. Think of other things.
18) If patient is still having fever in the 5th or 6th days of illness, with a high LDH and
high WBC, consider Hemophagocytic syndrome. Send Se Ferritin. Ask the lab to
dilute it if counts are >2000.
19) in postpartum patients, remember to take the amount of breastfeeding into
account. say a mother feeds 6-8X per day giving her child 6 ounces each time, that
would make up to about 1.2L, which is significant. there has been cases where
patients go into shock from not enough of fluid replacement after breastfeeding.


20) Liver function will often be deranged during deferverscence, and further
worsening may occur when the pt is in recovery. LFT during deferverscence may
help us in our management, and keep monitoring if it's deranged and pt is unwell.
LFT will remain deranged for the next 3 weeks, hence if pt remains well and PLT are
improving, LFT shouldnt affect us in our decision to discharge patient.



Classical rash: Islands of white in a sea of red

NS1 ELISA
The non-structural protein 1 (NS1) of the dengue viral genome has been shown to
be useful as a tool for the diagnosis of acute dengue infections. Dengue NS1 antigen
has been detected in the serum of DENV infected patients as early as 1 day post
onset of symptoms (DPO), and up to 18 DPO.



Dr Hu Mung Chee Dengue associated acalculous cholecystitis is uncommon.
Mild hepatic dysfunction is common in dengue fever, not only in cases of acalculous
cholecystitis. The increase in aminotransferases has been associated with increased
disease severity and might serve as an early indicator of dengue infection.

In a study, liver damage, and consequently increases in amino transferase levels,
were more frequent in patients with DHF. (Nimmannitya 1987; Kuo et al. 1992;
Mohan et al. 2000).

From 1973 to 1982, the observed hepatic involvement in dengue infection in
Thailand and Malaysia was mild and it manifested solely as increase in
aminotransferase levels. But after this period several cases of fulminant hepatitis
with high mortality have been reported (Lawn et al. 2003). Severe haemorrhage,
shock, metabolic acidosis and disseminated intravascular coagulation may
contribute to severe changes in liver. It should be remembered that concommitant
chronic liver disease, alcoholic steatonecrosis and hepatotoxic drug use (e.g.

salicyclates, acetaminophen etc.) during dengue infection may predispose to and


may even increase liver injury.
This is a landmark paper on Vertical transmission of Den3 virus, so VIRGIN
mosquitoes can infect!



Prof: I am concerned that not many doctors are aware of the danger of fluid
overload in DF. Aggressive infusion of iv fluids in the febrile phase is often seen. In
the Critical phase the plasma leakage in the state of fluid overload will lead to
worsening of fluid in the extra vascular space. Subsequently when reabsorption
takes place in recovery phase there can be intra vascular fluid overload.
These are both undesirable
IV Fluids for Dengue Linked to Progression to Severe Disease
Daniel M. Keller, PhD
July 13, 2012 (Bangkok, Thailand) A medical record review of adult patients with
dengue but no evidence at baseline of plasma leakage or significant volume
depletion suggests that such patients might not benefit from intravenous (IV) fluids
if they can absorb oral fluids. In fact, IV fluids might increase the risk for severe
disease.

Alex Cook, PhD, assistant professor at the School of Public Health and the
Department of Statistics and Applied Probability at the National University of
Singapore, told delegates here at the 15th International Congress on Infectious
Diseases that IV fluid administration is an unsettled issue.
He explained that dengue virus infection can be asymptomatic, can cause flu-like
illness, or can result in dengue hemorrhagic fever (DHF) or severe dengue. When
patients in Singapore are admitted to the hospital, they are often badly dehydrated
and get IV fluids. We wanted to know whether the IV fluid makes them better or
worse, he explained.
In an observational cohort study, Dr. Cook and colleagues found that for dengue
patients hospitalized from 2006 to 2008, those receiving more than 1 day of IV fluids
were more likely to progress to severe manifestations. Of the patients presenting
without DHF, 17% of those who received IV fluids went on to develop DHF, whereas
only 8% of those who did not receive IV fluids did. Similarly, of patients presenting
without severe dengue, 9% progressed to severe dengue if they received IV fluids, as
did 2% of those who did not receive IV fluids.
Dengue and its manifestations are a continuum, and some patients will be more
severely affected; "they are more likely to go on to get DHF or severe dengue, and
they are also more likely to be given IV fluids," Dr. Cook said.
The researchers used statistical methods to try to "disentangle" the effect of IV
fluids from the severity of the disease. They reviewed the medical records of 1529
adults with dengue but without DHF or severe dengue on admission to Tan Tock
Seng Hospital in Singapore. Polymerase chain reaction, serologic tests, and other
standard criteria were used to diagnose the dengue virus.
Using a statistical technique (penalized regression method), which adjusted for 98
potential demographic, clinical, and laboratory confounders, the researchers
assessed the effect of maintenance IV fluids on the development of DHF and severe
dengue, and on the length of hospital stay.
Of the 1529 study patients, 1346 were given IV fluids for 2 or more days and 183
were not given IV fluids. The patients getting IV fluids received a median daily
volume of 3500 mL (interquartile range, 3050 to 7000 mL) for a median of 3 days.
Dr. Cook noted that a weakness of the study is that only about 10% of the patients
did not receive IV fluids.
After hospital admission, 248 patients (16%) developed DHF and 122 (8%)
developed severe dengue. Patients who did not receive IV fluids had an adjusted
odds ratio (OR) of 1.0 for developing severe dengue or DHF; patients who received

IV fluids had a 3.6-fold greater risk of developing severe dengue (adjusted OR, 3.6;
95% confidence interval [CI], 1.1 to 12.0; P = .04).
Although the hospital stay for patients who received IV fluids was statistically
significantly longer than for patients who did not, it was "about 10%, just a few
hours, and we don't care much about that," Dr. Cook said. The patients receiving IV
fluids also had a nonsignificant 40% greater risk of developing DHF (OR, 1.4; 95% CI,
0.75 to 2.8; P = .27).
Dr. Cook noted that "it would be valuable to repeat this [analysis] for other cohorts
of patients to see whether the effect generalizes beyond Singapore, and to
potentially do a clinical trial." He suggested that a 2-year single-center randomized
clinical trial with 400 patients in each group (IV fluids or not) would be sufficiently
powered "to demonstrate definitively" whether IV fluid administration increases the
risk of developing severe dengue.
Thomas Grnewald, MD, PhD, assistant professor of internal medicine and head of
the division of infectious diseases and tropical medicine at the Klinikum St. Georg in
Leipzig, Germany, toldMedscape Medical Newsthat he treats European patients
with 1 to 2 L of IV fluid/day for 4 to 6 days without problems.
"Usually patients with dengue fever come to the European clinics in very dry
conditions, and because of the fever they have to get fluids.... It could be different in
Asian countries or in South American countries," he said. "We try not to give them
any antipyretics because of worse outcomes, which we know from malaria and
similar diseases. They prolong the disease and sometimes pose more problems;
patients have hepatitis, they have renal failure, and the antipyretics can increase
these problems." These differences in practice might lead to "bias in the statistics
[Dr. Cook] has shown," he explained.
Dr. Grnewald agrees that a randomized controlled clinical trial is a good idea,
especially in a country with a large dengue burden. There is "no other way to check
whether the fluid overload is really a pathogenic factor, which is iatrogenic, or
whether it is just a bystander and not really statistically significant," he said, adding
that the study should account for comedications, comorbidities, region of practice,
and the population being treated.
Dr. Cook noted that in 2009, before starting the study, he was hospitalized with
dengue and received IV fluid. "When you're in the hospital with dengue, IV fluid
feels great," he said.
15th International Congress on Infectious Diseases (ICID): Abstract 21.007.
Presented June 15, 2012

Principles of fluid therapy can be summarized as follows:



Fluid must be administered orally as much as possible. Intravenous supplementation
is necessary when the patient is not able to take fluids orally (severe vomiting,
prostration) or is in shock.

The rationale in fluid management is to keep enough fluid in the vascular system
during the leakage phase to avoid hypovolemia and while also avoiding overloading
the patient with too much fluid.

The recommended first-line intravenous fluid is crystalloids (0.9% saline).

The rate of intravenous fluid administration should be at stepwise increments or
decrements with at least 46-hourly hematocrit monitoring during the critical
phase. However, in situations of shock, immediate resuscitation with 20 mL/kg
boluses is recommended until blood pressure is recordable.

Colloids (eg, dextran) are recommended as second-line therapy when hypotension is
not responsive to boluses of intravenous crystalloids (with reference to local or
World Health Organization guidance).

A rise in hematocrit indicates further hemoconcentration due to leakage, and hence
a need for more fluids. However a drop in hematocrit may be due to either
convalescence (reabsorption of extravasated fluid) or internal bleeding. If the
patient is still ill and in critical phase with low platelet counts, always suspect
bleeding. Sometimes there may not be any overt external manifestations of
bleeding. In cases of suspected bleeding, the management strategy is transfusion of
fresh whole blood.

Administration of fluids should be guided by frequent monitoring and assessment of
intravascular volume status during the critical stage, and fluids should never be
administered at a constant rate without monitoring.

86) On Bowel prep for colonoscopy in ESRF patients
Over dinner when we were nicely slurping Noodles, keow teow and meat ball
soup, Yin Ling raised the question of bowel enemas! Yes, thats what we talk about
over fine dining.

In a patient who is anuric with end stage renal failure, how does one bowel prep him
for colonoscopy?
Hmmm, thoughts of fluid movement moving back and forth over colonic mucosa
floated in my mind, of dehydration and electrolyte abnormalities, of large quantities
of fluid that needed to be swallowed in gulps and voluminous osmotic diarrhoea.
Which agent to use?
Will phosphate kill?
Will over or dehydration occur?
OK time to consult Prof Google and Good friend Dr Abraham George.
Rule no 1: When in doubt CONSULT!
In 2006 the FDA issued its first warning that patients taking oral sodium phosphate
preparations are at risk for potential for acute kidney injury: mainly due to acute
phosphate nephropathy that can result in renal failure, especially in older adults.
Why Oral Sodium Phosphate Preparations Are Dangerous
First, oral sodium phosphate preparations can cause significant fluid shifts within the
colon resulting in intravascular volume depletion. Second, these preparations can
cause electrolyte disturbances including significant hyperphosphatemia,
hypocalcemia, and hypokalemia. A significant clinically important rise in serum
phosphate can even be seen in elderly patients with normal renal function. Lastly,
phosphate nephropathy may occur due to the transient and potentially severe
increase in serum phosphate combined with volume depletion from the fluid shifts.
Why the Same Holds True for Sodium Phosphate Enemas (aka Fleet enemas)
A fleet enema works as a hyper-osmotic laxative that draws in water into the
gastrointestinal tract. In healthy younger adults, this action shouldn't pose a
problem as the laxative action from a fleet enema occurs relatively quickly, so there
is little absorption of phosphate.
However, What if they are older, frailer, and taking multiple medications, including
some that may slow their bowels down like opioids or those with anticholinergics
properties?
Yaacov Ori and colleagues conducted a retrospective case series of 11 elderly
patients (mean age of 80). Ten of these patients received Fleet enemas for relief of
constipation and one received it as a proctoscopy prep. Three of these patients
received 500-800 mL of sodium phosphate and 8 patients received approximately
250mL (for a comparison, a typical over-the-counter Fleet enema comes in either a

118 and 197 ml dose). Baseline renal function was normal (eGFR by MDRD of
60mL/min) in 4 patients with a range of 25 to 57 mL/min in the other seven.
What they found was that renal function deteriorated in all 11 patients.
Hypotension and extreme hyperphosphatemia was prominent in 8 of these patients.
The serum calcium level was dangerously low in 8 patients. Five patients died. An
autopsy on one patient revealed calcium phosphate calcifications within the renal
tubular lumens.
The Take Home Point - Just Don't Use It
Fleet enemas should be relegated to this list of medications that should generally be
avoided unless your patient is a very robust older adult. We should also teach others
that if you do happen to order a Fleet enema for a hospitalized or nursing home
patient, you should never give a second dose in succession if the first trial fails.
The alternative is safer, Fortran is made from Poly-ethylene Glycol, when consumed,
it is not absorbed systemically. But it needs a lot of fluid to clear everything up.
With chronic kidney disease, Oral Sodium Phosphate should be avoided. For early
chronic kidney disease, and with late chronic kidney disease without dialysis,
patients should be prescribed with Poly ethylene glycol. Thrombosis through
arteriovenous fistulae for hemodialysis could be accompanied with dehydration and
hypotension. Because Poly Ethylene Glycol may expand intravascular volume, the
schedule of dialysis should be adjusted according to the intravascular volume status.
In order to preserve the residual renal function, intravascular volume depletion
should be avoided in patients with peritoneal dialysis.
Thanks yin ling and Dr Abraham for I learnt something useful and important today
over a nice meal while discussing the intricacies of bowel movements.
How I will miss Yin Ling when she goes to Penang!!! Haihhhh.

To all the yin lings out there in this cyber tutorial room,
In conclusion,
As I leave the stage,
Examine from head to toe
Before you dare diagnose
More harm is done
Because you did not look
Than from not knowing what's in the book
Above all do not be hasty proud and spot
Because you think you know a lot
The great clinicians may at one look know
But then you do not know
The great bitter lessons they humbly swallowed
So be diligent and safe
A meticulous groom to the patient bride.

Please realise that the study of clinical medicine is unlike any other schooling you
have gone through before. Here you are called on; you are asked a question; you
answer it. That is why I ask so many Questions.

Why don't the bedside teachers just give you a lecture? Because through the
questions, you learn to teach yourselves. By this method of questioning-answering,
questioning-answering, we seek to develop in you the ability to analyze that vast
complex of facts that constitutes the relationships between health and illnesses.

For the rest of your earthly life as a doctor until you join Hippocrates, Osler and Hwa
Tuo in CPC discussions, you will be dealing with Questions posed to you! What does
this symptom, this sign, this illness mean? What does this lab report imply?

Now, you may think, at times, that you have reached a correct and final answer. You
are assured that this is a delusion on your part, there is always another question;
there is always a question to follow your answer. Yes, you are on a treadmill. Look at
your seniors practising medicine for many years, do you not see this endless
treadmill we walk on? As soon as we think we have solved one patient's puzzle, 10
more appear, et infinitum. By this training process we hope to prepare you for the
real world of crowded wards and endless clinics.

The clinical questions spin the tumblers of your brain. You are on an operating table;
the questions are fingers probing your mind, urging you to think clearly and

rationally. We do brain surgery here. You teach yourselves the facts of medicine and
we train your minds to think like a doctor. The Facts of Medicine is the SCIENCE of
medicine, How to think like a competent doctor is the ART of medicine. You need
both.



To my beloved Medical students, the first and final message is to
pls learn the Art of Diagnosis and deduction well.
This is an art which can only be acquired by long and patient study, nor is life long
enough to allow any mortal to attain the highest possible perfection in it. And we
begin by mastering more elementary problems.

You should consider your brain as like an empty storeroom, and you have to stock it
with goods as you choose. A fool takes in all that he comes across, so that the
knowledge which might be useful to him gets crowded out, or at best is jumbled up
with a lot of other things, so that he has difficulty in laying his hands upon it. Too
much MP3s, KTVs, Candy Crush does exactly this BUT the skilled clinician is very
careful indeed as to what he takes into his storeroom. He will have nothing but the
tools which may help him in doing his work, but of these he has a large assortment
and all in the most perfect order. It is a mistake to think that that this little room has
elastic walls and can distend to any extent. Depend upon it there comes a time
when for every addition of knowledge good, bad or useless, you forgot something
that you knew before. It is of the highest importance, therefore, not to have useless
facts elbowing out the useful ones.

Always approach a patient with an absolutely blank mind, which is always an


advantage. Form no theories before hand, and after a competent history observe
meticulously and then only draw inferences from your observations.
History
Most people, if you describe a train of events to them, will tell you what the results
would be. They can put those events together in their minds, and argue from them
that something will come to pass. There are a few people, however, who, if you told
them a result, would be able to evolve from their own inner consciousness what the
steps were which led up to that result. This ability to work backwards and see the
natural history of the illness in its entirety is essential to a diagnostician. When we
see the entire sequence of events right from the start even though the patient may
have presented to us now 6 months into the disease when a final straw let him to
you, you will see the diagnosis.
There is no step in diagnosis today which is so important and so much neglected as
the art of tracing the entire natural history of the illness from the beginning.
Students today simply have forgotten this crucial step. Always lay great stress upon
it, and practice it till it becomes second nature.
It is a capital mistake to theorize before you have all the evidence.
Insensibly, one begins to twist the physical signs to suit theories, instead of theories
to suit what is seen. It biases the judgment.
The temptation to form premature theories upon insufficient data is the bane of this
noble profession.
Physical Examination
To a great mind, nothing is little. A little splinter haemorrhage means much to an
astute clinician. Let him on first seeing a patient learn to sharpen the faculties of
observation, where to look and what to look for. By a mans finger-nails, by his
hands, by his eyes, by his shins, by the callosities of his feet, by his expression, etc
by each of these things a disease' signature is plainly revealed. That all united should
fail to enlighten the competent inquirer in any case is almost inconceivable.
Never trust to general impressions, but concentrate yourself upon details.
Your method should be founded upon the observation of trifles.
Diagnosis
The most commonplace illnesses when it presents in an uncommon manner is often
the most mysterious, because the mind being dogmatic fails and no deductions may
be drawn. Remember that a Common Illness with an Unconventional presentation is
far more common than a Rare disease in its common presentation. Think of our Old
friends First!

Never guess. It is a shocking habit destructive to the logical faculty.


Observe the small facts upon which large inferences may depend.
When you have eliminated the impossible, whatever remains, however improbable,
must be the truth.
In making your Provisional Diagnosis, always look for possible alternatives, your
Differential Diagnosis, and provide for and against it. The truth is only arrived at by
the painstaking process of eliminating the untrue.
It should be your business as a medical student to know things. To train yourself to
see what others overlook.
To carry the art, however, to its highest pitch, it is necessary that the doctor should
be able to utilize all the facts which have come to his knowledge; and this in itself
implies, as you will readily see, a possession of all knowledge, which, even in these
days of internet and smartphones, is a somewhat rare accomplishment.
Nevertheless we should possess as much knowledge as possible which is likely to be
useful to our work.

A man should keep his little storeroom stocked with all the furniture that he is likely
to use, and the rest he can put away in the garage, where he can get it if he wants it.
Read nothing but the medical news and the standard texts.

The most practical thing that you ever can do in your life would be to balance it
between the wards and the annals of medicine. Everything comes in circles. The old
wheel turns, and the same spoke comes up. Illnesses had come before, and will be
again. Then when you have heard some slight indication of the course of events in a
patient, you should be able to guide yourself by the scores of other similar cases
which should occur to your memory.

When you are stuck, you should put yourself in your teachers place, and try to
imagine how he would proceed under the same circumstances.
It is of the highest importance in the art of diagnosis to be able to recognize, out of a
number of facts, which are incidental and which vital. But
make it a point of never having any prejudices, and of following docilely wherever a
fact may lead you.
Education never ends. It is a series of lessons with the greatest for the last.

WHAT MEDICAL Teachers MAKE



At a social function, a group of professionals were sitting around the dining table
chit chatting. A businessman decided to explore the problem of the poor pay of
teachers. He reasoned,
"Firstly, how much can one MAKE as a teacher? You need to mark assessments,
teach spoilt privileged kids, fill a hundred administrative forms to be financially just
above the water. How much can one make as a doctor teaching medical students?
You guys must be a really self sacrificing lot or be born into wealth to do all these".

"Be honest. What do you make from teaching?"
silence......

The medical school tutor answered "You really want to know what I make? I may be
very strict walking around with a cane in my hand BUT I make young minds work
harder than they ever thought they could. I show them that there is no limit to their
determination to be good.

I mesmerize restless young boys and girls to sit in awe through 120 minutes of
thinking skills on diagnosis and patient care when their attention span used to be no
more than 20mins..
I make them amazed at the beautiful art of clinical diagnosis.
I make them question themselves on every decision they make as to whether it is
the best for the patient.
I make them realise that every illness has a Real human face behind it; they are
patients NOT cases.
I teach them to have respect for their peers and teachers.

I make them feel that all of them can be great doctors if they want to.
I make them read, read, read the science of medicine.
I make them feel it is worth their while to sacrifice dates and parties for their
education.
I make students feel proud to wear a white coat and to be in the lineage of a great
and noble profession.
I make them happy to be in the midst of wards of human suffering, learning to care
and relieve.

Finally, I make them understand that if they use their intellect, work hard, and
follow their hearts, they can all succeed in life, for success is NOT necessarily
measured in dollars and cents, but in how much we can help our fellow men. I teach
them that they must NEVER be Wallpapers staying on the fringe of society but lead,
care and help society".

"And when people enquire what I make, I can hold my head up high ... and tell them
that

I MAKE A DIFFERENCE. Now what do you make?"
Let us not forget our duty as doctors as stated in the Hippocratic Oath to pass on our
skills and knowledge to the next generation, and to treat those who taught us this
art as our parents!

OSLER-ISM
Dear Yin Ling, 14th March 2014

I asked for Osler's help and he replied that Time Management is essential for
successful post graduate studies (and of course undergraduate as well).
The good physician Osler urged setting definite goals, here for you is Part 2A of
MRCP, while methodically planning each day.

Through his concept of day-tight compartments, Osler said to worry less about the
past or the future, but instead focus on the present. He again resonates with the
Buddhist teaching of Mindfulness of the present moment. His method was to set
aside specific hours of each day for writing, (In your case Studying) which is
sacrosanct, NOTHING, NO ONE can disturb those precious hours, while being sure
that he also had time for his interpersonal relationships (which in your case is the
Dinner with my family almost akin to the essential dinners that lawyers have to
participate in at the Inns of Law).


Dear Yin Ling, 16th March 2014

To succeed in your quest, Osler said that you must Find Mentors; both dead and
alive. Osler himself sought many mentors in his life and in tribute, dedicated his
most celebrated book, The Principles and Practice of Medicine, to three of them.

He was a serious student of many great writers. In addition to their studies, his
students were urged to read for half an hour each day from good philosophical
works. Your Thursday night classes are essential!

In addition, Osler had historical mentors long dead who inspired him by their lives
and work and they included William Harvey (1578-1657), and Thomas Sydenham
(1624-1689). Osler sought opportunities to surround himself with medical students.
They were frequently invited to partake in meals at his home in Baltimore.

Osler looked to students to stimulate him and serve as an antidote against
premature senility. These relationships were naturally mutually beneficial.



Dear Yin Ling, 17th March 2014

Osler said to tell you to Be Positive!!

He himself was, by all accounts, an optimist.
In Aequanimatus, Osler urged medical students to choose their path and decide
what type of doctor they were to be. He believed that we could create our own
future and decide what type of life we may live. To each one of you the practice of
medicine will be very much as you make itto one a worry, a care, a perpetual
annoyance; to another, a daily joy and a life of as much happiness and usefulness as
can well fall to the lot of man.

See, your life is in your hands. While we may not be able to change the outside, we
can always change the workings of our minds.

Most positive people give generously, and Osler was no exception. Throughout his
life he gave others what he hadbe it a coat to a man shivering in the cold, tutelage
to a student, or care for a patient. There are many stories of how he befriended and
was of help to others.

One day, Osler, on his way to an Oxford graduation and dressed in academic gown,
was asked to see a small boy with severe whooping cough complicated by
bronchitis. The child would not eat. The nurses and his parents tried to feed him

without success. Osler did not have much time but acted as though he had plenty.
He examined the child briefly, and then sat down at the bedside. He carefully peeled
a peach, coated it with sugar, cut it into small pieces, and offered them to the child
one at a time, telling the boy that it was special fruit. Hurrying off to the ceremony,
he gave the boys father a bleak prognosis but continued to visit the child daily for
the next 40 days. Because the boy had seen him as a magical figure in his academic
regalia, Osler brought his robe and put it on outside the room before each visit. The
child began to improve a few days after the first visit and made a full recovery.

Pls tell me that my students and the doctors around me will similarly act in such a
compassionate manner. Remember the Emperor's 3 Questions!

When is the Most Important TIME? Answer: NOW
Who is the Most Important Person? Answer: The PERSON you are with now
What is the Most Important thing to do? Answer: To do your best at this moment
for the Person in front of you

The Secret of success of a good doctor is not just knowledge and clinical acumen;
the very top secret is Simply this: TO CARE!


Dear Yin Ling, 18th March 2014


Your exam may be a few weeks away but Osler asked to remind you that the exam is
but only a solitary milestone in a long road. Lifelong learning was as important to
Osler as was lifelong teaching. In teaching we become better. In teaching we learn
more than we give.

Osler, like Hippocrates, strongly advocated the necessity to constantly sharpen your
skill of observation, an essential component to becoming a competent physician.
LOOK and LOOK again, what do you see? Even a novice will see that the patient has
got one ear missing but the good clinician sees the subtle variations from normal
that indicates a deviation from the healthy to the diseased state. All Students must
use all of their senses and the greatest sense to develop is OBSERVATION. WE
observe while the patient walk in, while he talks, while he undresses, and while he
climbs on the couch.

And the history is so important, that to Osler we must very carefully Listen to the
patient, he is telling you the diagnosis.


Dear Yin Ling, 19th March 2014

WE can talk about being a good caring doctor till the cows come home but it will
mean nothing. The Truth is not in words but in acts! Faith, talk, speeches without
works is Dead.

Your MRCP exams are Tough and for good reason for the physicians first duty is to
be competent at what he or she professes to be able to do, and to do it consistently
and well. That minimal body of core knowledge is essential for anyone who claims to
diagnose and heal. Bluntly put, acts of benevolent competence is compassion, while
compassion without competence is fraud.

We know of doctors who are kind, polite, warm, and caringyet their actions or
clinical judgment may be wrong. Treatment here is the placebo effect. And there are
diagnostic supermen/women who practice state-of-the-art evidence-based
medicine, but have such poor interactions with the patients, colleagues and staff
that ultimately all is but wasted efforts, and departmental environment, peer
relationship and patient care sabotaged for a simple lack of basic courtesy and
manners essential for relationships.

Clearly being dedicated to lifelong learning, being observant, developing methods,
and being thorough is essential for every healer. In addition, healers must be
mindful that they are taking care of a person and not a disease, that they are leading
a team of colleagues from the specialist to the Houseofficer to the medical student.
What impression are we giving them? What life lessons are we teaching them? One
of Oslers key recommendations was, Never leave the bedside without a word of
encouragement. And this applies to ALL, the patient and the whole team.

Teachers cum clinicians are role models, we hope to be an embodiment of the
humanistic physician imprinting in our charges minds the qualities we deem crucial
to success as a competent compassionate doctor. Osler wrote, The good physician
treats the disease; the great physician treats the patient who has the disease.


Final Words
Dear Yin Ling, 20th March 2014

Today is 20th march. In 20 days you will sit through 3 papers of 3 hours duration, 9
hours of mind squeezing to get that very last drop of medical wisdom out of you.

27 years ago, I sat in the lecture theatre at the Royal College of Physicians and
Surgeons of Glasgow to pass through a similar exam. There were no computers then
but slides projected on the screen and real papers to write on.
I plan to stop the Dear Yin Ling series today. You need to Bi Guan for the next 20
days, to calm the mind down. Your accommodation in Spore has been arranged and
Bro Jerry and his family will love you like their child.

I sign off now, 4000 plus members of this digital classroom wish you well.

Metta,
Prof

Today is 9th May 2014.
Yin Ling has just received her results. She has passed her MRCP pt2 examination.

Today is 3th Sept 2014.
Yin Ling has informed me that she is posted to Penang as a Medical Officer.
I will miss her terribly but I am happy that she has got a posting close to her
family. She is planning to sit for her PACES in a years time.

Vous aimerez peut-être aussi